Exit hesi- Pharm, RN Exit Hesi

¡Supera tus tareas y exámenes ahora con Quizwiz!

A nurse provides teaching regarding vitamin B 12 injections to a client with pernicious anemia. What statement by the client indicates that teaching was understood? "I must take this monthly for the rest of my life." "I should take this vitamin, as needed, when feeling fatigued." "Once my symptoms subside, I can stop taking this vitamin." "I need to have this available for use during exacerbations of anemia."

"I must take this monthly for the rest of my life." Because the intrinsic factor does not return to gastric secretions even with therapy, B 12 injections will be required for the remainder of the client's life. Vitamin B 12 must be taken on a regular basis for the rest of the client's life.

What is the maximum recommended intramuscular dose for medications in preschoolers? 0.5 mL 1.0 mL 1.5 mL 2.0 mL

1.0 mL The maximum recommended intramuscular dose in preschoolers should not exceed 1 mL.

A postmenopausal woman has been administered raloxifene for osteoporosis. Which parameter should be assessed to ensure the efficacy of the drug and thus its continuation of use? Body weight Bone density Calcium levels in urine Esophageal functioning

Bone density An increase in bone density indicates a good therapeutic response to raloxifene. Thus this parameter should be assessed during therapy for osteoporosis. Body weight may or may not increase with bone weight, and this parameter should not be used to assess a positive response to the drug. Calcium levels in the blood indicate response to the therapy rather than calcium levels in urine. Esophageal dysfunction is a contraindication to bisphosphonate therapy.

76. After a client has an enteral feeding tube inserted, the most accurate method for verification of placement is A) Abdominal x-ray B) Auscultation C) Flushing tube with saline D) Aspiration for gastric contents

D) Aspiration for gastric contents The correct answer is A: Abdominal x-ray

What should the nurse monitor when a client is receiving a platelet aggregation inhibitor such as clopidogrel? Nausea Epistaxis Chest pain Elevated temperature

Epistaxis The high vascularity of the nose, combined with its susceptibility to trauma (e.g., sneezing, nose blowing), makes it a frequent site of hemorrhage. Nausea, chest pain, and elevated temperature usually are not associated with anticoagulant therapy.

Famotidine (Pepcid) is prescribed for a client with peptic ulcer disease. The client asks the nurse what this medication does. Which action does the nurse mention when replying? Increases gastric motility Neutralizes gastric acidity Facilitates histamine release Inhibits gastric acid secretion

Inhibits gastric acid secretion Famotidine decreases gastric secretion by inhibiting histamine at H 2 receptors. Increases gastric motility, neutralizes gastric acidity, and facilitates histamine release are not actions of famotidine.

A client with osteoporosis is prescribed raloxifene. What should the nurse monitor in the client? Check serum creatinine Monitor urinary calcium Monitor liver function tests Observe for anxiety and drowsiness

Monitor liver function tests Raloxifene increases the risk for hepatic disease. Therefore the liver function test is monitored in a client who is prescribed this drug. Serum creatinine is checked in a client who is prescribed zoledronic acid. Urinary calcium is monitored in a client who is prescribed calcium supplements. Anxiety and drowsiness is observed in a client who is prescribed risedronate.

A client is receiving doxepin. For which most dangerous side effect of tricyclic antidepressants will a nurse monitor the client? Mydriasis Dry mouth Constipation Breast hypertrophy

Mydriasis Mydriatic action causes dilated pupils, which can precipitate an acute attack of glaucoma, resulting in blindness. Although dry mouth, constipation, and breast hypertrophy are all side effects, none is as serious as mydriasis.

Which drug is used to induce spontaneous abortions? Oxytocin Estradiol Progestins Clomiphene

Oxytocin Oxytocin is used for spontaneous or induced abortions. Estradiol may be given to alleviate postmenopausal hot flashes. Progestins are used for birth control. Clomiphene may be used as a fertility drug.

A nurse is reviewing a plan of care for a client who was admitted with dehydration as a result of prolonged watery diarrhea. Which new prescription will the nurse question? Oral psyllium Oral potassium supplement Parenteral half-normal saline Parenteral albumin

Parenteral albumin Albumin is hypertonic and will draw additional fluid from the tissues into the intravascular space. Oral psyllium will absorb the watery diarrhea, giving more bulk to the stool. An oral potassium supplement is appropriate because diarrhea causes potassium loss. Parenteral half-normal saline is a hypotonic solution, which can correct dehydration.

What will the nurse include when developing a teaching plan for a client receiving digoxin for left ventricular failure? Sleep flat in bed Follow a low-potassium diet Take the pulse three times a day Rest periodically throughout the day

Rest periodically throughout the day Rest decreases demand on the heart and will prevent fatigue. Sleeping with the head slightly elevated facilitates respiration. The client needs potassium. A low-potassium diet when the client is taking digoxin predisposes the client to toxicity and dangerous dysrhythmias. To avoid becoming obsessed with the pulse rate, the client should take the pulse less often; once daily is adequate.

A community nurse questions the parents of a child with chickenpox about relatives or friends who are undergoing specific prolonged medication therapy. What type of medication will put a person at risk if he or she is exposed to the varicella virus? Anticonvulsant Antihypertensive Topical antibiotic Systemic corticosteroid

Systemic corticosteroid Individuals taking steroids have reduced resistance to infection and may become fatally ill if exposed to the varicella virus. Anticonvulsants and antihypertensives do not reduce body resistance; therefore susceptibility is not increased. Topical antibiotics do not affect body resistance because their effect is not systemic.

28. A client with pneumococcal pneumonia had been started on antibiotics 16 hours ago. During the nurse's initial evening rounds the nurse notices a foul smell in the room. The client makes all of these statements during their conversation. Which statement would alert the nurse to a complication? A) "I have a sharp pain in my chest when I take a breath." B) "I have been coughing up foul-tasting, brown, thick sputum." C) "I have been sweating all day." D) "I feel hot off and on."

The correct answer is B: "I have been coughing up foul tasting, brown, thick sputum."

81. When administering enteral feeding to a client via a jejunostomy tube, the nurse should administer the formula A) Every four to six hours B) Continuously C) In a bolus D) Every hour

The correct answer is B: Continuously

6. The client with infective endocarditis must be assessed frequently by the home health nurse. Which finding suggests that antibiotic therapy is not effective, and must be reported by the nurse immediately to the healthcare provider? A) Nausea and vomiting B) Fever of 103 degrees Fahrenheit (39.5 degrees Celsius) C) Diffuse macular rash D) Muscle tenderness

The correct answer is B: Fever of 103 degrees F (39.5 degrees C)

18. The nurse is caring for a client in hypertensive crisis in an intensive care unit. The priority assessment in the first hour of care is A) Heart rate B) Pedal pulses C) Lung sounds D) Pupil responses

The correct answer is D: Pupil responses

32. The nurse is reviewing laboratory results on a client with acute renal failure. Which one of the following should be reported immediately? A) Blood urea nitrogen 50 mg/dl B) Hemoglobin of 10.3 mg/dl C) Venous blood pH 7.30 D) Serum potassium 6 mEq/L

The correct answer is D: Serum potassium 6 mEq/L

The nurse cares for a client who has schizophrenia and is taking chlorpromazine. The nurse instructs the family members to inform the nurse if any adverse effects develop. Which side effects are considered late extrapyramidal side effects? Tremors Restless movements Muscle spasms of neck Worm-like tongue movements

Worm-like tongue movements Chlorpromazine is a first-generation antipsychotic drug that may cause extrapyramidal side effects. Late extrapyramidal side effects include fine, worm-like tongue movements. Tremors, restless movements, and muscle spasms of the neck, back, tongue, or face are early extrapyramidal side effects.

A client develops a seizure disorder as a result of a traumatic fall. When the client returns to the clinic for a routine visit, the client states, "I have not had a seizure in 2 years. When can I stop taking my antiseizure medications?" What is the nurse's best response? "A gradual reduction in seizure medication may be considered." "You will require medication for the rest of your life." "Enough time has passed since the last seizure. The medication probably will be discontinued at this visit." "A minimum of 10 years without seizures is necessary before discontinuation of medications is considered."

"A gradual reduction in seizure medication may be considered." Specific protocols are designed to gradually reduce the dosage of antiseizure medications after a client is seizure free, provided the electroencephalogram is within acceptable limits. The client is monitored for seizure activity because recurrence is greatest within the first year after drug withdrawal. Depending on the status of the client, antiseizure medications may not be necessary for life. Medications must be withdrawn slowly to prevent an abrupt reduction in serum drug levels, which may precipitate a seizure. The response "A minimum of 10 years without seizures is necessary before discontinuation of medications is considered" indicates too long a time.

A client develops a gallstone that becomes lodged in the common bile duct. An endoscopic sphincterotomy is scheduled. The client asks the nurse what will be done to prevent pain. What should the nurse reply? "All you'll need is an oral painkiller." "Epidural anesthesia usually is given." "You will get a local injection at the site." "An intravenous sedative usually is administered."

"An intravenous sedative usually is administered." An intravenous sedative usually is administered to produce effective sedation (conscious sedation) for the procedure. An oral analgesic is insufficient for this procedure. Epidural anesthesia is not necessary. A local anesthetic is insufficient for this procedure.

The mother of a 7-month-old infant who becomes irritable when teething tells the nurse, "My aunt said to wipe my baby's gums with wine to ease the pain." What is the best response by the nurse? "You can try the wine, but be sure it's diluted." "Your aunt means well, but that's not a good idea." "The wine will help kill the pain, but don't use it too often." "An over-the-counter topical gel can be used, but make sure it's for teething."

"An over-the-counter topical gel can be used, but make sure it's for teething." Providing information is a nonjudgmental way to address unsafe child care practices. There are safe over-the-counter analgesic products specifically formulated to ease the discomfort of teething. Alcohol ingestion is contraindicated and illegal for all children. Being judgmental about the aunt's approach may close communication; the nurse should offer acceptable alternatives.

A client with rheumatoid arthritis takes aspirin routinely to reduce pain. The client asks whether it is the arthritis, the aspirin, or some other ear problem that causes the bilateral ear buzzing the client is now experiencing. What is an appropriate nursing response? "The ringing in your ears is a sign of an ear infection." "Aspirin may have caused some nerve damage in your ear." "Accumulation of ear wax causes ringing in the ears." "Your symptoms are an expected response to the aging process."

"Aspirin may have caused some nerve damage in your ear." Aspirin may damage the eighth cranial (acoustic) nerve, causing ringing in the ears and impaired hearing. Pain, not ringing in the ears, is a sign of otitis media. Diminished hearing, not ringing, occurs because of mechanical obstruction of the outer ear. Aging may cause decreasing acuity in the extremes of pitch, but it does not cause ringing in the ears.

The nurse is caring for a client with tuberculosis. Which suggestions from the nurse will be beneficial for the client? "Take the daily dose during daytime." "Avoid exposure to any inhalation irritants." "Eat foods that are rich in protein, vitamins C and B." "Cover the mouth and nose with a tissue when coughing or sneezing." "Avoid sputum specimens for 2 to 4 weeks once drug therapy is initiated."

"Avoid exposure to any inhalation irritants." "Eat foods that are rich in protein, vitamins C and B." "Cover the mouth and nose with a tissue when coughing or sneezing." A client with tuberculosis should avoid exposure to any inhalation irritants because these can cause further lung damage. To increase physical stamina, the client should eat a well-balanced diet that includes foods that are rich in iron, protein, and vitamins C and B. While coughing or sneezing, the client should cover the mouth and nose with a tissue to prevent spread of infection. A client with tuberculosis should take the daily dose at nighttime to prevent nausea. Sputum specimens are usually needed every 2 to 4 weeks once the drug therapy is initiated. When the results of three consecutive sputum cultures are negative it indicates that the client is no longer infectious.

What instructions about the use of nitroglycerin should the nurse provide to a client with angina? "Identify when pain occurs, and place two tablets under the tongue." "Place one tablet under the tongue, and swallow another when pain is intense." "Before physical activity, place one tablet under the tongue, and repeat the dose in 5 minutes if pain occurs." "Place one tablet under the tongue when pain occurs, and use an additional tablet after the attack to prevent recurrence."

"Before physical activity, place one tablet under the tongue, and repeat the dose in 5 minutes if pain occurs." Anginal pain, which can be anticipated during certain activities, may be prevented by dilating the coronary arteries immediately before engaging in the activity. Generally one tablet is administered at a time; doubling the dosage may produce severe hypotension and headache. The sublingual form of nitroglycerin is absorbed directly through the mucous membranes and should not be swallowed. When the pain is relieved, rest generally will prevent its recurrence by reducing oxygen consumption of the myocardium.

The nurse is comparing Meso-2,3-dimercaptosuccinic acid (DMSA) and British antilewisite (BAL) as chelating agents to be used as treatment for lead poisoning. Which statements indicate correct knowledge regarding chelating agents? "DMSA is given orally while BAL is given intramuscularly." "Neither DMSA nor BAL can be given in conjunction with iron." "Both DMSA and BAL are given in repeated doses over several days." "The client's lead level should be over 70 mcg/dL for both DMSA and BAL." "DMSA is contraindicated in clients with peanut allergies but BAL carries no such warning."

"DMSA is given orally while BAL is given intramuscularly." "Both DMSA and BAL are given in repeated doses over several days." DMSA is given orally. The capsule can be opened and sprinkled on a small amount of food or can be swallowed whole. BAL is administered as a deep intramuscular injection. DMSA is given over a 19-day course of treatment, and BAL is also administered in repeated doses over several days. BAL should not be given in conjunction with iron, while DMSA can be used in conjunction with iron. BAL is used for severe lead toxicity, with lead levels greater than or equal to 70 mcg/dL. DMSA is used in lead levels of 45 to 69 mcg/dL and an absence of symptoms. BAL is contraindicated in children with peanut allergies, but DMSA carries no such warning.

The nurse is preparing a client who is on metformin therapy and is scheduled to undergo renal computed tomography with contrast dye. What does the nurse anticipate the primary healthcare provider to inform the client regarding the procedure? "Discontinue metformin 1 day prior to procedure." "Discontinue metformin a half-day prior to procedure." "Discontinue metformin 3 days following the procedure." "Discontinue metformin 7 days following the procedure."

"Discontinue metformin 1 day prior to procedure." Metformin can react with the iodinated contrast dye that is given for a renal computed tomography (CT) and cause lactic acidosis. Therefore the nurse anticipates an instruction that the client should discontinue the metformin 1 day before the procedure. Stopping the metformin a half-day before the renal CT may not reduce the risk of lactic acidosis. The client is advised to discontinue the metformin for at least 48 hours after the procedure. It is not necessary to discontinue metformin for 3 to 7 days after a renal CT with contrast media.

A 6-year-old child is about to have an intravenous catheter inserted. What is the nurse's best response when the child starts to cry? "Do you want the IV in your right or left arm?" "Do you want me to come back in a little while?" "The needle will hurt for a second; don't be afraid." "The medication in the IV will make you feel better."

"Do you want the IV in your right or left arm?" Giving the child a choice provides a sense of control over the fearful situation. Children need assistance in coping and should not be left alone when afraid. Six-year-old children fear bodily harm, especially disfigurement. Fears should be acknowledged, not minimized. The child is too fearful to care about the effect of the medication.

A client with diabetes asks how exercise will affect insulin and dietary needs. What information does the nurse share about insulin and exercise? "Exercise increases the need for carbohydrates and decreases the need for insulin." "Exercise increases the need for insulin and increases the need for carbohydrates." "Regular physical activity decreases the need for insulin and decreases the need for carbohydrates." "Intensive physical activity decreases the need for carbohydrates but does not affect the need for insulin."

"Exercise increases the need for carbohydrates and decreases the need for insulin." Exercise increases the uptake of glucose by active muscle cells without the need for insulin; carbohydrates are needed to supply energy for the increased metabolic rate associated with exercise. The need for insulin is decreased.

A toddler with hemophilia A is receiving factor VIII. The mother asks the nurse, "If my child hurts himself or herself, I'll give 2 children's ibuprofen. Is that right?" How will the nurse respond? "That's right. Ibuprofen will ease the pain." "Give your child acetaminophen. Ibuprofen may cause bleeding." "No. I'll explain why your child isn't allowed pain medications." "You seem concerned about giving medications to your child."

"Give your child acetaminophen. Ibuprofen may cause bleeding." The parent is asking a specific question that should be answered by the nurse. Ibuprofen is contraindicated because it interferes with platelet function and may cause more bleeding; therefore an analgesic such as acetaminophen should be administered because it does not interfere with coagulation. Analgesics are permitted, provided they do not have anticoagulant effects.

Hydrochlorothiazide, a thiazide diuretic, has been prescribed for a client with hypertension. The client reports hearing that furosemide is more effective and requests a prescription change. How should the nurse respond? "HCTZ has fewer side effects." "HCTZ does not cause dizziness." "HCTZ is only taken when needed." "HCTZ does not cause dehydration."

"HCTZ has fewer side effects." Side effects from thiazides generally are minor and rarely result in discontinuation of therapy. Dizziness is a side effect of all diuretics. There is a potential for dehydration with all diuretics. All diuretic medications are taken regularly as directed.

Dysmenorrhea is suspected in a client with symptoms including backache, diarrhea, syncope, and headache. After reviewing the data, the primary healthcare provider decides not to prescribe diclofenac. Which of these statements made by the client support this decision? "I am allergic to aspirin." "I am allergic to penicillin." "I have a history of kidney stones." "I have a history of bleeding ulcers." "I have a history of deep vein thrombosis."

"I am allergic to aspirin." "I have a history of bleeding ulcers." "I have a history of deep vein thrombosis." Backache, diarrhea, syncope, and headache are symptoms of dysmenorrhea. A nonsteroidal antiinflammatory drug, such as diclofenac, is the primarily prescribed drug, but it should not be given to a client who is allergic to aspirin. The drug is also contraindicated in clients with a history of bleeding ulcers. Because anticoagulants may be prescribed to the client with deep vein thrombosis, diclofenac use is contraindicated in a client with such a history. Penicillin allergy may not contraindicate the use of diclofenac. Renal calculi also may not contraindicate the use of diclofenac. Therefore a client with a history of renal calculi can likely be safely prescribed diclofenac for dysmenorrhea.

A client with glaucoma is receiving a carbonic anhydrase inhibitor. Which statement made by the client will require the nurse to notify the primary healthcare provider? "I have asthma." "I use contact lenses." "I am allergic to sulfonamides." "I have been taking phenelzine medication for three months."

"I am allergic to sulfonamides." Carbonic anhydrase inhibitors are used for the treatment of glaucoma. These drugs are similar to sulfonamides, and if a client is allergic to the sulfonamides, they may have a chance of developing an allergy. Beta-adrenergic blockers such as betaxolol hydrochloride and carteolol are used for treating glaucoma. These drugs, when absorbed systemically, may lead to constriction of the pulmonary smooth muscles and narrow airway. Clients who wear contact lenses is not an issue with carbonic anhydrase inhibitors; however, if using adrenergic agonists clients should not use the eyedrops with the contact lenses in place and should wait 15 minutes after using the drug to put in the lenses. Clients taking antidepressants such as phenelzine should not be prescribed adrenergic agonists because it may lead to a hypertension crisis when taken together.

A client who has been taking the prescribed dose of zolpidem for 5 days returns to the clinic for a follow-up visit. When interviewing the client, the nurse identifies that the medication has been effective when the client makes which statement? "I have less pain." "I have been sleeping better." "My blood glucose is under control." "My blood pressure is coming down."

"I have been sleeping better." Zolpidem is a sedative-hypnotic that produces central nervous system depression in the limbic, thalamic, and hypothalamic areas of the brain. Zolpidem is not an analgesic, antidiabetic, or antihypertensive medication.

A nurse is counseling a client with tuberculosis regarding isoniazid (INH) therapy. Which statement made by the client indicates the nurse needs to follow up? "I should take a multivitamin supplement daily." "I should take the medication 1 hour before eating." "I should immediately report to the primary healthcare provide if my skin and eyes appear yellow." "I should apply sunscreen and wear sun-protective clothing while going outside."

"I should apply sunscreen and wear sun-protective clothing while going outside." Applying sunscreen needs to be followed up since this is a misconception and needs to be corrected. INH is a first-line medication used in the treatment of tuberculosis. This medication is not a photosensitive medication. All the rest are correct statements and require no follow up. Clients taking INH may have low levels of vitamin B complex; therefore, the client should take a daily supplement to prevent peripheral neuropathy. The client should take the medication 1 hour before meals because the presence of food may prevent the absorption of the medication from the gastrointestinal tract. Yellow discoloration of the skin and eyes should be immediately reported because it may indicate medication-induced liver toxicity or failure.

The nurse is caring for an infant who has been prescribed nystatin for oral thrush. The nurse teaches the mother how to prevent aggravation of the condition. Which statements by the mother indicate the need for further teaching? "I should rinse the infant's mouth with plain water after feeding." "I should boil the pacifier for at least 20 minutes on alternate days." "I should apply the medication at least 20 minutes before feeding." "I should apply the medication to the infant's oral cavity four times a day." "I should boil the reusable nipples for at least 5 minutes after washing."

"I should boil the pacifier for at least 20 minutes on alternate days." "I should apply the medication at least 20 minutes before feeding." "I should boil the reusable nipples for at least 5 minutes after washing." Nystatin is used to treat oral thrush in infants. Boiling the pacifier for 20 minutes on alternate days is inadequate because daily boiling of the pacifier is the best way to ensure efficient sterilization and killing of pathogens. Therefore, pacifiers should be boiled daily for at least 20 minutes. Nystatin should be administered to the child after feeding, not before feeding. Reusable nipples should be boiled at least 20 minutes after washing to remove spores, which are heat-resistant. Rinsing the infant's mouth after feeding with plain water reduces the risk of infection in the infant. Nystatin should be applied four times a day to the infant's oral cavity to ensure effective drug action.

A registered nurse teaches a nursing student about considerations for administering medication in infants. Which statement of the nursing student indicates a need for additional learning? "I should administer nasal drops 20-30 minutes before a feeding." "I should pull the ear pinna up and back while administering ear drops." "I should wait until the infant stops crying for administering oral medication." "I should restrain the head and place an eye drop at the corner near the nose if the infant is uncooperative."

"I should pull the ear pinna up and back while administering ear drops." The ear pinna should be pulled down and back while administering eardrops in infants. Pulling the pinna up and back is recommended for an adult or a child older than 3 years of age. Nasal drops should be administered 20 to 30 minutes before a feeding because potential congestion caused by nasal medications may make it difficult for the infants to suck. If the infant is crying, wait until he or she calms to prevent medication aspiration. Infants often squeeze their eyes tightly shut to avoid eye drops. Therefore to administer drops in an uncooperative infant, the infant's head should be gently restrained and the drops should be placed at the corner nearest the nose.

After teaching the client about the precautions to be taken during bisphosphonate therapy, the nurse is evaluating the statements of the client. Which statement made by the client indicates the need for further teaching? "I should take the medication with water." "I should take the medication with a meal." "I should stop taking the medication abruptly." "I should remain upright for at least 30 minutes after taking the medication."

"I should take the medication with a meal." Bisphosphonates have the potential to cause erosive esophagitis and should therefore be taken with a large glass of water and the client should remain upright for at least 30 minutes after taking the medication to facilitate passage through the esophagus. They are poorly absorbed in the oral formulation, so should be taken on an empty stomach, first thing in the morning. Bisphosphonates should not be stopped abruptly unless on the orders of the primary healthcare provider.

The nurse provides discharge medication education to a client who has been switched from a prescription for heparin to a prescription for warfarin sodium. Which client statement indicates to the nurse that teaching was effective? "I will avoid taking aspirin and NSAIDs." "I will avoid exercise and will spend most of the day working at my desk." "I will need to have regular complete blood counts to guide warfarin dosage." "Before going to the dentist, I will ask my healthcare provider for antibiotics."

"I will avoid taking aspirin and NSAIDs." Acetaminophen should be used when an analgesic is required because it does not interfere with platelet aggregation. Acetylsalicylic acid ( aspirin) should be avoided because it interferes with platelet aggregation. Immobility causes venous pooling and can predispose the client to deep vein thrombosis. Antibiotics are not necessary when going to the dentist; this is done when clients have cardiac problems, such as rheumatic fever or cardiac surgery. A prothrombin time (PT) or international normalized ratio (INR), not a complete blood count, needs to be done periodically.

A client with metastatic melanoma is being treated with interferon gamma 1b. The nurse concludes that the teaching about this drug is understood when the client makes which statement? "I will increase my fluid intake to several quarts (liters) every day." "I need to discard any reconstituted solution at the end of the week." "I can continue driving my car as before as long as I have the stamina." "I should be able to continue my usual activity while taking this medication."

"I will increase my fluid intake to several quarts (liters) every day." Increasing fluid intake to several quarts (liters) every day helps flush the kidneys and prevent nephrotoxicity, especially during the early phase of treatment. Reconstituted solution can be stored in the refrigerator for one month. Confusion, dizziness, and hallucinations are side effects of this drug; the client should avoid hazardous tasks, such as driving or using machinery. Activity may have to be altered because fatigue and other flulike symptoms are common with this drug.

A registered nurse provides dietary instructions to a client who is prescribed isocarboxazid for depression. Which statements made by the client indicates a need for further education? "I will limit my intake of fish." "I will limit my intake of yogurt." "I will limit my intake of bananas." "I will limit my intake of beer." "I will limit my intake of chocolate."

"I will limit my intake of fish." "I will limit my intake of yogurt." "I will limit my intake of beer." If a client is unresponsive to other antipsychotic drugs, isocarboxazid (a monoamine oxidase inhibitor) is used to treat depression. Clients taking isocarboxazid should avoid foods high in tyramine and should limit the intake of foods that contain a moderate amount of tyramine content because these foods may cause a hypertensive crisis. Fish, yogurt, and major domestic beers are low in tyramine content and should not be avoided as they generally do not cause hypertensive crisis. Large amounts of bananas or chocolate should be avoided as they can cause a reaction.

A nurse at a women's health clinic confirms that client teaching regarding the use of an oral contraceptive is understood when the client makes which statement? "I can stop the pill and try to get pregnant right away." "I may miss two periods and not worry about being pregnant." "I will take my pill at the same time every day." "I am so glad we won't have to use condoms even if I miss just one pill during the month."

"I will take my pill at the same time every day." Taking the pill at the same time every day makes it more effective, and the client should be instructed to do so. A woman should wait 2 to 3 months after stopping the oral contraceptive pill before attempting to become pregnant. If two consecutive menstrual cycles are missed, the client should stop the contraceptive pill and perform a pregnancy test. The client should use a barrier method of contraception for the first month of pill use and when a pill is missed to help prevent conception.

A healthcare provider prescribes inhaled corticosteroids for a 6-year-old child with asthma. The nurse concludes that the mother understands the teaching about the side effects of this medication when the mother makes which statement? "I'll watch for frequent urination." "I'll check for white patches in the mouth." "I'll be alert for short episodes of not breathing." "I'll monitor for an increased blood glucose level."

"I'll check for white patches in the mouth." Oral candidiasis is a potential side effect of inhaled steroids because of steroids' antiinflammatory effect; the child should be taught to rinse the mouth after each inhalation. Frequent urination is not a side effect of steroid therapy. Apneic episodes are not a side effect of steroid therapy. Hyperglycemia is not a side effect of inhaled steroid therapy; it may occur when steroids are administered for a systemic effect.

Which instructions should the nurse include in the teaching plan for a client with hyperlipidemia who is being discharged with a prescription for cholestyramine? "Increase your intake of fiber and fluid." "Take the medication before you go to bed." "Check your pulse before taking the medication." "Contact your healthcare provider if your skin or sclera turn yellow."

"Increase your intake of fiber and fluid." Fiber and fluids help prevent the most common adverse effect of constipation and its complication: fecal impaction. The medication should be taken with meals. The pulse is not affected. Cholestyramine binds bile in the intestine; therefore, it reduces the incidence of jaundice.

A client with tuberculosis asks the nurse why vitamin B 6 (pyridoxine) is given with isoniazid. What explanation should the nurse provide? "It will improve your immunologic defenses." "The tuberculostatic effect of isoniazid is enhanced." "Isoniazid interferes with the synthesis of this vitamin." "Destruction of the tuberculosis organisms is accelerated."

"Isoniazid interferes with the synthesis of this vitamin." Isoniazid often leads to vitamin B 6 (pyridoxine) deficiency because it competes with the vitamin for the same enzyme; this deficiency most often is manifested by peripheral neuritis, which can be controlled by regular administration of vitamin B 6. Vitamin B 6 does not improve immune status. Pyridoxine does not enhance isoniazid effects. Pyridoxine does not destroy organisms.

A 7-year-old boy with a diagnosis of attention deficit-hyperactivity disorder (ADHD) is receiving methylphenidate. His mother asks about its action and side effects. What is the nurse's initial response? "This medicine increases the appetite." "This medicine must be continued until adulthood." "It is a short-acting medicine that must be given with each meal." "It is a stimulant that has a calming effect on children with your son's disorder."

"It is a stimulant that has a calming effect on children with your son's disorder." Although the exact mechanism is unknown, clinical improvements have been reported with sympathomimetic amines such as methylphenidate. After the purpose and action of the drug are explained, the nurse should review side effects with the parent. The appetite of a child taking methylphenidate usually diminishes. The child should be medicated for as short a period as possible. Each child is evaluated individually. The duration of methylphenidate is 3 to 6 hours, or 8 hours with the extended-release form.

The nurse is preparing to administer an intramuscular injection to a preschool-age client. Which is the most appropriate statement by the nurse prior to this procedure? "We will give you your shot when your mommy comes back." "This is a magic sword that will give you your medicine and male you all better." "It is all right to cry. After we are done, you can go to the box and pick out your favorite sticker." "I will wipe your skin with a magic wipe and then hold the needle like this and say 'one, two, three, go' and give you your shot. Are you ready?"

"It is all right to cry. After we are done, you can go to the box and pick out your favorite sticker." The most appropriate response by the nurse is to acknowledge the child's feelings and allow the child to pick out a sticker at the conclusion of the injection. Waiting for the mother to come back would be inappropriate because toddlers do not have an understanding of time. The nurse should not make statements that are not true and might confuse the child. Giving elaborate descriptions and using colorful language are inappropriate because the instructions are unclear and lengthy.

A nurse is teaching a client about ampicillin that has been prescribed for a severe infection. Which statement indicates to the nurse that the client needs further teaching? "I should report any problems with my hearing." "I may be required to get additional blood tests." "It is okay for me to stop taking this medication after I improve." "If I develop a fever, I will notify my primary healthcare provider."

"It is okay for me to stop taking this medication after I improve." It is most important for the client to complete the antibiotic prescription to prevent the development of antibiotic-resistant bacteria. Ototoxicity is an adverse effect of aminoglycoside antibiotics such as gentamicin. Blood tests for toxicity may be required. Because the client has an infection, it is important to report temperature elevation.

A registered nurse is teaching a nursing student about the various mechanisms that antiviral drugs use to control human immune deficiency (HIV) infections. Which statement made by the nursing student needs correction? "Maraviroc blocks CCR 5 receptors on CD4+ T-cells." "Saquinavir blocks the fusion between HIV and the host cell." "Zidovudine acts as a counterfeit base for reverse transcriptase." "Nelfinavir prevents the breakup of viral protein strand into smaller fragments."

"Saquinavir blocks the fusion between HIV and the host cell." Saquinavir is a protease inhibitor that works by blocking the HIV enzyme protease; it is not a fusion inhibitor. Saquinavir prevents the viral protein strand from being lysed into active smaller fragments. Entry inhibitors such as maraviroc block CCR 5 receptors on CD4+ T-cells. Zidovudine is a nucleoside reverse transcriptase inhibitor that acts as a counterfeit base for the HIV enzyme reverse transcriptase. Protease inhibitors such as nelfinavir prevent the breakup of viral protein strand into smaller active viral particles.

The healthcare provider prescribes neostigmine for a client with myasthenia gravis. The nurse evaluates that the client understands the teaching about this drug when the client makes what statement regarding drug management plans? "Keep the drug in a container in the refrigerator." "Take the drug at the exact time that is listed on the prescription." "Plan to take the drug between meals to promote absorption." "Expect that the onset of the action of the drug will occur several hours after I take it."

"Take the drug at the exact time that is listed on the prescription." Neostigmine should be taken as prescribed, usually before meals, to limit dysphagia and possible aspiration. Keeping neostigmine refrigerated is not necessary; it may be kept at room temperature. Neostigmine should be taken with milk to prevent gastrointestinal irritation; usually it is taken about 30 minutes before meals. The onset of the action of neostigmine occurs 45 to 75 minutes after administration; the duration of its action is 2.5 to 4 hours.

A client with tuberculosis is prescribed isoniazid. What statements should the nurse tell the client? "Take the drug on an empty stomach." "Report any changes in vision to your primary healthcare physician." "Take daily multiple vitamins that contain B-complex." "Wear protective clothing when going outdoors during the day." "Report darkening of the urine or a yellowish skin discoloration."

"Take the drug on an empty stomach." "Take daily multiple vitamins that contain B-complex." "Report darkening of the urine or a yellowish skin discoloration." Isoniazid should be taken on an empty stomach because food prevents absorption of the drug. Multiple vitamins that contain the vitamin B-complex should be taken along with isoniazid because the drug depletes vitamin B. A client on isoniazid should report darkening of the urine and yellowish skin discoloration because these conditions are signs of liver toxicity. A client on ethambutol should be taught to report changes in vision. A client on pyrazinamide is instructed to wear protective clothing if he or she will be exposed to sunlight.

An 8-year-old girl who is receiving methotrexate and undergoing cranial radiation is very weak. Her mother asks the nurse whether she may give her daughter vitamins. How does the nurse respond? "That's an excellent idea. I'll try to get a prescription for her." "Unfortunately, vitamins won't make her feel any better now." "That won't be possible. Vitamins interfere with the action of methotrexate." "After we receive the laboratory reports, your daughter will be getting vitamins."

"That won't be possible. Vitamins interfere with the action of methotrexate." Many vitamin supplements contain folic acid, which negates the action of methotrexate, a folic acid antagonist. Vitamin therapy is contraindicated, so the nurse would not try to obtain a prescription, and vitamins will not be prescribed after the lab reports have come back. Although vitamins contribute to well-being, stating that the client won't feel any better does not answer the question.

After reviewing the immunization record of a 4-year-old child with acute bronchitis and mild febrile illness, the nurse notices the need for a consecutive vaccination of DTaP. What advice should the nurse give to the child's mother? "The child needs the DTP vaccine." "The child needs to receive another Tdap vaccine." "The child needs immediate immunization against pertussis." "The child needs the Tdap vaccine after the acute bronchitis has subsided."

"The child needs to receive another Tdap vaccine." The nurse should advise the mother that the child needs to receive the consecutive dose of Tdap in accordance with the immunization schedule. The DTP vaccine should not be administered when the child's DTaP vaccination has already been started. Children who are not fully immunized against pertussis do not receive a single dose of Tdap; the nurse should not advise immediate immunization against pertussis. Tdap may be administered in the presence of acute bronchitis associated with mild febrile illness.

Ampicillin 250 mg by mouth every 6 hours is prescribed for a client who is to be discharged. Which statement indicates to the nurse that the client understands the teaching about ampicillin? "I should drink a glass of milk with each pill." "I should drink at least six glasses of water every day." "The medicine should be taken with meals and at bedtime." "The medicine should be taken one hour before or two hours after meals."

"The medicine should be taken one hour before or two hours after meals." Ampicillin is a form of penicillin that should be given on an empty stomach; food delays absorption. The response "I should drink a glass of milk with each pill" is incorrect; opaque liquids, such as milk, delay absorption of this drug. The response "I should drink at least six glasses of water every day" is not necessary; however, it is appropriate with sulfonamides to prevent crystalluria. The response "The medicine should be taken with meals and at bedtime" is incorrect; food delays absorption of this drug.

A client asks the nurse how psychotropic medications work. How does the nurse reply? "These medications decrease the metabolic needs of your brain." "These medications increase the production of healthy nervous tissue." "These medications affect the chemicals used in communication between nerve cells." "These medications regulate the sensory input received from the external environment."

"These medications affect the chemicals used in communication between nerve cells." Most psychotropic medications affect neurotransmitters such as dopamine and norepinephrine, which enter the synapses between neurons, allowing them to signal each other. Psychotropic medications do not work by changing the metabolic needs of the brain. Psychotropic medications do not increase the production of nervous tissue. Although there may be some effect on sensory input, this is because of the change in neurotransmitters.

A healthcare provider prescribes an antibiotic intravenous piggyback twice a day for a client with an infection. The healthcare provider prescribes peak and trough levels 48 and 72 hours after initiation of the therapy. The client asks the nurse why there is a need for so many blood tests. What reason does the nurse provide? "They determine adequate dosage levels of the drug." "They detect if you are having an allergic reaction to the drug." "The tests permit blood culture specimens to be obtained when the drug is at its lowest level." "These allow comparison of your fever to when the blood level of the antibiotic is at its highest."

"They determine adequate dosage levels of the drug." Drug dose and frequency are adjusted according to peak and trough levels to enhance efficacy by maintaining therapeutic levels. Peak and trough levels reveal nothing about allergic reactions. Blood cultures are obtained when the client spikes a temperature; they are not related to peak and trough levels of an antibiotic. A sustained decrease in fever is the desired outcome, not reduction just at peak serum levels of the medication.

A healthcare provider prescribes oral loperamide (Maalox) and intravenous ranitidine for a client with burns and crushing injuries caused by an accident. The client asks how these medications work. What is the nurse's best response? "They decrease irritability of the bowel." "They limit acidity in the gastrointestinal tract." "They are very effective in clients with multiple trauma." "They work in the way that antiemetics do."

"They limit acidity in the gastrointestinal tract." Increased acidity caused by the stress occurring with burns and crushing injuries contributes to the formation of Curling ulcer; ranitidine, an H 2 antagonist, decreases the formation of gastric acid, and Maalox, an antacid, neutralizes gastric acid once it is formed. These drugs do not decrease irritability of the bowel; their purpose is to decrease gastrointestinal acidity. The response "They are very effective in clients with multiple trauma" does not answer the client's question. The mechanisms of action for these drugs are not the same as those in antiemetics.

A nurse is caring for a female client who is receiving rifampin for tuberculosis. Which statements indicate that the client understands the teaching about rifampin? "This drug may be hard on my liver so I must avoid alcoholic drinks while taking it." "This drug may reduce the effectiveness of the oral contraceptive I am taking." "I cannot take an antacid within 2 hours before taking my medicine." "My healthcare provider must be called immediately if my eyes and skin become yellow."

"This drug may be hard on my liver so I must avoid alcoholic drinks while taking it." "This drug may reduce the effectiveness of the oral contraceptive I am taking." "My healthcare provider must be called immediately if my eyes and skin become yellow." Alcohol may increase the risk of hepatotoxicity. Rifampin has teratogenic properties and may reduce the effectiveness of oral contraceptives. Yellowing of the eyes and skin are signs of hepatitis and should be reported immediately. An antacid may be taken 1 hour before taking the medication. The capsule may be opened and the powder mixed with applesauce.

The nurse is providing teaching to the parents of a preschool-age client who is prescribed iron supplements for iron-deficiency anemia. Which parental statements indicate the need for further education? "We will mix the iron with milk to enhance absorption." "We will mix the iron with black tea to enhance absorption." "We will mix the iron with orange juice to enhance absorption." "We will avoid giving our child green tea because it can decrease absorption." "We will avoid feeding our child tomatoes because it can decrease absorption."

"We will mix the iron with milk to enhance absorption." "We will mix the iron with black tea to enhance absorption." The parental statements that indicate the need for further education include mixing the iron with milk and black tea to enhance absorption. Milk contains phosphorus and black tea contains tannins, both which decrease the absorption of iron. Orange juice increases the acidity of the stomach, which enhances absorption. Green tea and tomatoes (an oxalate) are avoided as both will decrease the absorption of iron.

A client is started on fluphenazine. What will the nurse emphasize in the teaching about this drug? "Driving is forbidden while you're taking this drug." "You'll have a feeling of increased energy while taking this drug." "You should increase your fluid intake to help prevent constipation." "Your essential hypertension will be controlled indirectly by this drug." "You need to use sunscreen for any outdoor activity, no matter what time of year."

"You should increase your fluid intake to help prevent constipation." "You need to use sunscreen for any outdoor activity, no matter what time of year." Constipation is a common side effect of fluphenazine decanoate; increased dietary fluids and fiber help to limit constipation. Extreme photosensitivity is a common side effect of fluphenazine decanoate. Once the client's medication has been adjusted and the central nervous system response noted, driving may be permitted; drowsiness usually subsides after the first few weeks. Energy usually is decreased. Although this drug can cause hypotension, it does not consistently lower blood pressure.

The primary healthcare provider has prescribed rifampin to a client with tuberculosis. Which instructions by the nurse will be beneficial to the client? "You should report any yellow tinge to your skin." "Your soft contact lenses will be stained permanently." "You should report any reddish orange tinge to your secretions." "You need to drink at least 8 ounces of water with the medication." "You should report any increased tendency to bruising or bleeding."

"You should report any yellow tinge to your skin." "Your soft contact lenses will be stained permanently." "You should report any increased tendency to bruising or bleeding." Rifampin is a first-line drug in the treatment of tuberculosis and clients should report any yellow tinge to the skin because this may be a sign of liver toxicity or failure. Staining of bodily fluids such as tears, urine, and sweat, is commonly associated with rifampin, so warning the client that contact lenses will be stained will be beneficial. The client should be instructed to immediately report any increased tendency to bruising or bleeding because this may indicate liver toxicity or damage. The need to drink at least 8 ounces of water with the medication is beneficial information for a client prescribed pyrazinamide. A reddish orange tinge to secretions is common with rifampin and not harmful, so it need not be reported.

A primary healthcare provider has prescribed isoniazid to a client with tuberculosis. Which instruction by the nurse will be most beneficial to the client? "You should take the drug on an empty stomach." "Your soft contact lenses will be stained permanently." "You must use an additional method of contraception." "You need to drink at least 8 ounces of water with the medication."

"You should take the drug on an empty stomach." Isoniazid is used as first-line drug therapy for tuberculosis. Absorption of the drug from the gastrointestinal tract can be prevented or slowed by the presence of food and antacids, so the client should be instructed to take the drug on an empty stomach. Staining of bodily fluids is commonly associated with rifampin. Rifampin reduces the effectiveness of oral contraceptives, so an additional method of contraception is required for any female client prescribed this drug who also uses birth control pills. The instruction to drink at least 8 ounces of water with the medication would be beneficial fora client who has been prescribed pyrazinamide.

A client is waiting for a kidney transplant. What explanation should the nurse include when teaching the client about the transplant? "Production of urine will be delayed after surgery." "You will require immunosuppressive drugs daily for the rest of your life." "Symptoms of rejection include a decrease in temperature and blood pressure." "You will need to modify your program of work and recreation, including sports."

"You will require immunosuppressive drugs daily for the rest of your life." Immunosuppressive agents are administered to reduce the immune system's tendency to reject the transplanted organ. Urine production occurs almost immediately. Fever, not hypothermia, will occur. Hypotension will not occur. An increased blood pressure may occur because of fluid retention. Recreation and exercise are encouraged, although contact sports should be avoided.

A laboring client who is positive for group B Streptococcus (GBS) is given an initial dose of 2 g of ampicillin at 9 AM. According to established guidelines for intrapartum management of this client, what should the next dose be? 2 g given at 10 AM 1 g given at 11 AM 2 g given at noon 1 g given at 1 PM

1 g given at 1 PM The established guidelines for intrapartum antibiotic prophylaxis for a client infected with GBS is an initial dose of 2 g followed by a 1-g dose every 4 hours.

The nurse observed seizures in a client who is taking lithium for cycles of mania. Which laboratory parameters may lead to this condition? 1 mEq/L (1 mmol/L) serum lithium levels 3 mEq/L (3 mmol/L) serum lithium levels 135 mEq/L (135 mmol/L) serum sodium levels 140 mEq/L (140 mmol/L) serum sodium levels

3 mEq/L (3 mmol/L) serum lithium levels Serum lithium levels exceeding 2.5 mEq/L (2.5 mmol/L) may cause seizures, gastrointestinal discomfort, tremors, confusion, and somnolence. Therefore, a serum lithium level of 3 mEq/L (3 mmol/L) serum may lead to seizures. The desired long-term serum lithium level is 1 mEq/L (1 mmol/L). The normal range of serum sodium levels is from 135 to 145 mEq/L (135-145 mmol/L).

A client is rescued from a house fire and arrives at the emergency department 1 hour after the rescue. The client weighs 132 pounds (60 kilograms) and is burned over 35% of the body. The nurse expects that the amount of lactated Ringer solution that will be prescribed to be infused in the next 8 hours is what? 2100 mL 4200 mL 6300 mL 8400 mL

4200 mL In the first 8 hours 4200 mL should be infused. According to the Parkland (Baxter) formula, one half of the total daily amount of fluid should be administered in the first 8 hours. Because the client weighs 60 kg (132 pounds ÷ 2.2 kg = 60 kg), the calculation is 60 kg × 4 mL/kg × 35% burns = 8400 mL per day; half of this amount should be infused within the first 8 hours. 2100 mL, 6300 mL, and 8400 mL are incorrect calculations.

A client reports severe itching, especially at night. On assessment, the nurse finds the presence of burrows in the flexor surfaces of the wrists and in the anterior axillary folds. Which treatment is most appropriate for this client? Antipruriticlotion Υ-Benzene hexachloride 5% Permethrin topical lotion 0.9%Spinosad topical suspension

5% Permethrin topical lotion Severe itching, especially at night, and the presence of burrows in the flexor surface of the wrists and in the anterior axillary folds indicate scabies, which is caused by the Sarcoptes scabieimite. Treatment involves the use of 5% permethrin topical lotion for one overnight application and a second application 1 week later. This treatment can yield 95% eradication of the condition. Antipruriticlotion is required in cases of intense burning, local pain, swelling, and itching. Υ-Benzene hexachloride is a common treatment for pediculosis. In cases of pediculosis involving the scalp, 0.9% spinosad topical suspension may be helpful.

Which intravenous fluid should the nurse classify as hypertonic? Ringer solution 5% dextrose in water Lactated Ringer solution 5% dextrose in normal saline

5% dextrose in normal saline An isotonic solution has the same osmolarity as body fluids. A hypertonic solution has a higher osmolarity than body fluids; it pulls fluid from cells, causing them to shrink and the extracellular space to expand. This hypertonic solution provides 586 mOsm/kg. This isotonic solution provides 309 mOsm/kg. The other isotonic solutions provide 278 mOsm/kg.

A client with tuberculosis asks the nurse how long chemotherapy will be continued. What is the nurse's most accurate reply? 1 to 2 weeks 4 to 5 months 6 to 12 months 3 years or longer

6 to 12 months The tubercle bacillus is a drug-resistant organism and takes a long time to be eradicated; usually a combination of three medications is used for a minimum of 6 months, and at least 6 months beyond culture conversion. One to 2 weeks or 4 to 5 months are too short a time for eradication of this organism. Usually, the organism can be eradicated in a shorter period of time than 3 years, unless a resistant strain of the bacillus has developed.

When is the first dose of Rotarix vaccine administered in infants? Birth to 6 weeks 6-12 weeks 12-18 weeks 18-24 weeks

6-12 weeks The Rotarix vaccine prevents rotavirus gastroenteritis and diarrhea-related problems. The vaccination requires two doses. The first dose of Rotarix should be given between 6 and 12 weeks followed by the second dose, which is given four or more weeks after the first dose. Administering the vaccine before the age of 6 weeks is too early, and administering it at 12-18 weeks or 18-24 weeks is too late.

How long after the last dose will the nurse schedule to have a client's blood drawn to evaluate the serum lithium level? 2 to 4 hours 4 to 6 hours 6 to 8 hours 8 to 12 hours

8 to 12 hours Absorption and excretion of lithium occur 8 to 12 hours after the last dose. Absorption and excretion rates vary; concentrations may be falsely higher at 2 to 4 hours, at 4 to 6 hours, or at 6 to 8 hours, affecting the reliability of readings.

An obese client must self-administer insulin at home. The nurse will teach the client to inject insulin at which angle? 30-degree angle 60-degree angle 45-degree angle 90-degree angle

90-degree angle Injection should be made at a 90-degree angle for most patients, including those of normal weight. Injecting at a 30-degree angle or a 60-degree angle is not appropriate for the obese, normal weight, the child, or the thin client. If injecting into a child or a thin client, the injection should be made at a 45-degree angle.

A nurse gave a client the prescribed sodium polystyrene sulfonate. What assessment finding indicates that the drug has been effective? The presence of diarrhea A narrowing of the QRS complex An increase in serum calcium level A decrease in serum potassium level

A decrease in serum potassium level Sodium polystyrene sulfonate is given to treat hyperkalemia. Therefore the effectiveness of the medication is determined by a decreasing serum potassium level. Sodium polystyrene sulfonate binds with the potassium in the gastrointestinal system and often causes diarrhea. Sodium polystyrene sulfonate has no effect on serum calcium levels. A wide QRS complex is a late finding in hyperkalemia. Sodium polystyrene sulfonate takes time to work and therefore would not be the drug of choice for hyperkalemia evidenced by a widening QRS complex.

Which physiologic characteristics of newborns affect drug dosage considerations? A newborn's less regulated body temperature Immature liver and kidneys Thick and less permeable skin Lungs with weaker mucous barriers Bacteria-killing acid in the stomach

A newborn's less regulated body temperature Immature liver and kidneys Lungs with weaker mucous barriers The body temperature of newborns is less regulated and dehydration occurs easily. This characteristic affects the drug dose consideration in newborns. Metabolism and excretion are impaired in pediatric clients due to an immature liver and kidneys. The lungs in pediatric clients have weak mucous barriers; this characteristic also affects the drug dosage considerations in newborns. A newborn's skin is thin and more permeable. The newborn has no acid in the stomach to kill the bacteria; therefore, drug absorption from the gastrointestinal tract is affected, thus impacting drug dosage considerations.

A nurse discusses the implications of diet and fluid intake with a client who is receiving lithium therapy. What will the nurse teach the client and family about nutrition? A regular diet should be maintained. Daily fluid intake should be limited to 1 L. Daily salt intake should be limited to 2000 g. A weight-reducing diet should be implemented.

A regular diet should be maintained. A regular diet maintains sodium balance; lithium decreases sodium resorption by the renal tubules. Initially, weight-reducing diets deplete body fluids; this can result in lithium retention and toxicity. Lithium is excreted in urine; decreased fluid intake and a consequent decreased urine output can result in lithium retention and toxicity. Limitation of daily salt intake to 2000 g is unsafe, because lithium decreases sodium resorption, resulting in sodium excretion; sodium intake should not be decreased.

A healthcare provider prescribes aspirin therapy for a client with arthritis, and the nurse provides teaching about the undesirable side effects of this medication. What responses should the client identify as reasons to notify the healthcare provider? SATA A. Nausea B. Constipation C. Easy bruising D. Decreased pulse E. Ringing in the ears

A, C, E

A nurse is caring for a client with severe preeclampsia who is receiving magnesium sulfate. What adverse effects indicate that the serum magnesium level may be excessive? Absence of the knee-jerk reflex Urine output of 100 mL/hr Blood pressure of 140/90 mm Hg Apical pulse of 80 beats/min Respiratory rate of 11 breaths/min

Absence of the knee-jerk reflex Respiratory rate of 11 breaths/min An absence of the knee-jerk reflex is a manifestation of hyporeflexia; it is a possible indication of magnesium sulfate toxicity. A respiratory rate of 11 breaths/min is cause for concern; any rate slower than 12 breaths/min is a sign of magnesium sulfate toxicity. A urinary output of 100 mL/hr is adequate; output of less than 30 mL/hr indicates inadequate excretion of magnesium sulfate and the potential for toxicity. The maternal blood pressure is not directly related to magnesium sulfate administration or toxicity; however, decreased blood pressure indicates that the treatment has been effective. A pulse rate of 80 beats/min is an expected pulse rate, not an indicator of toxicity.

A client at 32 weeks' gestation is admitted to the birthing unit because she is having regular labor contractions. A prescription for 6 mg of intramuscular dexamethasone, twice a day for 2 days, is issued. The client asks why she needs this medicine. The nurse explains that this corticosteroid is prescribed because it does what? Promotes sleep May stop contractions Relaxes uterine muscles Accelerates fetal lung maturity

Accelerates fetal lung maturity Dexamethasone is a glucocorticoid that aids fetal production of surfactant, which is needed for postnatal lung expansion; it is not until 35 weeks' gestation that there is enough surfactant to confirm fetal lung maturity by way of amniocentesis. Glucocorticoids are not sedatives. Tocolytics, not glucocorticoids, may stop contractions. Tocolytics, not glucocorticoids, relax the uterine muscles.

A client who is on monoclonal antibody medication reports rigors, headache, myalgia, and gastrointestinal disturbances. The medical history shows that the client is on muromonab-CD3. Which medications could be beneficial for the reported symptoms? Sirolimus Cyclosporine Acetaminophen Diphenhydramine Methylprednisolone

Acetaminophen Diphenhydramine Methylprednisolone Acetaminophen, diphenhydramine, and methylprednisolone are administered to reduce the adverse effects associated with muromonab-CD3 use. When monoclonal antibodies are administered, a flulike syndrome occurs during the first few days of treatment because of cytokine release. Rigors, headache, myalgia, and gastrointestinal disturbances are the adverse effects of monoclonal antibodies. Sirolimus and cyclosporine are used in combination to reduce the graft loss that occurs in transplant recipients.

Which drug does a nurse anticipate may be prescribed to produce diuresis and inhibit formation of aqueous humor for a client with glaucoma? Chlorothiazide Acetazolamide Methazolamide Aclidinium bromide

Acetazolamide Acetazolamide is a carbonic anhydrase inhibitor that decreases inflow of aqueous humor and controls intraocular pressure in acute angle-closure glaucoma attack. Chlorothiazide has no effect on the eye. Methazolamide lowers ocular pressure but does not decrease the inflow of aqueous humor. Aclidinium bromide is a bronchodilator.

A nurse is caring for a client with diabetes who is scheduled for a radiographic study requiring contrast. Which should the nurse expect the health care provider to prescribe? Acetylcysteine before the test Renal-friendly contrast medium for the test Forced diuresis with mannitol after the test Hydration with dextrose and water throughout the test

Acetylcysteine before the test Acetylcysteine is an antioxidant that scavenges oxygen free radicals, which are released when contrast medium causes cell death to renal tubular tissue; it also induces slight vasodilation. Contrast that is renal friendly does not exist. Mannitol is not necessary. Saline alone provides better protection of the kidneys from contrast-induced nephropathy. Hydration with saline, not dextrose and water, affords some protection from kidney damage caused by contrast media; dextrose will increase the glucose level in an individual with diabetes and thus is contraindicated.

Which antimicrobial medication acts on susceptible pathogens by inhibiting nucleic acid synthesis? Penicillin Actinomycin Erythromycin Cephalosporin

Actinomycin Actinomycin is an antimicrobial medication that acts on susceptible pathogens by inhibiting nucleic acid synthesis. Penicillin acts on susceptible pathogens by inhibiting cell wall synthesis. Erythromycin acts on susceptible pathogens by inhibiting biosynthesis and reproduction. Cephalosporin acts on susceptible pathogens by inhibiting cell wall synthesis.

Although a client in labor is prepared and plans to participate in the labor and birth process, she states that she is in severe discomfort. The nurse administers the prescribed butorphanol. Which phase of labor is the safest time for the nurse to administer this medication? Early phase Active phase Transition phase Expulsion phase

Active phase Respiratory depression of the newborn will not occur if the medication is given during the active phase; it should not be given when birth is expected to occur within 2 hours. The level of pain during the early phase can usually be managed with other strategies such as breathing techniques or diversion; giving an opioid early in labor may slow the progress of labor. An opioid should be avoided in the 2 hours preceding birth; giving it to a client in the transition phase can cause respiratory depression in the newborn. Giving the medication when birth is imminent is contraindicated because it may cause respiratory depression in the newborn; the mother's level of consciousness will be altered as well, making it difficult for her to cooperate with requests for her to push.

A nurse is admitting a 2-year-old toddler with a tentative diagnosis of cystic fibrosis to the pediatric unit. Pilocarpine is used as part of the diagnostic process. The nurse knows that the pilocarpine will stimulate which process? Secretion of mucus Activity of sweat glands Excretion of pancreatic enzymes Release of bile from the gallbladder

Activity of sweat glands Pilocarpine is a cholinergic that is applied to the skin to stimulate sweat production; the sweat is then tested to confirm the diagnosis of cystic fibrosis. Pilocarpine does not stimulate the secretion of mucus, the excretion of pancreatic enzymes, or the release of bile from the gallbladder.

A client with hypertension has labor pains before the 35th week of gestation. Which pharmacologic intervention does the nurse anticipate to be beneficial for this client? Preparing the client for an abortion Administer terbutaline Administer magnesium sulfate Administer sedatives and maintain hydration

Administer magnesium sulfate Magnesium sulfate is the drug of choice for maintaining pregnancy in preterm labor in hypertensive clients. An abortion is advisable if the preterm labor starts before the 20th week of gestation because the fetus may be nonviable. Terbutaline is used to maintain pregnancy in preterm labor by relaxing the uterine smooth muscle. The use of sedatives and body hydration measure are nonpharmacologic interventions to maintain pregnancy in preterm labor.

After thoracic surgery for removal of a cancerous lesion in the lung, the client is drowsy, complains of pain when awakened, and then falls asleep. The client has a prescription for morphine sulfate via IV every 3 hours as needed for pain. The client's preoperative blood pressure was 128/76 mm Hg. Postoperative assessments reveal that the client's blood pressure ranges between 90/60 and 100/70 mm Hg. What is the nurse's best initial action? Administer morphine as prescribed. Obtain a prescription for a vasoconstrictor. Give half the prescribed amount of morphine. Withhold morphine until the blood pressure stabilizes.

Administer morphine as prescribed. Morphine may decrease the blood pressure further; clients who are drowsy may sleep without medication immediately after surgery. The healthcare provider should be notified to reevaluate pain management. Administering the morphine is contraindicated because morphine is an opioid analgesic that can lower further the blood pressure. A vasoconstrictor will not relieve the pain. Administration of a medication dosage other than that prescribed is not an independent nursing function.

A client is taking phenytoin to treat clonic-tonic seizures. The client's phenytoin level is 16 mg/L. Which action should the nurse take? Hold the medication and notify the healthcare provider. Administer the next dose of the medication as prescribed. Hold the next dose and then resume administration as prescribed. Call the healthcare provider to obtain a prescription with an increased dose.

Administer the next dose of the medication as prescribed. Administering the next dose of the medication as prescribed is within the therapeutic range of 10 to 20 mg/L (40 to 80 mcmol/L); the nurse should administer the drug as prescribed. The phenytoin level is within the therapeutic range of 10 to 20 mg/L (40-80 mcmol/L); there is no need to hold the dose and notify the healthcare provider. Holding the next dose and then resuming administration as prescribed is unsafe and will reduce the therapeutic blood level of the drug. Calling the healthcare provider to obtain a prescription with an increased dose is unnecessary; the blood level is within the therapeutic range.

A platelet transfusion is to be administered to a child with acute lymphocytic leukemia. What will the nurse do first? Administer the platelets rapidly through the intravenous (IV) line Set the IV pump to run for 8 hours Flush the IV line with a dextrose solution Check the vital signs every 2 hours during the transfusion

Administer the platelets rapidly through the intravenous (IV) line Platelets are fragile and should be administered as quickly as possible, within 1 hour, or as fast as the child can tolerate the infusion. There are minimal numbers of red blood cells (RBCs) and white blood cells contained within the infusion, which reduces the risk of a severe reaction. Platelets must be infused within 1 hour. They may be infused as rapidly as the child's cardiovascular status will tolerate. A dextrose solution is not appropriate for flushing a blood derivative line because it may cause hemolysis of RBCs. Two hours is too long an interval between checks of the child's vital signs. Vital signs should be obtained before the infusion, 15 minutes after initiation of the infusion, and at the end of the infusion.

At 12 weeks' gestation a client who is Rh negative expels the total products of conception. What is the nursing action after it has been determined that she has not been previously sensitized to Rh-positive red blood cells? Administering Rho(D) immune globulin within 72 hours Making certain that Rho(D) immune globulin is administered at the first clinic visit Withholding the Rho(D) immune globulin because the gestation lasted only 12 weeks Withholding the Rho(D) immune globulin because it is not indicated after fetal death

Administering Rho(D) immune globulin within 72 hours Rho(D) immune globulin must be given within 72 hours of delivery if the client has not been sensitized previously, regardless of the duration of the gestation. It would not be effective at the first clinic visit, because antibodies have been produced already. Rho(D) immune globulin is always indicated at the termination of a pregnancy, even with a short-term pregnancy or one involving fetal demise.

A client who is taking haloperidol has developed tardive dyskinesia. Which therapy is beneficial for the client? Administering benzodiazepines Providing anticholinergics therapy Administering nonsteroidal antiinflammatory drugs Switching to other first-generation antipsychotic drugs

Administering benzodiazepines The long-term usage of first-generation antipsychotics such as haloperidol increases the risk of tardive dyskinesia. The client should be treated with benzodiazepines. Any anticholinergics drugs should be discontinued in the client. Nonsteroidal antiinflammatory drugs may not be beneficial for the client. The client should not switch to another first-generation antipsychotic because the risk of tardive dyskinesia still remains.

A 9-month-old infant has been prescribed iron supplements. During follow-up visits, the nurse observes that the infant's iron-deficiency anemia has not improved despite treatment. Which action taken by the parents does the nurse recognize as the reason for the lack of improvement? Administering iron supplements through a straw Administering iron supplements along with orange juice Administering iron supplements with whole cow's milk Administering iron supplements at the back of the mouth

Administering iron supplements with whole cow's milk Whole cow's milk binds with free iron and reduces drug absorption. Therefore, the infant has developed drug insufficiency for maximum therapeutic action. Administering iron supplements through a straw does not reduce drug absorption; it prevents the iron from staining the infant's teeth. Orange juice increases the absorption of iron supplements. Administering iron supplements at the back of the mouth does not reduce drug absorption; it prevents the iron from staining the infant's teeth.

A client with a recent history of sinusitis develops meningitis and demonstrates a positive Brudzinski sign. What is the priority nursing care? Monitoring intracranial pressure Adding pads to the side of the bed Administering prescribed antibiotics Hydrating the client with hypotonic saline

Administering prescribed antibiotics The Brudzinski sign (when the neck is flexed while in the supine position, flexion of the hips occurs) indicates bacterial meningitis, a complication of sinusitis; the client's greatest need is a regimen of antibiotics to which the causative agent is sensitive. Bacterial meningitis causes increased intracranial pressure and it is important for the nurse to monitor for manifestations of increased intracranial pressure; however, in this circumstance, it is not the priority because monitoring alone does not affect outcomes . Because of the risk for seizures in bacterial meningitis, padded side rails are an important nursing intervention; however, this intervention does not have priority over instituting the appropriate antibiotic therapy to eradicate the cause of the meningitis. The data do not indicate a need for a hypotonic solution for hydrating the client.

A client is admitted to the hospital for medical management of acute pancreatitis. Which nursing action is most likely to reduce the pancreatic and gastric secretions of a client with pancreatitis? Encouraging clear liquids Obtaining a prescription for morphine Assisting the client into a semi-Fowler position Administering prescribed anticholinergic medication

Administering prescribed anticholinergic medication Anticholinergic drugs block the neural impulses that stimulate pancreatic and gastric secretions; they inhibit the action of acetylcholine at postganglionic cholinergic nerve fibers. Oral fluids stimulate pancreatic secretion and are contraindicated. Morphine sulfate is an analgesic and therefore does not decrease gastric secretions; in the past morphine sulfate was contraindicated for pain control with pancreatitis because it can precipitate spasms of the smooth musculature of the pancreatic ducts and the sphincter of Oddi. However, recent research indicates that it is the drug of choice over meperidine hydrochloride because the metabolites of meperidine hydrochloride can cause central nervous system irritation and seizures. The semi-Fowler position decreases pressure against the diaphragm; it will not decrease pancreatic secretions.

What should the nurse include in the discharge instructions for a client who will be receiving total parenteral nutrition (TPN) at home? Changing the TPN access device daily Contacting and scheduling professionals to administer the TPN Listing the schedule of the days the client is to receive the TPN Administering the TPN while working around the client's normal activities

Administering the TPN while working around the client's normal activities The less disruptive the procedure, the greater the acceptance by the client. Most often, total parenteral nutrition is set up to run daily during sleeping hours. Depending on the type of circulatory access used, it may not need to be changed for weeks. The client or a significant other can be taught the principles of administration.

Which action by the nurse while administering human growth hormone ensures effective therapy? Administration at nighttime Administration via oral route Administration along with meals Administration by metered spray

Administration at nighttime Human growth hormone therapy shows best results when the hormone is administered at nighttime because the body naturally produces growth hormone at night. Therefore the normal body rhythm is being mimicked to ensure effective therapy. Subcutaneous injections of growth hormone yield effective results. Hyperpituitarism is treated by the administration of bromocriptine, which should be taken along with food to reduce side effects. Desmopressin acetate is administered either orally or intranasally with a metered spray to treat diabetes insipidus.

Methylphenidate is prescribed for a 6-year-old boy with the diagnosis of attention deficit-hyperactivity disorder (ADHD). The nurse teaches the father about safe medication administration and concludes that the instructions have been understood when the father says that he should administer it at which time? At bedtime After breakfast When the child gets hungry When the child's behavior is out of control

After breakfast Methylphenidate (Ritalin SR) may cause nausea, anorexia, and dry mouth, which interfere with appetite and adequate food intake; therefore it should be administered after the child has eaten breakfast. Methylphenidate is a cerebral stimulant that can interfere with sleep; it should not be administered within 6 hours of bedtime. It should be taken exactly as prescribed, not on an as-needed basis.

A client is prescribed sertraline, an antidepressant. What does the nurse include when preparing a teaching plan about the side effects of this drug? Seizures Agitation Tachycardia Agranulocytosis

Agitation Sertraline, a selective serotonin reuptake inhibitor (SSRI), inhibits neuronal uptake of serotonin in the central nervous system, thus potentiating the activity of serotonin. Central nervous system side effects of this drug include agitation, anxiety, confusion, dizziness, drowsiness, and headache. Seizures are a side effect of clozapine, an antipsychotic, not sertraline, which is an antidepressant. Tachycardia is a side effect of tricyclic antidepressants, not sertraline, which is an SSRI antidepressant. A decrease in the production of granulocytes (agranulocytosis) causing a pronounced neutropenia is a side effect of clozapine, not sertraline.

A client with schizophrenia who has been taking clozapine is to be started on 10 mg of olanzapine instead. The nurse explains to the client, in terms that can be understood, that olanzapine is being substituted for clozapine because it does not produce which side effect? Hypotension Gastric upset Agranulocytosis Metabolic syndrome

Agranulocytosis Although neutropenia may occur, agranulocytosis does not occur as a side effect of olanzapine. Cardiovascular responses, such as hypotension, are side effects of both medications. Dyspepsia, nausea, vomiting, anorexia, and other gastrointestinal disturbances occur with both medications. Metabolic syndrome may occur with olanzapine. Metabolic syndrome is a cluster of conditions including weight gain, increased cholesterol and triglyceride levels, hyperglycemia, and diabetic ketoacidosis.

A client has been taking the prescribed dose of clozapine. The nurse will assess the client for which life-threatening side effect of this drug? Polycythemia Agranulocytosis Hypertensive crisis Pseudoparkinsonism

Agranulocytosis Agranulocytosis occurs in 1% to 2% of clients receiving clozapine and is potentially fatal; weekly blood counts are necessary. Polycythemia is not a side effect of clozapine. Clozapine may cause hypotension; hypertensive crisis is a side effect of monoamine oxidase inhibitors. Pseudoparkinsonism may occur, but it can be managed with anticholinergic medications.

A client who has just started on a regimen of haloperidol is observed pacing and shifting weight from one foot to the other. What side effect does the nurse document in the client's chart? Akathisia Parkinsonism Tardive dyskinesia Acute dystonic reaction

Akathisia Restlessness or the desire to keep moving (akathisia) can occur within 6 hours of the first dose of haloperidol; this side effect is associated with most neuroleptics. Parkinsonian side effects include masklike facies, tremors, and shuffling gait. Tardive dyskinesia is a severe, largely irreversible, extrapyramidal side effect occurring after prolonged treatment with phenothiazines. Acute dystonic reaction is characterized by severe, bizarre muscle contractions.

A nurse is caring for an older adult who is taking acetaminophen for the relief of chronic pain. Which substance is most important for the nurse to determine if the client is taking because it intensifies the most serious adverse effect of acetaminophen? Alcohol Caffeine Saw palmetto St. John's wort

Alcohol Too much ingestion of alcohol can cause scarring and fibrosis of the liver. Eighty-five percent to 95% of acetaminophen is metabolized by the liver. Acetaminophen and alcohol are both hepatotoxic substances. Metabolites of acetaminophen, along with alcohol, can cause irreversible liver damage. Caffeine stimulates the cardiovascular system, not the liver. In addition, caffeine does not interact with acetaminophen. Saw palmetto is not associated with increased liver damage when taking acetaminophen. It often is taken for benign prostatic hypertrophy because of its antiinflammatory and antiproliferative properties in prostate tissue. St. John's wort is classified as an antidepressant and is not associated with increased liver damage when taking acetaminophen. However, it does decrease the effectiveness of acetaminophen.

A client receiving cisplatin therapy has developed tumor lysis syndrome (TLS). Which medication should the nurse administer to treat the TLS? Mesna Flavoxate Allopurinol Aprepitant

Allopurinol Allopurinol should be administered to this client to promote purine excretion. Cisplatin is a nephrotoxic agent that is used in clients with cancer. TLS is the precipitation of metabolites (purine and potassium) of cell breakdown. Mesna and flavoxate are used to treat hemorrhagic cystitis in clients on chemotherapy; mesna is a protectant while flavoxate manages symptoms. Aprepitant is used to prevent nausea and vomiting in a client on the day of chemotherapy.

A nurse is interviewing an adolescent who is to start a chemotherapeutic drug regimen that includes vincristine. Which side effect is it most important for the nurse to prepare the adolescent to expect? Alopecia Constipation Loss of appetite Peripheral neuropathy

Alopecia A side effect of vincristine is alopecia. To adolescents, who are very concerned with identity, hair loss represents a tremendous threat to self-image. Constipation, although very serious, is not as important to the adolescent as a side effect that affects appearance. Although anorexia will be a concern while the adolescent is undergoing chemotherapy, it is not as important before the start of the regimen. Although neurologic side effects are serious, they are not as important to the adolescent before the start of chemotherapy.

A client who has an adenocarcinoma of the descending colon with a partial obstruction is receiving doxorubicin intravenously (IV) to reduce the tumor mass. The nurse monitors the client for signs and symptoms of doxorubicin toxicity. What clinical finding indicates that toxicity has occurred? A. Minor skin rash B. Blue tinge to the urine C. Alteration in cardiac rhythm D. Increased feeling of nervousness

Alteration in cardiac rhythm Doxorubicin is cardiotoxic and causes dysrhythmias. Doxorubicin toxicity causes severe, not minor, dermatitis. Blue-tinged urine is a side effect of doxorubicin, not a toxic effect. Feelings of nervousness are a side effect of doxorubicin, not a toxic effect.

Prednisone is prescribed for a client with an exacerbation of colitis. What does the nurse teach the client before administering the first dose? Multiple choice question The client will be protected from getting an infection. Symptoms associated with the colitis will decrease slowly over time. Although the medication causes anorexia, weight loss may not occur. Although the medication decreases intestinal inflammation, it will not cure the colitis.

Although the medication decreases intestinal inflammation, it will not cure the colitis. Prednisone inhibits phagocytosis and suppresses other clinical phenomena of inflammation; this is a symptomatic treatment that is not curative. The response usually is rapid. The drug suppresses the immune response and increases the potential for infection. Appetite is increased; weight gain may result from this or from fluid retention.

A primary healthcare provider diagnoses chronic low-grade depression in a client. Which drug may be prescribed for this client? Alprazolam Lithium salts Amitriptyline Clomipramine

Amitriptyline Dysthymia (chronic low-grade depression) can be treated by the administration of antidepressant drugs such as amitriptyline. Alprazolam is the drug of choice for treating anxiety disorders. Lithium salt is prescribed to treat bipolar disorders. Clomipramine is a tricyclic antidepressant drug prescribed for treating obsessive-compulsive disorder.

A pregnant woman presents with a body temperature of 103 °F, cough, headache, muscle aches, chest pain, severe joint pain and night sweats and is diagnosed with coccidioidomycosis. Which medication will the nurse most likely observe prescribed on the medication administration record? Doxycycline Ciprofloxacin Pyrazinamide Amphotericin B

Amphotericin B An elevated body temperature of 103 °F, cough, headache, muscle aches, chest pain, severe joint pain and night sweats are symptoms of coccidioidomycosis, a fungal infection. Pregnant women can safely take amphotericin B because the drug will not affect the fetus. Doxycycline is a tetracycline that may lead to discoloration of the teeth in the newborn. Ciprofloxacin is a broad-spectrum antibiotic used to treat various bacterial infections and is ineffective with a fungal infection. Pyrazinamide is one of the first-line treatments for tuberculosis.

A nurse explains to a client with trigeminal neuralgia that a treatment is effective on a temporary (6 to 18 months) basis. The nurse is referring to which treatment? Weekly intravenous injections of cobra venom A lidocaine injection of the ventral root of the eleventh spinal nerve Microvascular decompression of the blood vessels at the nerve root An alcohol injection of the peripheral branch of the fifth cranial nerve

An alcohol injection of the peripheral branch of the fifth cranial nerve A nerve block of the trigeminal (fifth cranial) nerve with alcohol is a conservative approach that lasts 6 to 18 months. Weekly intravenous injections of cobra venom have been tried but provide little, if any, relief. Lidocaine is not used; cranial nerve XI is the spinal accessory nerve that innervates the sternocleidomastoid and trapezius muscles. Microvascular decompression of the blood vessels at the nerve root is not a conservative approach; this is the most commonly used surgical procedure for trigeminal neuralgia. Neuralgia may recur in 30% of clients within 6 years.

A client with the diagnosis of primary hypertension is started on a regimen of hydrochlorothiazide. The nurse is providing instructions regarding this medication. What information should the nurse include? A common side effect is decreased sexual libido. One dose should be omitted if dizziness occurs when standing up. The client should adjust the dosage daily based on his blood pressure. An antihypertensive medication will likely be required for the remainder of life.

An antihypertensive medication will likely be required for the remainder of life. If medication is necessary to control primary hypertension, usually it is a lifetime requirement. The client should not adjust the dosage without the healthcare provider's direction. Impotence may occur with some antihypertensive medications but not with hydrochlorothiazide. The drug should not be stopped; orthostatic hypotension can be controlled by a slow change of body position.

A client who is obtunded has a blood pressure of 80/35 mm Hg after a blood transfusion. In an effort to support renal perfusion, the nurse administers dopamine at 2 mcg/kg/min as prescribed. What is the most relevant outcome indicating effectiveness of the medication for this client? A decrease in blood pressure An increase in urinary output A decrease in core temperature An increase in level of consciousness

An increase in urinary output As renal perfusion increases, urinary output also should increase; doses greater than 10 mcg/kg/min can cause renal vasoconstriction and decreased urinary output. A change in blood pressure is not a direct predictor of the effectiveness of dopamine given at a level of 2 mcg/kg/min; at 10 mcg/kg/min a client will experience an increased cardiac output and an increased blood pressure. Body temperature does not indicate improved renal perfusion. In this situation, improvement of renal perfusion is not directly related to the client's level of consciousness.

A client has an anaphylactic reaction after receiving intravenous penicillin. What does the nurse conclude is the cause of this reaction? An acquired atopic sensitization occurred. There was passive immunity to the penicillin allergen. Antibodies to penicillin developed after a previous exposure. Potent antibodies were produced when the infusion was instituted.

Antibodies to penicillin developed after a previous exposure. Hypersensitivity results from the production of antibodies in response to exposure to certain foreign substances (allergens). Earlier exposure is necessary for the development of these antibodies. This is not a sensitivity reaction to penicillin; hay fever and asthma are atopic conditions. It is an active, not passive, immune response. Antibodies developed when there was a previous, not current, exposure to penicillin.

A client with newly diagnosed rapid-cycling bipolar disorder will be meeting with the nurse for an educational session about the pharmacological approach that is planned. Which classification of medication does the nurse expect to discuss? Antianxiety medication Antiparkinson medication Antidepressant medication Anticonvulsant medication

Anticonvulsant medication Anticonvulsant medications are therapeutic for clients with rapid-cycling bipolar disorder. Antianxiety medications are not primarily used for rapid-cycling bipolar disorder. Antianxiety medications may be helpful for clients with treatment-resistant mania. Antiparkinson medications are not used for rapid-cycling bipolar disorder. An antidepressant medication is not used unless the client also is taking an antipsychotic medication.

Which assessment should the nurse obtain before administering digoxin to a client? Apical heart rate Radial pulse on the left side Radial pulse in both right and left arms Difference between apical and radial pulses

Apical heart rate Because digoxin slows the heart rate, the apical pulse should be counted for 1 minute before administration. If the apical rate is below a preset parameter (usually 60 beats/min), digoxin should be withheld because its administration may further decrease the heart rate. Some protocols permit waiting for one hour and retaking the apical rate; the result determines if it is administered or if the healthcare provider is notified. Obtaining the radial pulse on the left side is not as accurate as an apical pulse; the client also may have an atrial dysrhythmia, which cannot be detected through a radial rate alone. Obtaining the radial pulse in both right and left arms is not as accurate as an apical pulse; the client also may have an atrial dysrhythmia, which cannot be detected through a radial rate alone. Obtaining the difference between apical and radial pulses is a pulse deficit, not a pulse rate.

Which interventions should the nurse anticipate will be prescribed initially for a client who had a hemorrhoidectomy? . Giving an enema Applying moist heat Administering stool softeners Encouraging showers as needed Providing occlusive dressings to the area

Applying moist heat Administering stool softeners Moist heat dilates the blood vessels, thereby increasing circulation to the area; this is soothing and promotes healing. Stool softeners are prescribed to avoid straining on defecation and constipation. Enemas may be prescribed several days after surgery if the client has not had a bowel movement. Baths, especially sitz baths, are advised to promote healing and cleaning of the area. Occlusive dressings are not used. Light applications of witch hazel may be used to promote drainage and healing.

50. To prevent unnecessary hypoxia during suctioning of a tracheostomy, the nurse must A) Apply suction for no more than 10 seconds B) Maintain sterile technique C) Lubricate 3 to 4 inches of the catheter tip D) Withdraw catheter in a circular motion

Applying suction for more than 10 seconds

A client will be discharged with a peripherally inserted central venous catheter (PICC) for administration of peripheral parenteral nutrition (PPN). What would be appropriate for the nurse to include in the client's discharge teaching? Learning how to change the percutaneous catheter Determining which days to self-administer the PPN solution Arranging for professional help to monitor the alternative nutrition Scheduling administration of the PPN solution around mealtimes

Arranging for professional help to monitor the alternative nutrition Professional assistance will ensure correct administration, which may limit complications such as intravascular overload and sepsis; eventually, the client may self-administer the PPN with supervision. Learning how to change the percutaneous catheter usually is done by an appropriate health care provider. PPN usually is administered every day. The PPN solution usually is administered as an intermittent infusion while the client is sleeping at night, not at mealtimes; this allows for independent movement during the day.

A nurse prepares to administer intravenous (IV) albumin to a client with ascites. What effect does the nurse anticipate? Ascites and blood ammonia levels will decrease. Decreased capillary perfusion and blood pressure. Venous stasis and blood urea nitrogen level will increase. As extravascular fluid decreases, the hematocrit will decrease.

As extravascular fluid decreases, the hematocrit will decrease. Serum albumin is administered to maintain blood volume and normal oncotic (osmotic) pressure; it does this by pulling fluid from the interstitial spaces into the intravascular compartment. Serum albumin does affect blood ammonia levels; fluid accumulated in the abdominal cavity is removed via a paracentesis. The administration of albumin results in a shift of fluid from the interstitial to the intravascular compartment, which probably will increase the blood pressure. Albumin administration does not affect venous stasis or the blood urea nitrogen level.

Which step should the nurse refrain from while giving an injection to a preschooler? Awakening the child, if asleep Asking the parent to restrain the child Distracting the child with conversation Applying lidocaine ointment over the injection site

Asking the parent to restrain the child The nurse should not ask the parent to restrain the child. The parent should act as a comforter. Before giving an injection, the nurse should awaken the child. Distracting the child with conversation, bubbles, or a toy reduces pain perception. The nurse should apply lidocaine ointment over the injection site before giving the injection to reduce pain.

Which medications are associated commonly with upper gastrointestinal (GI) bleeding? . Aspirin Ibuprofen Ciprofloxacin Acetaminophen Methylprednisolone

Aspirin Ibuprofen Methylprednisolone Nonsteroidal antiinflammatory drugs (NSAIDs), including acetylsalicylic acid and ibuprofen, and corticosteroids such as methylprednisolone, are known causes of drug-induced gastrointestinal (GI) bleeding by causing irritation and erosion of the gastric mucosal barrier. Acetaminophen is a safe alternative to NSAIDs to reduce the risk of GI bleeding. Ciprofloxacin, an antibiotic, has not been associated with GI bleeding.

Which medications are associated commonly with upper gastrointestinal (GI) bleeding? . Aspirin Ibuprofen Ciprofloxacin Acetaminophen Methylprednisolone

Aspirin Ibuprofen Methylprednisolone Nonsteroidal antiinflammatory drugs (NSAIDs), including acetylsalicylic acid and ibuprofen, and corticosteroids such as methylprednisolone, are known causes of drug-induced gastrointestinal (GI) bleeding by causing irritation and erosion of the gastric mucosal barrier. Acetaminophen is a safe alternative to NSAIDs to reduce the risk of GI bleeding. Ciprofloxacin, an antibiotic, has not been associated with GI bleeding.

A nurse notes gentamycin in the prescription of an older adult with osteomyelitis. Which nursing interventions should be conducted before starting therapy? Assessing renal function Assessing hydration status Checking the erythrocyte count Checking the blood platelet count Assessing serum thyroxin levels

Assessing renal function Assessing hydration status Because gentamycin can increase the risk of nephrotoxicity, the nurse should assess a client's renal function before starting therapy. Dehydration can further increase the risk of nephrotoxicity; therefore the client's hydration status should also be checked before starting therapy. Gentamycin generally does not impact erythrocyte and blood platelet counts nor does it affect serum thyroxin levels.

Corrective surgery for hypertrophic pyloric stenosis is completed, and the infant is returned to the pediatric unit with an intravenous (IV) infusion in progress. What is the priority nursing action? Applying adequate restraints Administering a mild sedative Removing the nasogastric tube Assessing the IV site for infiltration

Assessing the IV site for infiltration Assessment of the IV site is a priority. The infant will need IV fluids until oral feedings are possible. Restraints are not needed. Administering a sedative is not necessary and should not be done until a full assessment is completed. If the infant has a nasogastric tube in place, it should not be removed until an assessment of bowel sounds and nausea and vomiting is done.

A client is prescribed ranitidine 150 mg daily to treat peptic ulcer disease (PUD). Which instruction would the nurse give to the client about when to take this medication? As needed With meals At bedtime When indigestion occurs

At bedtime Ranitidine is administered typically in a single dose at bedtime. This medication is used for 4 to 6 weeks in combination with other therapy; it is not used as needed, with meals, or when indigestion occurs.

A client is known to be on lithium therapy for the treatment of depression. What is the expected adverse effect of this drug? Ataxia Confusion Blurred vision Paradoxical anxiety

Ataxia Ataxia is the expected adverse effect in clients who are undergoing lithium therapy for the treatment of depression. Confusion is one of the adverse effects of diazepam. Blurred vision and paradoxical anxiety are the adverse effects of buspirone.

The nurse is providing induction therapy to a client to prevent rejection after an organ transplant. Which medication will the nurse most likely administer? Atgam Mycophenolate Acetaminophen Diphenhydramine

Atgam Atgam is a lymphocyte immune globulin, which is administered as an induction therapy or to treat acute rejection in organ transplant individuals. The purpose of induction therapy is to severely immunosuppress individuals immediately after transplantation to prevent early rejection in a client who has had an organ transplant. Mycophenolate has been shown to decrease the incidence of late graft loss; side effects include gastrointestinal toxicities. Acetaminophen and diphenhydramine are administered to treat headache, myalgias, and rigors that may occur with muromonab-CD3.

Which medication should be listed as the antidote to a nerve agent in the disaster plan for a terrorist attack? Atropine Dopamine Epinephrine Norepinephrine

Atropine Atropine should be listed as the antidote for nerve agent poisoning in the disaster plan for a terrorist attack. Nerve gases cause the patient's system to be flooded with acetylcholine, which results in an overstimulation of the nerve cell. Atropine works by blocking the acetylcholine receptors. Therefore the receptors will not pick up the acetylcholine. Dopamine, epinephrine, and norepinephrine are not medications used to treat nerve agent poisoning.

While assessing the pupils of a client, a healthcare professional notices pupillary dilatation. Which drug intake might have resulted in this condition? Heroin Atropine Morphine Pilocarpine

Atropine The intake of eye medications such as atropine will cause dilatation of the pupils. Heroin, morphine, and pilocarpine cause pupillary constriction.

While a pacemaker catheter is being inserted, the client's heart rate drops to 38 beats/min. What medication should the nurse expect the healthcare provider to prescribe? Digoxin Lidocaine Amiodarone Atropine sulfate

Atropine sulfate Atropine blocks vagal stimulation of the sinoatrial (SA) node, resulting in an increased heart rate. Digoxin slows the heart rate; hence it would not be indicated in this situation. Lidocaine decreases myocardial sensitivity and will not increase the heart rate. Amiodarone is an antidysrhythmic drug used for ventricular tachycardia; it will not stimulate the heart rate.

Which nursing interventions would help to ensure the safe administration of antiepileptics to children? Avoid carbonated beverages while taking valproic acid Use a graduated device to deliver an oral dose Administer oral forms of valproic acid with milk Encourage the wearing of a medical alert bracelet Maintain a record of symptoms of seizures before, during, and after treatment with anti-epileptics

Avoid carbonated beverages while taking valproic acid Use a graduated device to deliver an oral dose Encourage the wearing of a medical alert bracelet Maintain a record of symptoms of seizures before, during, and after treatment with anti-epileptics The nurse should advise a child who is taking valproic acid to avoid carbonated beverages because they cause gastric irritation. A graduated device should be used for the accurate measurement of liquids. The nurse should encourage the child to wear a medical alert bracelet all the time that contains information about the child's allergies, diagnosis, and drug therapy. The nurse should maintain a record of any symptoms of seizures before, during, and after treatment to prevent the progression of disease. Oral forms of valproic acid should not be taken with milk because they may cause mucosal irritation.

A client is recovering from a kidney transplant. Which medications should the nurse expect to be prescribed for this client's maintenance therapy? Basiliximab Azathioprine Prednisone Cyclosporine Antithymocyte globulin-equine

Azathioprine Prednisone Cyclosporine Antithymocyte globulin-equine Maintenance therapy is the continuous immunosuppression used after a solid organ transplant. The drugs used for routine therapy after solid organ transplantation include an antiproliferative agent such as azathioprine, a corticosteroid such as prednisone, and a calcineurin inhibitor such as cyclosporine. Baxiliximab is a monoclonal antibody used to treat acute rejection episodes. Antithymocyte globulin-equine is a polyclonal antibody used to treat acute rejection episodes.

Which topical agent would be beneficial in preventing new lesions and treating preexisting acne? Doxycycline Azelaic acid Isotretinoin Azithromycin

Azelaic acid Azelaic acid, a topical antibacterial, is used to prevent new lesions and treat existing acne. Doxycycline and isotretinoin may both be used to treat acne, but these are administered orally, not topically. Azithromycin is not a drug of choice for treating acne.

What would be the drug of choice for a client who is diagnosed with chlamydia? Imiquimod Ceftriaxone Azithromycin Benzathine penicillin

Azithromycin Azithromycin is recommended for clients with chlamydia infections. Imiquimod is beneficial for treating genital warts in clients with human papillomavirus infections. Ceftriaxone is the drug of choice for treating gonorrhea. Benzathine penicillin is recommended for treating syphilis.

A client with cirrhosis of the liver and ascites has been taking chlorothiazide, a thiazide diuretic. Why did the provider add spironolactone to the client's medication regimen? A.To stimulate sodium excretion B. To help prevent potassium loss C. To increase urine specific gravity D. To reduce arterial blood pressure

B

A client with myasthenia gravis improves and is discharged from the hospital. The discharge medications include pyridostigmine bromide 10 mg every 6 hours. The nurse evaluates that the drug regimen is understood when the client makes which statement? A. "I will take the medication on an empty stomach." B. "I need to set an alarm so I take the medication on time." C. "It will be important to check my heart rate before taking the medication." D. "I should monitor for an increase in blood pressure after taking the medication."

B

A healthcare provider prescribes enoxaparin 30 mg subcutaneously daily. Which measure would the nurse take when administering this medication? A. Push over 2 minutes. B.Administer in the abdomen. C.Rub site after administration. D. Remove air pocket from prepackaged syringe before administration.

B

Which side effect should the nurse monitor for when administering androgen therapy? Baldness Headaches Gastric irritation Orthostatic hypotension

Baldness Androgen therapy may cause baldness, gynecomastia, and acne. Headaches, gastric irritation, and orthostatic hypotension are associated with bromocriptine, which is used to treat hyperpituitarism.

A client admitted for uncontrolled hypertension and chest pain was prescribed a low-sodium diet and started on furosemide. The nurse should instruct the client to include which foods in the diet? Liver Apples Cabbage Bananas

Bananas Furosemide is a loop diuretic that eliminates potassium by preventing renal absorption. Bananas have a significant amount of potassium. Bananas: 450 mg; cabbage: 243 mg; liver: 73.6 mg; apples: 100-120 mg.

A client with depression has not responded to a tricyclic antidepressant and outpatient electroconvulsive therapy (ECT). The healthcare provider prescribes selegiline, and the nurse teaches the client about food to be avoided while taking this medication. Which foods identified by the client allow the nurse to conclude that the instructions have been understood? Fresh fish Beer Fried chicken Licorice Leafy vegetables

Beer Licorice Foods containing tyramine can cause hypertensive crisis and should be eliminated from the diet. These foods include pickled herring, beer, wine, chicken livers, aged or natural cheese, caffeine, cola, licorice, avocados, bananas, and bologna. Chocolate in moderation is safe for some patients, but it does contain caffeine. Overripe fruits and caffeine have high levels of tyramine, which can cause dangerous hypertension in clients taking monoamine oxidase inhibitors (MAOIs). Also, large amounts of caffeine can increase blood pressure and should be avoided. There is no need to limit the intake of fish, chicken, or leafy vegetables while taking an MAOI.

Which medication is the first-line treatment for acne in adolescents? Tretinoin Doxycycline Clindamycin Benzoyl peroxide

Benzoyl peroxide Benzoyl peroxide is an effective first-line medication used to treat acne in adolescents; this medication is effective against inflammatory and noninflammatory acne. Tretinoin, doxycycline, and clindamycin are also used to treat acne, but they are not first-line agents.

During a follow-up visit the client, who is undergoing treatment for mental illness, complains of painful muscle spasms. The nurse suspects that the client may have pseudoparkinsonism. Which drugs does the nurse anticipate to be prescribed by the primary healthcare provider to counter this condition? Clozapine Benztropine Haloperidol Risperidone Trihexyphenidyl

Benztropine Trihexyphenidyl The client's painful muscle spasms may be the symptom of pseudoparkinsonism. Benztropine and trihexyphenidyl are the anticholinergic drugs used to treat painful muscle spasms caused by antipsychotic drugs. Clozapine, haloperidol, and risperidone are the antipsychotic drugs that may be the cause of this adverse effect.

A client has recently started taking a new neuroleptic drug, and the nurse notes extrapyramidal effects. Which drug does the nurse anticipate will be prescribed to limit these side effects? Zolpidem Hydroxyzine Dantrolene Benztropine mesylate

Benztropine mesylate Benztropine, an anticholinergic, helps balance neurotransmitter activity in the central nervous system (CNS) and helps control extrapyramidal tract symptoms. Zolpidem is a sedative-hypnotic drug used for short-term insomnia. Hydroxyzine is a sedative that depresses activity in the subcortical areas in the CNS; it is used to reduce anxiety. Dantrolene, a muscle relaxant, has a direct effect on skeletal muscle by acting on the excitation-contraction coupling of muscle fibers and not at the level of the CNS, unlike most other muscle relaxation drugs.

Which prostaglandin agonist is used in the treatment of clients with glaucoma? Carteolol Bimatoprost Brinzolamide Apraclonidine

Bimatoprost Bimatoprost is the prostaglandin agonist used in the treatment of glaucoma. Carteolol is the beta-adrenergic blocker used for treatment of glaucoma. Brinzolamide is the carbonic anhydrate inhibitor used for the treatment of glaucoma. Apraclonidine is the adrenergic agonist used in the treatment of glaucoma.

A client with an intractable infection is receiving vancomycin. Which laboratory blood test result should the nurse report? Hematocrit: 45% Calcium: 9.0 mg/dL (2.25 mmol/L) White blood cells (WBC): 10,000 mm 3 (10 X 10 9/L) Blood urea nitrogen (BUN): 30 mg/dL (10.2 mmol/L)

Blood urea nitrogen (BUN): 30 mg/dL (10.2 mmol/L) Vancomycin is a nephrotoxic medication. An elevated BUN can be an early sign of toxicity. The BUN of a healthy adult is 10 to 20 mg/dL (3.6-7.1 mmol/L). This hematocrit is expected in a healthy adult; the range is from 40 to 52. The expected range of the WBC count is 5,000 to 10,000 mm 3 (5-10 X 10 9/L) for a healthy adult. This calcium level is within the expected range of 9.0 to 10.5 (2.25-2.75 mmol/L) for a healthy adult.

A nurse is assessing the status of a school-aged child with leukemia who is receiving vincristine. What laboratory test result alerts the nurse that fluid intake should be increased? Urine pH of 6 Urine specific gravity of 1.020 Blood uric acid level of 7.5 mg/dL Blood urea nitrogen level of 15 mg/dL

Blood uric acid level of 7.5 mg/dL The normal blood uric acid level for children ranges from 2.5 to 5.5 mg/dL. An increase in the uric acid level caused by the destruction of cells may lead to renal problems; increased fluid intake helps dilute the urine. A urine pH of 6 is within the expected range of 4.5 to 8. A urine specific gravity of 1.020 is within the expected range of 1.005 to 1.030 (usually 1.010 to 1.025). A blood urea nitrogen level of 15 mg/dL is within the expected range of 5 to 18 mg/dL.

A nurse is providing discharge instructions about digoxin. Which response should a nurse include as a reason for a client to withhold the digoxin? Chest pain Blurred vision Persistent hiccups Increased urinary output

Blurred vision Visual disturbances, such as blurred or yellow vision, may be evidence of digoxin toxicity. Chest pain is not a toxic effect of digoxin. Persistent hiccups are not related to digoxin toxicity. An increased urinary output is not a sign of digoxin toxicity; it may be a sign of a therapeutic response to the drug and an improved cardiac output.

A client is on tobramycin therapy. Which assessment findings should be given priority? Throat sores Blurred vision Watery diarrhea Hearing impairment Decreased sense of smell

Blurred vision Hearing impairment Decreased sense of smell The client may suffer blurred vision, hearing impairment, or decreased sense of smell as side effects associated with tobramycin. These findings should be given high priority, and measures to reverse the toxicity should be taken so as to prevent permanent damage. Excessive use of tobramycin is associated with side effects that include nephrotoxicity, neurotoxicity, and hearing deficit. Neurotoxicity results in damage to the nerves that affect the functioning of sensory organs (e.g., eyes, ears, nose, skin). Throat sores and watery diarrhea are the symptoms of cephalosporins such as cephazolin.

A client who is visiting the family planning clinic is prescribed an oral contraceptive. As part of teaching, the nurse plans to inform the client of the possibility of which adverse effect? Cervicitis Ovarian cysts Fibrocystic breasts Breakthrough bleeding

Breakthrough bleeding Breakthrough bleeding, or midcycle bleeding, commonly occurs when women start using oral contraceptives. If it persists, the dosage should be changed. There is no evidence that cervicitis, ovarian cysts, or fibrocystic breasts are related to the use of oral contraceptives.

A friend asks a student nurse whether there is a drug marketed for smoking cessation. The student nurse researches a drug textbook to find the answer to this question. What antidepressant drug does the student nurse learn is available for this clinical application? Diazepam Bupropion Fluvoxamine Chlordiazepoxide

Bupropion Bupropion is marketed for the expressed use of smoking cessation, because it decreases craving. Diazepam is an antianxiety drug and it is not used for smoking cessation. Fluvoxamine, a selective serotonin reuptake inhibitor antidepressant, is most often prescribed for individuals with obsessive-compulsive disorder; it is not used for smoking cessation, because nicotine may decrease its effectiveness. Chlordiazepoxide is an antianxiety medication used to diminish alcohol withdrawal symptoms.

The nurse noticed increased blood pressure in a client on treatment for depression. Which antidepressant drug does the nurse ask the primary healthcare provider to reconsider? Fluoxetine Bupropion Trazodone Mirtazapine

Bupropion Bupropion is the antidepressant drug used in the treatment of depression. The adverse effect of this drug is increased blood pressure. Fluoxetine is used in the treatment of depression. Anxiety and insomnia are the adverse effects of this drug. Trazodone is used in the treatment of depression. Sedative effects are the adverse effects of this drug. Mirtazapine is used in the treatment of depression. Drowsiness and abnormal dreams are the adverse effects of this drug.

A client in active labor becomes very uncomfortable and asks a nurse for pain medication. Nalbuphine is prescribed. How does this medication relieve pain? By producing amnesia By acting as a preliminary anesthetic By inducing sleep until the time of birth By acting on opioid receptors to reduce pain

By acting on opioid receptors to reduce pain Nalbuphine is classified as an opioid analgesic and is effective in relieving pain; it induces little or no newborn respiratory depression. Nalbuphine does not induce amnesia, act as an anesthetic, or induce sleep.

Loratadine, 10 mg by mouth once a day in the morning, is prescribed for a 15-year-old girl with hay fever. The girl tells the school nurse that she is concerned that she will be sleepy for a quiz the next day. How should the nurse respond? By explaining that this medication rarely causes drowsiness By advising her to take half a tablet in the morning before school By suggesting that she skip the next day's dose if she can tolerate the hay fever By recommending that she call the allergist for a prescription containing a stimulant

By explaining that this medication rarely causes drowsiness Loratadine causes little or no drowsiness or anticholinergic side effects. Even if the medication did cause drowsiness, the nurse does not have the legal authority to alter the prescribed dose. It is not necessary to call the allergist because loratadine rarely causes drowsiness.

A nurse is assessing the therapeutic action of drugs classified as tumor necrosis factor (TNF) inhibitors. What client response indicates to the nurse that a drug with this classification is effective? A. Continued remission in a client with ovarian cancer B. Increased insulin production in a client with diabetes mellitus C. Reduction of inflammatory joint pain in a client with rheumatoid arthritis D. Vasodilation of coronary arteries in a client with ischemic heart disease

C

The health care provider prescribes an oral hypoglycemic for the patient with type 2 diabetes. What will the nurse need to consider when developing the teaching plan? A. Oral hypoglycemics work by decreasing absorption of carbohydrates. B. Oral hypoglycemics work by stimulating the pancreas to produce insulin. C. Clients taking oral hypoglycemics may subconsciously relax dietary rules to gain a sense of control. D. Clients with type 2 diabetes do not need to be concerned about serious adverse effects from oral hypoglycemics.

C

The nurse suspects serotonin syndrome in a client prescribed second-generation antidepressants for depression. Which assessment findings observed by the nurse would be beneficial in diagnosing the severity of the syndrome? SATA A. Delirium B. Hyperreflexia C. Hyperthermia D. Rhabdomyolysis

C,D

While assessing a client with presence of nephrotoxicity, lymphoma, and hypertension, the medical history reveals that the client is on immunosuppressant drug therapy. Which drug class might have caused these conditions? Corticosteroids Cytotoxic drugs Monoclonal antibodies Calcineurin inhibitors

Calcineurin inhibitors Calcineurin inhibitors such as cyclosporine act on T helper cells to prevent production and release of IL-2 and γ-interferon. This class of medications can cause adverse effects such as nephrotoxicity, lymphoma, hypertension, gingival hyperplasia, and hirsutism. Corticosteroids may cause peptic ulcer, osteoporosis, and hyperglycemia. Cytotoxic drugs may cause bone marrow suppression, hypertension, diarrhea, and nausea. Monoclonal antibodies may cause pulmonary edema, hypersensitivity reactions, fever/chills, and chest pain.

A client is experiencing both tingling of the extremities and tetany. What should the nurse anticipate will be prescribed by the healthcare provider? Dialysis Calcium supplements Mechanical ventilation Intravenous fluids with potassium

Calcium supplements Paresthesias and tetany are signs of hypocalcemia, which is corrected by the administration of calcium. Dialysis is indicated for hyperkalemia and renal failure. Mechanical ventilation is indicated for respiratory insufficiency. Intravenous fluids with potassium are indicated for hypokalemia.

A client is receiving doxorubicin as part of a chemotherapy protocol. The nurse should assess the client for which major life-threatening side effect of doxorubicin? Anemia Cardiotoxicity Pulmonary fibrosis Ulcerative stomatitis

Cardiotoxicity Heart failure and dysrhythmias are the primary life-threatening toxic effects unique to doxorubicin. When bone marrow is depressed to precarious levels, the dose is altered or blood components administered. Pulmonary fibrosis is not an adverse effect of doxorubicin or of any of the other antineoplastic agents. Ulcerative stomatitis is an uncomfortable side effect of doxorubicin, but it is not life threatening as are the primary life-threatening toxic effects unique to doxorubicin.

Which adverse effect on pediatric clients is associated with nalidixic acid? Kernicterus Gray syndrome Tendon rupture Cartilage erosion

Cartilage erosion Nalidixic acid can cause cartilage erosion in pediatric clients. Sulfonamides can cause kernicterus. Chloramphenicol can cause Gray syndrome in neonates and infants. Fluoroquinolones can cause tendon rupture in pediatric clients.

A client with inflammatory bowel disease is receiving total parenteral nutrition (TPN) via an infusion pump. What is most important for the nurse to do when administering TPN? Change the TPN solution bag every 24 hours, even if there is solution left in the bag. Monitor the client's blood glucose level every 2 hours at the bedside with a glucometer. Instruct the client to breathe shallowly when changing the TPN tubing using sterile techniques. Speed up the rate of the TPN infusion if the amount delivered has fallen behind the prescribed hourly rate.

Change the TPN solution bag every 24 hours, even if there is solution left in the bag. TPN solutions are high in glucose and are administered at room temperature, factors that increase the risk of microbial growth in the solution; they should be changed daily or sooner if they appear cloudy. Monitoring the blood glucose level every 2 hours is too frequent in ordinary circumstances; the client's blood glucose level should be monitored every 4 to 6 hours to identify the presence of hyperglycemia, a metabolic complication of TPN. The client should not breathe while the TPN catheter is changed because it may result in an air embolus; the Valsalva maneuver should be performed by the client for the few seconds it takes to switch the tubing. An excess amount of glucose will be infused if the rate of the TPN is increased, and the endogenous insulin will be inadequate to meet this demand, resulting in hyperglycemia.

A nurse understands that after the administration of alprazolam it is important to assess the client for side effects. What will the nurse do initially? Measure urine output. Check the blood pressure. Look for abdominal distention. Check the size of the pupils frequently.

Check the blood pressure. Hypotension is a major side effect of alprazolam that occurs early in therapy. An alteration in urine output is not a common side effect; however, urine retention may occur after prolonged use. Abdominal distention is not a common side effect, but abdominal distention from constipation may occur after prolonged use. Blurred vision, not dilated pupils, may occur. Dilated pupils associated with central nervous system depression are not a common side effect, although they may occur with overdose or prolonged use.

Which nursing interventions may promote safe drug administration in a child diagnosed with heart failure who is receiving digoxin? Checking for compliance with the client's drug regimen Monitoring the client's serum potassium and magnesium levels regularly Administering digoxin only through the intramuscular route Calculating the correct dosage form, prescribed amounts, and the prescriber's order Monitoring and recording the client's intake and output, heart rate, blood pressure, daily weight, and respiration rate regularly

Checking for compliance with the client's drug regimen Monitoring the client's serum potassium and magnesium levels regularly Calculating the correct dosage form, prescribed amounts, and the prescriber's order Monitoring and recording the client's intake and output, heart rate, blood pressure, daily weight, and respiration rate regularly

An estrogen-progestin oral contraceptive is prescribed for a client. Which adverse effects should the nurse teach the client to report to the primary healthcare provider? Lethargy Dizziness Chest pain Constipation Unilateral breast soreness Calf tenderness

Chest pain Unilateral breast soreness Calf tenderness Estrogen-progestin contraceptives have been associated with breast malignancy; clinical manifestations include breast soreness, thickening, and lack of symmetry. These contraceptives have also been associated with thrombophlebitis; clinical manifestations of thrombophlebitis include calf tenderness and redness and heat over the affected area. If the clot travels, it could present as a pulmonary embolism, so chest pain should be reported. Lethargy, dizziness, and constipation are not expected side effects of this medication.

A client with rheumatoid arthritis does not want the prescribed cortisone and informs the nurse. Later, when the nurse attempts to administer cortisone, the client asks what the medication is and the nurse gives an evasive answer. The client takes the medication and later discovers that it was cortisone. The client states an intent to sue. What factors in this situation must be considered in a legal action? Clients have a right to refuse treatment. Nurses are required to answer clients truthfully. The healthcare provider should have been notified. The client had insufficient knowledge to make such a decision. Legally prescribed medications are administered despite a client's objections.

Clients have a right to refuse treatment. Nurses are required to answer clients truthfully. The healthcare provider should have been notified. Clients who are mentally competent have the right to refuse treatment; the nurse must respect this right. A client's questions must always be answered truthfully. The healthcare provider should be notified when a client refuses an intervention so that an alternative treatment plan can be formulated. This is done after the nurse explores the client's reasons for refusal. The client had a discussion with the nurse that indicated that the client had sufficient information to make the decision to refuse the medication. The client has a right to refuse treatment; this right takes precedence over the healthcare provider's prescription.

The nurse is preparing a teaching plan for clients receiving antitubercular medications. Which teaching plan needs correction? Clients taking ethambutol should drink plenty of fluids. Clients taking Isoniazid should take the drug with food. Clients taking pyrazinamide should wear a hat while going out in the sun. Clients taking rifampin should use other contraceptive methods even after stopping the medication.

Clients taking Isoniazid should take the drug with food. Taking isoniazid with food should be corrected. The presence of food may slow or even prevent the absorption of isoniazid from the stomach. Therefore the client should take the medication on an empty stomach, either 1 hour before or 2 hours after eating. Ethambutol may increase uric acid formation. The client should drink plenty of water to reduce uric acid precipitation and kidney problems. Pyrazinamide is a photosensitive medication that may increase the risk of sunburn. Therefore the client should wear a hat and protect himself or herself from sun exposure. Rifampin may decrease the efficiency of oral contraceptives. Therefore the nurse should instruct the client to use an additional method of contraception even after stopping the medication.

A pregnant adolescent seeks treatment for blackheads. Which drug would be safe for the adolescent? Tretinoin Isotretinoin Doxycycline Clindamycin

Clindamycin Blackheads are noninflammatory open comedones. For such a condition, topical clindamycin is the drug of choice, and it is also safe for use during pregnancy. Tretinoin, isotretinoin, and doxycycline are contraindicated during pregnancy.

A client visited the primary healthcare provider complaining of inflammatory lesions on the face. The primary healthcare provider examined and confirmed it as an inflammatory disorder of the sebaceous glands. Which medications does the nurse anticipate being prescribed by the primary healthcare provider? Bacitracin Mupirocin Clindamycin Erythromycin Metronidazole

Clindamycin Erythromycin Clindamycin and erythromycin are topical antibiotics used in the treatment of acne vulgaris, which occurs due to inflammation of the sebaceous glands. Bacitracin is an over-the-counter topical antibiotic used in the treatment of dermatologic problems. Mupirocin is used in the treatment of superficial Staphylococcus infections such as impetigo. Topical metronidazole is used in the treatment of rosacea and bacterial vaginosis.

Which drug used to promote fertility may cause esophageal burns? Estrogen Clomiphene Nifedipine Indomethacin

Clomiphene Clomiphene is a serum selective receptor modulator that may cause esophageal burns. Estrogen may cause a thromboembolism. Nifedipine may cause maternal fetal problems. Indomethacin may cause birth defects.

Which drug does the nurse anticipate to be prescribed to a client seeking treatment for infertility? Clomiphene Misoprostol Dinoprostone Methylergonovine

Clomiphene Clomiphene causes the maturation of ovarian follicles, which leads to ovulation. This drug is used to promote fertility. Misoprostol and dinoprostone are prostaglandins that cause uterine muscle contractions. Methylergonovine is an ergot alkaloid used to reduce postpartum uterine hemorrhage.

A client who underwent treatment for infertility gave birth to triplets. She often complained of breast pain during the infertility treatment. Which drug may have been administered to the client for infertility treatment? Estradiol Haloperidol Clomiphene Promethazine

Clomiphene Clomiphene is an ovarian stimulant that may cause multiple pregnancies. Breast pain is an adverse effect of clomiphene. Estradiol helps to reduce postmenopausal hot flashes. Haloperidol and promethazine are contraindicated while using clomiphene.

What is the best drug of choice for treating obsessive-compulsive disorder? Imipramine Lithium salts Amitriptyline Clomipramine

Clomipramine Clomipramine is a tricyclic antidepressant drug prescribed for treating of obsessive-compulsive disorder. Childhood enuresis necessitates the administration of imipramine. Lithium salt is prescribed to treat bipolar disorders. Dysthymias can be treated by the administration of antidepressant drugs such as amitriptyline.

A client is admitted for treatment of obsessive-compulsive disorder that is interfering with activities of daily living. Which medication does the nurse anticipate the primary healthcare provider will prescribe? Benztropine Amantadine Clomipramine Diphenhydramine

Clomipramine Clomipramine potentiates the effects of serotonin (antiobsessional effect) and norepinephrine in the central nervous system; it diminishes obsessive-compulsive behaviors. Benztropine and amantadine are antiparkinsonian agents, not antianxiety agents. Diphenhydramine is an antihistamine, not an antianxiety agent.

Which statement is true regarding antipsychotic drugs? All first- and second-generation antipsychotics are equally effective. Second-generation antipsychotics pose a risk of extrapyramidal symptoms. First-generation antipsychotics pose a significant risk of metabolic side effects. Clozapine is more effective than other second-generation antipsychotics.

Clozapine is more effective than other second-generation antipsychotics. Clozapine is a second-generation antipsychotic drug that is more effective than other second-generation antipsychotics. Most (but not all) first- and second-generation antipsychotics are equally effective. Second-generation antipsychotics may cause metabolic side effects such as diabetes and dyslipidemia. First-generation antipsychotics may cause extrapyramidal side effects.

A client with Hodgkin disease enters a remission period and remains symptom free for 6 months before a relapse occurs. The client is diagnosed at stage IV. What therapy option does the nurse expect to be implemented? Radiation therapy Combination chemotherapy Radiation with chemotherapy Surgical removal of the affected nodes

Combination chemotherapy A protocol consisting of three or four chemotherapeutic agents that attack the dividing cells at various phases of development is the therapy of choice at this stage; alternating courses of different protocols generally are used. Radiation, alone or in combination with chemotherapy, is used in stages IA, IB, IIA, IIB, and IIIA. Radiation with chemotherapy is recommended for use in stage IIIA. Surgical removal of the affected nodes is not a therapy for Hodgkin disease at any stage. The nodes may be removed for biopsy or irradiated as part of therapy.

A client who is to receive nitrogen mustard as part of a drug protocol for cancer asks how this drug works in the body. What does the nurse identify as the drug's mechanism of action? Interference of the cellular protein synthesis Inhibition of the synthesis of purine and pyrimidine Binding with DNA to interfere with RNA production Combining with DNA strands and interfering with cell replication

Combining with DNA strands and interfering with cell replication Alkylating agents, of which nitrogen mustard is one, combine with DNA strands and interfere with cell replication. Some chemotherapeutic drugs are believed to act by interfering with cellular protein synthesis, but nitrogen mustard does not. Inhibiting the synthesis of purine and pyrimidine is the mechanism of action of antimetabolites. Antibiotics, not nitrogen mustard, used in cancer chemotherapy are believed to act by binding with DNA to interfere with RNA production.

Sublingual nitroglycerin tablets are prescribed to control periodic episodes of chest pain in the patient with stable angina. Which instruction should the nurse include when teaching the client about sublingual nitroglycerin? Once the tablet is dissolved, spit out the saliva. Take tablets 3 minutes apart up to a maximum of five tablets. Common side effects include headache and low blood pressure. Once opened, the tablets should be refrigerated to prevent deterioration.

Common side effects include headache and low blood pressure. The primary side effects of nitroglycerin are headache and hypotension. It is not necessary to spit out saliva into which nitroglycerin has dissolved. For pain that is not relieved, additional tablets may be taken every 5 minutes up to a total of three tablets. It should be stored at room temperature.

A toddler with a puncture wound to the sole is brought to the emergency department. Because of a language barrier the caregiver cannot provide a clear history of previous tetanus immunizations. Tetanus immunoglobulin (TIG) is prescribed by the healthcare provider. The nurse explains to the caregiver that this medication is given because it has what action? Produces lifelong passive immunity to tetanus Confers short-term passive defense against tetanus Induces long-lasting active protection from tetanus Stimulates the production of antibodies to fight tetanus

Confers short-term passive defense against tetanus Tetanus immunoglobulin contains antibodies, not the live or attenuated virus; it confers short-term passive immunity that is temporary. Tetanus toxoid, not TIG, stimulates the production of antibodies.

The nurse is providing teaching to a client who recently has been diagnosed with type 1 diabetes. The nurse reinforces the importance of monitoring for ketoacidosis. What are the signs and symptoms of ketoacidosis? Confusion Hyperactivity Excessive thirst Fruity-scented breath Decreased urinary output

Confusion Excessive thirst Fruity-scented breath Diabetic ketoacidosis signs and symptoms often develop quickly, sometimes within 24 hours. Diabetic ketoacidosis is a serious complication of diabetes that occurs when the body produces high levels of ketones (blood acids). Diabetic ketoacidosis develops when the body is unable to produce enough insulin. Without enough insulin, the body begins to break down fat as an alternative fuel. This process produces a buildup of ketones (toxic acids) in the bloodstream, eventually leading to diabetic ketoacidosis if untreated. Signs and symptoms include excessive thirst, frequent urination, nausea and vomiting, abdominal pain, weakness or fatigue, shortness of breath, fruity-scented breath, and confusion. Frequent urination, not decreased urination, is a symptom. Weakness or fatigue, not hyperactivity, is a symptom.

A pregnant woman reports night sweats, sleep disturbances, and weight gain. Upon diagnosis, the woman has low levels of estrogen and is treated with hormonal therapy. Which teratogenic effect is likely to occur in the newborn? Goiter Neural tube defects Cleft lip with cleft palate Congenital defects of the female reproductive system

Congenital defects of the female reproductive system Estrogen may cause congenital defects of the female reproductive system in the fetus. Antithyroid drugs may cause a goiter. Carbamazepine may cause neural tube defects. Nitrofurantoin may cause cleft lip with cleft palate.

While assessing the skin of a client, the nurse notices edema at the dorsum of the foot and ankle. Which pre-disposing condition does the nurse anticipate in the client? Neurotrauma Hypothyroidism Hyperthyroidism Congestive heart failure

Congestive heart failure Edema at the dorsum of foot and ankle may be due to congestive heart failure; therefore congestive heart failure is the pre-disposing condition. Neurotrauma is the condition in which there is increased temperature caused by increased blood flow to the skin. Hypothyroidism is caused due to an endocrine imbalance. Hyperthyroidism is the condition caused by an increase in moisture content.

A nurse is planning care for a client with cancer who is receiving the plant alkaloid vincristine. In contrast to the side effects of most chemotherapeutic agents, what is a common side effect of vincristine that the nurse must address in the client's care plan? Nausea Alopecia Constipation Hyperuricemia

Constipation Although most chemotherapy causes diarrhea, vincristine can cause severe constipation, impaction, or paralytic ileus. Nausea, alopecia, and hyperuricemia are side effects shared with most other chemotherapeutic agents.

A client with schizophrenia who is receiving an antipsychotic medication begins to exhibit a shuffling gait and tremors. The primary healthcare provider prescribes the anticholinergic medication benztropine, 2 mg daily. What will the nurse assess the client for daily when administering these medications together? Constipation Hypertension Increased salivation Excessive perspiration

Constipation The anticholinergic activity of each drug is magnified, and adverse effects such as paralytic ileus may occur. Hypotension, not hypertension, occurs with anticholinergic medications. Dryness of the mouth, not increased salivation, occurs with anticholinergic medications. Decreased, not increased, perspiration occurs with anticholinergic medications.

Which statement related to teratogens and their effects needs to be corrected? Behavioral effects are difficult to observe. Controlled experiments can be done in humans. Animal studies may not be applicable to humans. Teratogens may not have immediate effects.

Controlled experiments can be done in humans. Controlled experiments are not easy to perform in humans because of various factors such as the risk to the fetus, risk of the death of the mother or the fetus before the experiment's completion, and the high doses of the drugs that may cause unwanted effects. Studies aiming to observe the behavioral effects are difficult to document because the behavior varies from person to person. Results from animal studies may not comply with those of human studies. Teratogens may cause immediate effects in some cases and may cause a delay in the effects in other cases.

A nurse teaches a client about Coumadin and concludes that the teaching is effective when the client agrees not to drink which juice? Apple juice Grape juice Orange juice Cranberry juice

Cranberry juice Antioxidants in cranberry juice may inhibit the mechanism that metabolizes warfarin, causing elevations in the international normalized ratio, resulting in hemorrhage. Apple juice, grape juice, and orange juice are fine to drink.

Which drug is most appropriate for relieving a painful muscle spasm in the back of a client with osteoarthritis (OA)? Tramadol Hyaluronate Diclofenac epolamine patch Cyclobenzaprine hydrochloride

Cyclobenzaprine hydrochloride Cyclobenzaprine hydrochloride is a muscle relaxant administered to relieve painful muscle spasms, especially those resulting from OA of the vertebral column. While tramadol is a weak opioid drug that may also be given to relieve pain in clients with OA, it is not as effective against painful muscle spasms. Hyaluronate is a specific injection for knee and hip pain associated with OA. The diclofenac epolamine patch is used in clients with signs and symptoms of knee OA.

A client has been taking 3 mg of risperidone twice a day for the past 8 days. At the follow-up appointment, the client reports tremors, shortness of breath, a fever, and sweating. What will the nurse do? A. Call 911 and have the client transported to the nearest psychiatric unit. B. Check the number of risperidone tablets left in the prescription bottle to see whether there was an overdose. C. Request a prescription for 2 mg of intramuscular benztropine stat and assess the client in 10 to 15 minutes for symptom relief. D. Take the client's vital signs and arrange for immediate transfer to a hospital

D

A pregnant client states to the nurse, "I have been advised to take an over-the-counter medication for the flu. The drug label says it is a category B drug." How should the nurse respond to these statements? A. "Because this drug causes fetal abnormalities, you should not take this drug." B. "This drug causes fetal risks when administered and should not be used in pregnancy." C. "This drug has been reported to have adverse effects in animal fetuses so this drug should be avoided." D. "This drug does not show risks to an animal fetus so you can safely take it while pregnant."

D

The alkylating chemotherapeutic agent cyclophosphamide is prescribed for a school-aged child with cancer. What is the most important nursing assessment while the child is receiving this medication? Extent of alopecia Changes in appetite Hyperplasia of gums Daily intake and output

Daily intake and output Hemorrhagic cystitis is a potentially serious adverse reaction to cyclophosphamide that can sometimes be prevented with increased fluid intake because the fluid flushes the bladder. The extent of hydration can be measured with hourly documentation of intake and output. Alopecia is expected; however, it is a benign side effect, and the hair will regrow when therapy is completed. A change in appetite is expected but is not a serious side effect of cyclophosphamide administration. Hyperplasia of the gums is unrelated to cyclophosphamide administration.

A client with a history of liver disease is found to have endometriosis. Which drug is contraindicated in this client? Danazol Celecoxib Leuprolide Ketoconazole

Danazol Danazol is a synthetic androgenic steroid that acts by suppressing secretion of follicle-stimulating hormone and luteinizing hormone. This results in decreased secretion of estrogen and progesterone and regression of endometrial tissue. It may result in decreased lipoprotein levels and an increase in low-density lipoprotein. It is contraindicated in clients with liver disease. Celecoxib, a nonsteroidal antiinflammatory drug, should be used with caution in liver disease. Leuprolide is a gonadotropin-releasing hormone (GnRH) agonist; it may be safe for use in clients with liver disease. Ketoconazole is a nonsteroidal antiinflammatory drug and should be used with caution in clients with liver disease.

Which teratogen causes masculinization of the female fetus? Danazol Vitamin A Topiramate Tetracycline

Danazol Danazol is an androgen that may cause masculinization of the female fetus as a teratogenic effect. Vitamin A may cause fetal cardiac defects. Topiramate may cause fetal growth delays. Tetracycline may cause tooth and bone abnormalities as teratogenic effects.

While assessing a client with schizophrenia who is receiving chlorpromazine, the nurse finds lead pipe rigidity, sudden high fever, and sweating. Which drugs would be prescribed by the healthcare provider? Loxapine Dantrolene Thiothixene Haloperidol Bromocriptine

Dantrolene Bromocriptine Lead pipe rigidity, sudden high fevers, and sweating are symptoms of neuroleptic malignant syndrome; this condition is an adverse effect of chlorpromazine. Drugs used to treat this syndrome are dantrolene and bromocriptine. Loxapine, thiothixene, and haloperidol are the first-generation antipsychotics that should not be prescribed because these may lead to severe complications.

What should the nurse consider as the goal of therapy when administering allopurinol to a client with gout? Increase bone density Decrease synovial swelling Decrease uric acid production Prevent crystallization of uric acid

Decrease uric acid production Allopurinol interferes with the final steps in uric acid formation by inhibiting the production of xanthine oxidase. This drug prevents the formation of uric acid; it does not affect bone density. Allopurinol has no effect on swelling of the synovial membranes. This medication prevents the synthesis of uric acid, not its crystallization.

A nurse administers lactulose to a client with cirrhosis of the liver. Which laboratory test change leads the nurse to determine that the lactulose is effective? Decreased amylase Decreased ammonia Increased potassium Increased hemoglobin

Decreased ammonia Lactulose destroys intestinal flora that break down protein and in the process give off ammonia. In clients with cirrhosis, ammonia is inadequately detoxified by the liver and can build to toxic levels. Amylase levels are associated with pancreatic problems. Increased potassium levels are associated with kidney failure. Hemoglobin is increased when the body needs more oxygen-carrying capacity, such as in smokers, or in high altitudes.

A client who takes insulin for type 1 diabetes has a psychosis and is to receive haloperidol. Which response does a nurse anticipate with this drug combination? Depressed respiration Intensified action of both drugs Decreased control of the diabetes Increased danger of extrapyramidal side effects

Decreased control of the diabetes Haloperidol alters the effectiveness of exogenous insulin, and the combination of haloperidol and insulin must be used with caution. The occurrence of respiratory depression is more likely with a combination of antipsychotics and barbiturates. Intensified action of both drugs would be more likely to occur if the antipsychotic were fluoxetine. There are no data to support a claim of increased danger of extrapyramidal side effects.

A newborn's mother is being treated with clemastine while breast-feeding. Which physiologic factors alter the pharmacokinetic properties of this drug in the neonate? Decreased fat content Increased protein binding Immature blood-brain barrier Increased first pass elimination Decreased glomerular filtration rate

Decreased fat content Immature blood-brain barrier Decreased glomerular filtration rate In neonates, the fat content is low due to a large amount of total body water. As a result, the drug readily enters the brain due to the neonate's immature blood-brain barrier. Neonates have immature kidneys; therefore, the glomerular filtration rate is decreased. Protein binding is decreased in neonates because the liver is immature and produces less protein. First stooling is delayed in neonates due to liver immaturity.

A client admitted with preeclampsia is receiving magnesium sulfate. Which assessment finding indicates that a therapeutic level of the medication has been reached? Increased fetal activity Decreased urine output Deep tendon reflexes of +2 Respiratory rate of 10 breaths/min

Deep tendon reflexes of +2 Hyperreflexia of severe preeclampsia is +3 to +4; therefore a deep tendon reflex of +2, which is an active, expected reflex, indicates that a therapeutic level of the drug has been reached. A diminished reflex or absence of the reflex indicates that the serum magnesium level is too high. Because magnesium sulfate is a central nervous system depressant, a respiratory rate of at least 12 breaths/min should be maintained. Alterations in fetal activity are not indicators of a therapeutic magnesium sulfate level. Oliguria is a sign of severe preeclampsia; diuresis is a therapeutic effect of magnesium sulfate administration.

What is the priority goal for a client with asthma who is being discharged from the hospital with prescriptions for inhaled bronchodilators? Is able to obtain pulse oximeter readings Demonstrates use of a metered-dose inhaler Knows the healthcare provider's office hours Can identify the foods that may cause wheezing

Demonstrates use of a metered-dose inhaler Clients with asthma use metered-dose inhalers to administer medications prophylactically or during times of an asthma attack; this is an important skill to have before discharge. Pulse oximetry is rarely conducted in the home; home management usually includes self-monitoring of the peak expiratory flow rate. Although knowing the healthcare provider's office hours is important, it is not the priority; during a persistent asthma attack that does not respond to planned interventions, the client should go to the emergency department of the local hospital or call 911 for assistance. Not all asthma is associated with food allergies.

A client is treated with lorazepam for status epilepticus. What effect of lorazepam does the nurse consider therapeutic? Slows cardiac contractions Dilates tracheobronchial structures Depresses the central nervous system (CNS) Provides amnesia for the convulsive episode

Depresses the central nervous system (CNS) Lorazepam, an anxiolytic and sedative, is used to treat status epilepticus because it depresses the CNS. Slower cardiac contractions are not an effect of lorazepam. Dilating tracheobronchial structures is not an effect of lorazepam. Providing amnesia for the convulsive episode is not an effect of lorazepam.

A nurse determines that the teaching about the side effects of azithromycin has been understood when the adolescent client identifies which problem as the most common side effect of this medication? Tinnitus Diarrhea Dizziness Headache

Diarrhea Diarrhea initially is related to gastrointestinal irritation; later it is related to loss of intestinal flora, which may lead to overgrowth of drug-resistant microbes, resulting in superinfection. This also causes diarrhea. Tinnitus, dizziness (vertigo), and headache all may occur, but none is the most common side effect.

A client is admitted to the acute medical unit for severe amphetamine intoxication. Which medications will a nurse anticipate will be prescribed to counteract the effects of stimulant intoxication? Diazepam Propranolol Benztropine Bupropion Amitriptyline

Diazepam Propranolol Because stimulants act by increasing both adrenaline and dopamine, seizures may occur. Diazepam can reduce the chance of seizures. Because amphetamines act by increasing adrenaline, which can stimulate the heart, propranolol, a beta-blocker, will decrease this adrenergic stimulation. Benztropine, a cholinergic blocker, is not indicated as a treatment for stimulant intoxication. Bupropion is contraindicated, because it increases dopamine and adrenaline, which will exacerbate stimulant intoxication. Amitriptyline is contraindicated, because it increases dopamine and adrenaline, which will exacerbate stimulant intoxication.

Which drug would place a client's offspring at risk for vaginal cancer? Danazol Estrogen Valproic acid Diethylstilbestrol

Diethylstilbestrol Diethylstilbestrol is a synthetic nonsteroidal estrogen used in the treatment of menopausal and postmenopausal disorders. When it is consumed during pregnancy, it may show the delayed teratogenic effect of vaginal cancer in female offspring by the age of 18. Danazol is an androgen that may cause masculinization of the female fetus. Estrogen causes congenital defects of the female reproductive organs. Valproic acid, a drug used to control seizures, may cause neural tube defects.

Which drug taken by a pregnant woman shows a delayed teratogenic effect in the offspring, making the effect difficult to identify? Aspirin Heparin Ethyl alcohol Diethylstilbestrol

Diethylstilbestrol Diethylstilbestrol may cause vaginal cancer in a female child 18 or more years after birth. Aspirin suppresses contractions during labor and may cause bleeding in the mother. Heparin does not cause fetal harm but may cause osteoporosis in the mother. Ethyl alcohol causes drug dependence or alcohol withdrawal syndrome in neonates.

A client reports nausea, vomiting, and seeing a yellow light around objects. A diagnosis of hypokalemia is made. Upon a review of the client's prescribed medication list, the nurse determines that what is the likely cause of the clinical findings? Digoxin (Lanoxin) Furosemide (Lasix) Propranolol (Inderal) Spironolactone (Aldactone)

Digoxin (Lanoxin) These are signs of digitalis toxicity, which is more likely to occur in the presence of hypokalemia. Although furosemide most likely contributed to the hypokalemia, the client's symptoms are consistent with digitalis toxicity. Although propranolol can cause nausea, vomiting, and blurred vision, the presence of hypokalemia and yellow vision are more suggestive of digitalis toxicity. A side effect of spironolactone is hyperkalemia, not hypokalemia.

Which drug aids in uterine evacuation in cases of miscarriage? Oxytocin Misoprostol Dinopristone Methylergonovine

Dinopristone Dinopristone is a prostaglandin E 2 abortifacient that causes uterine evacuation in cases of miscarriage. Oxytocin induces labor by enhancing uterine contractions and promotes milk ejection during lactation. Misoprostol is a stomach protectant that helps in cervical ripening. Methylergonovine is an oxytocic ergot alkaloid used to treat postpartum uterine atony and hemorrhage.

A pregnant woman who is past her due date is hospitalized for a labor induction. Which drug should be administered to the client? Clomiphene Menotropins Dinoprostone Choriogonadotropin alfa

Dinoprostone Dinoprostone is a prostaglandin that stimulates uterine contractions to promote the progression of labor. Clomiphene, menotropins, and choriogonadotropin alfa are fertility drugs that are used to increase the likelihood of conception in an infertile woman.

An infusion of oxytocin is administered to a client for induction of labor. After several minutes the uterine monitor indicates contractions lasting 100 seconds with a frequency of 130 seconds. What is the next nursing action? Discontinuing the infusion Checking the fetal heart rate Slowing the oxytocin flow rate Turning the client on her left side

Discontinuing the infusion Contractions lasting too long and occurring too frequently can lead to fetal hypoxia; stopping the oxytocin infusion should stop the contractions, thereby increasing oxygen flow to the fetus. The fetal heart rate should be monitored, but this is not the priority. Oxytocin will continue to promote uterine contractions; this is unsafe because the prolonged, frequent contractions decrease oxygen flow to the fetus. Turning the client on her left side will promote placental perfusion, but it is not the priority at this time.

A client sustains severe burns over 40% of the surface area of the body. The nurse is assigned to care for the client during the first 48 hours after the injury. What clinical finding does the nurse anticipate if the client develops water intoxication? Sooty-colored sputum Frothy, pink-tinged sputum Disorientation with twitching Urine output of 25 mL/hr

Disorientation with twitching Excess extracellular fluid moves into cells (water intoxication). Intracellular fluid excess in sensitive brain cells causes altered mental status; other signs include anorexia, nausea, vomiting, twitching, sleepiness, and convulsions. Sooty-colored sputum indicates inhalation of smoke or flames. Frothy, pink-tinged sputum is associated with pulmonary edema. Decreased urinary output indicates insufficient fluid replacement or dehydration.

The nurse is preparing discharge instructions for a client who was prescribed enalapril for treatment of hypertension. Which instruction is appropriate for the nurse to include in the client's teaching? Do not change to a standing position suddenly. Lightheadedness is a common adverse effect that need not be reported. The medication may cause a sore throat for the first few days. Schedule blood tests weekly for the first 2 months.

Do not change to a standing position suddenly. Enalapril is classified as an angiotensin-converting enzyme (ACE) inhibitor. It is used to treat hypertension and congestive heart failure. It can also be used to treat a disorder of the ventricles. Angiotensin is a chemical that causes the arteries to become narrow. ACE inhibitors help the body produce less angiotensin, which helps the blood vessels relax and open up, which, in turn, lowers blood pressure. Clients should be advised to change positions slowly to minimize orthostatic hypotension. A healthcare provider should be notified immediately if the client is experiencing lightheadedness or feeling like he or she is about to faint, as this is a serious side effect. This medication does not cause a sore throat the first few days of treatment. Presently, there are no guidelines that suggest blood tests are required weekly for the first 2 months.

Ten minutes after administering nalbuphine via intravenous piggyback to a primigravida in active labor, the nurse notes a fetal heart rate of 132 with minimal variability. The client states that the pain is more tolerable and she is able to use her breathing techniques more effectively. Contractions continue every 2 to 3 minutes and are of 60 seconds' duration. What is the nurse's next action? Reposition the client on the left side to increase placental perfusion. Administer oxygen via mask to minimize apparent fetal compromise. Have an opioid antagonist available to be administered to the infant at the time of birth. Document the findings, including the stable fetal heart rate variability after administering the opioid infusion.

Document the findings, including the stable fetal heart rate variability after administering the opioid infusion. A common side effect of an opioid analgesic is decreased fetal heart rate variability. Because the fetal heart rate and the length and duration of the contractions remain stable and the analgesic appears to be effective, the only nursing action is to document the findings. Repositioning the client is not necessary because the data do not indicate decreased placental perfusion. It is not necessary to administer oxygen because the data do not indicate fetal compromise. Naloxone, an opioid antagonist, may need to be administered to the newborn, but the present data do not indicate that this is necessary.

A 2-year-old child is admitted with gastroenteritis and dehydration. Peripheral intravenous fluids are prescribed. What is the most appropriate site for the first intravenous insertion? Scalp vein near the fontanel Venous arch on top of the foot Dorsal metacarpals of the hand Basilic vein at the antecubital fossa

Dorsal metacarpals of the hand The choice of first insertion site should be distal (low) on the periphery of an extremity and progress proximally (upward) toward the trunk; the upper extremities are the most appropriate sites for intravenous insertions for adults and children older than 1 year. Scalp veins are used for infants only if peripheral veins are inaccessible. Foot veins should not be used once a child is walking. The antecubital fossa should be avoided because the arm will have to be immobilized to stabilize the intravenous insertion site to prevent an infiltration.

A client is admitted to the emergency department with profuse vomiting. The client reports that the vomitus was bright red in color. What does the nurse identify as the priority intervention? Begin gastric lavage Obtain stool for occult blood Ascertain the client's eating habits Draw blood for typing and crossmatching

Draw blood for typing and crossmatching Immediate blood replacement is indicated. A type and crossmatch will ensure that the correct blood type will be administered to the client, preventing a transfusion reaction. Beginning gastric lavage is not the priority, although it may be done later. Obtaining a stool for an occult blood test is not the priority, although it may be done later. Ascertaining the client's eating habits is not the priority, although it may be done later when completing an admission history and physical.

What are the pharmacokinetic reasons for drug sensitivity in infants? Small body size Drug absorption Renal drug excretion Protein binding of drugs Hepatic drug metabolism

Drug absorption Renal drug excretion Protein binding of drugs Hepatic drug metabolism Increased drug sensitivity in infants is a result of the immature state of pharmacokinetic processes such as drug absorption, renal drug excretion, protein binding of drugs, and hepatic drug metabolism. A small body is not a pharmacokinetic parameter.

A female client whose ECG exhibits multiple premature ventricular complexes is prescribed oral disopyramide. Which side effects should the nurse include when teaching the client about this drug? . Dry mouth Rhinorrhea Constipation Hyperglycemia Stress incontinence

Dry mouth Constipation Dry mouth occurs because of its anticholinergic properties. Constipation is a side effect of this nonnitrate antidysrhythmic because of its anticholinergic properties. A thin, watery discharge from nose (rhinorrhea) does not occur with this medication because of its anticholinergic properties. Hypoglycemia, not hyperglycemia, may occur. Urinary hesitancy and retention, rather than stress incontinence, occur.

Which medications are used to treat generalized anxiety disorder (GAD)? Duloxetine Venlafaxine Clonazepam Escitalopram Clomipramine

Duloxetine Venlafaxine Escitalopram Duloxetine, venlafaxine, and escitalopram are antidepressants approved for the treatment of generalized anxiety disorder (GAD). Clonazepam and clomipramine are used to treat panic disorders.

Which drug may cause malformations of the male external genitalia? Etretinate Dutasteride Methimazole Propylthiouracil

Dutasteride Dutasteride is a 5-alpha-reductase inhibitor that causes malformations of external genitalia in males. Etretinate is a vitamin A derivative that may cause multiple central nervous system defects. Methimazole and propylthiouracil are antithyroid drugs that may cause goiters and hypothyroidism.

A client is undergoing diagnostic testing to determine if the client has myasthenia gravis. The nurse understands that the test that is most specific for determining the presence of this disease is what? Electromyography Pyridostigmine test History of physical deterioration Edrophonium chloride test

Edrophonium chloride test Edrophonium chloride test uses a drug that is a cholinergic and an anticholinesterase; it blocks the action of cholinesterase at the myoneural junction and inhibits the destruction of acetylcholine. Its action of increasing muscle strength is immediate for a short time. The results of an electromyography will be added to the database, but they are nonspecific. Pyridostigmine is a slower-acting anticholinesterase drug that is prescribed commonly to treat myasthenia gravis; edrophonium chloride is used instead of pyridostigmine to diagnose myasthenia gravis because, when injected intravenously, it immediately increases muscle strength for a short time. The results of a history and physical are added to the database, but the data collected are not as definitive as another specific test for the diagnosis of myasthenia gravis.

A nurse is caring for a client who is receiving serum albumin. What indicates that the albumin is effective? Improved clotting of blood Formation of red blood cells Activation of white blood cells (WBCs) Effective cardiac output

Effective cardiac output Serum albumin, a protein, establishes the plasma colloid osmotic (oncotic) pressure because of its high molecular weight and size. Indicators of adequate osmotic pressure include an effective cardiac output. Blood clotting involves blood protein fractions other than albumin; for example, prothrombin and fibrinogen are within the alpha- and beta-globulin fractions. Red blood cell formation (erythropoiesis) occurs in red marrow and can be related to albumin only indirectly; albumin is the blood transport protein for thyroxine, which stimulates metabolism in all cells, including those in red bone marrow. Albumin does not activate WBCs; WBCs are activated by antigens and substances released from damaged or diseased cells.

A lactating mother is administered oxytocin. What could be the function of oxytocin in the client? Ejection of milk Induction of labor Induction of abortion Control of uterine bleeding

Ejection of milk Oxytocin helps in ejection of milk in lactating mothers. Induction of labor and abortion are the functions performed by oxytocin, but not in a lactating mother. Oxytocin controls uterine bleeding after the delivery.

A client is admitted to the hospital with a diagnosis of acute salmonellosis. What does the nurse expect the health care provider to prescribe? Cathartics Electrolytes Antidiarrheals Antispasmodics

Electrolytes Fluids of dextrose and normal saline and electrolytes are administered to prevent profound dehydration caused by excessive loss of water and electrolytes through diarrheal output. Cathartics are not necessary; salmonellosis is a condition that increases intestinal peristalsis, causing diarrhea. Antidiarrheals are not used when a bacterial infection is diagnosed because slowed peristalsis impedes the excretion of the microorganism. Antispasmodics are not used when bacterial infection is diagnosed because slowed peristalsis decreases excretion of the microorganism.

A client is to receive metoclopramide intravenously 30 minutes before initiating chemotherapy for cancer of the colon. The nurse explains that metoclopramide is given for what purpose? Stimulate production of gastrointestinal secretions Enhance relaxation of the upper gastrointestinal tract Prolong excretion of the chemotherapeutic medication Increase absorption of the chemotherapeutic medication

Enhance relaxation of the upper gastrointestinal tract The relaxation effect increases the passage of food through the gastrointestinal tract, limiting reverse peristalsis, gastroesophageal reflux, and vomiting, all of which are precipitated by chemotherapeutic agents. Metoclopramide does not stimulate the production of gastrointestinal secretions. Metoclopramide has no effect on the excretion of chemotherapeutic medications. Metoclopramide has no effect on the absorption of chemotherapeutic medications.

A client at 30 weeks' gestation is admitted in preterm labor. An intravenous solution of the tocolytic agent ritodrine is started. The client asks why the drug is being administered. How will the nurse respond? Enhances uterine relaxation Prevents fetal hypoglycemia Stimulates fetal lung maturity Counteracts adverse reactions of other drugs

Enhances uterine relaxation Ritodrine is given to promote uterine relaxation. Although ritodrine may cause fetal hyperglycemia and neonatal hypoglycemia, it is not given for this purpose. Glucocorticoids such as betamethasone are given to stimulate fetal lung maturity. Ritodrine does not have a role in counteracting adverse reactions.

A client has colorectal cancer and is receiving cetuximab. Which process does cetuximab inhibit? Proteasome activity BCR-ABL tyrosine kinase (TK) Anaplastic lymphoma kinase Epidermal growth factor receptors (EGFRs)

Epidermal growth factor receptors (EGFRs) Cetuximab is an EGFR-tyrosine TK inhibitor that acts by inhibiting EGFRs in clients with colorectal cancer. Bortezomib inhibits proteasome activity in clients with multiple myeloma. Dasatinib acts by inhibiting BCR-ABL TK in clients with chronic myeloid leukemia. Crizotinib acts by inhibiting anaplastic lymphoma kinase (ALK) in clients with locally advanced or metastatic non-small cell lung cancer that is ALK positive.

A nurse is caring for an obese client in early labor. The anesthesiologist discusses several types of analgesia and anesthesia with the client and recommends one. The client requests clarification before signing the consent form. Which type has the anesthesiologist recommended? Epidural anesthesia Oral opioid analgesia Pudendal nerve anesthesia Intravenous opioid analgesia

Epidural anesthesia Epidural anesthesia during the first stage of labor decreases metabolic and respiratory demands and is preferred for obese clients. Obese women are sensitive to systemic opioids, which predispose them to respiratory depression; oral medications do not have a uniform rate of absorption and are not recommended during labor. A pudendal block does not reach the uterus, so contractions are felt; it is used during the second stage of labor.

Steroid therapy is prescribed for a client with common signs and symptoms of multiple sclerosis. In response to the steroid therapy, what symptom does the nurse expect to decrease? Emotional lability Muscular contractions Pain in the extremities Episodes of vision loss

Episodes of vision loss Steroids decrease the inflammatory process around the optic nerve, thus improving vision; visual impairment is the most common physiological manifestation of multiple sclerosis. Steroids are associated with increased emotional lability. Steroids are not effective in easing muscle contractions. Pain in the extremities is not common unless spasms are present; steroids do not relieve spasms.

Which statement regarding transdermal estradiol is true? Estradiol reduces estrogen levels. Estradiol is slowly absorbed from skin. Estradiol patches are applied monthly. Estradiol should be used along with sunscreen products.

Estradiol is slowly absorbed from skin. Estradiol is slowly absorbed from the skin for up to 8 hours after application. Estradiol is administered to increase the levels of estrogen in postmenopausal women to ease hot flashes. Estradiol is available as an emulsion that is typically applied to the thighs or calves. Estradiol transdermal films are changed once or twice a week, while gel is applied daily. Sunscreen products should not be applied with estradiol because sunscreen may reduce the absorption of estradiol.

Which drugs used for the treatment of plaque psoriasis will the nurse administer subcutaneously? . Alefacept Infliximab Etanercept Adalimumab Ustekinumab

Etanercept Adalimumab Ustekinumab Etanercept, adalimumab, and ustekinumab are administered subcutaneously. Alefacept is given via the intramuscular route. Infliximab is administered via the intravenous route.

Which antitubercular medications may increase a client's risk for gout? Rifampin Isoniazid Bedaquiline Ethambutol Pyrazinamide

Ethambutol Pyrazinamide Pyrazinamide and ethambutol increase uric acid formation and may increase the risk of gout. Rifampin and isoniazid may lead to liver toxicity. Bedaquiline does not increase the risk of gout; it is used for multidrug-resistant tuberculosis.

A client is admitted and diagnosed with myasthenia gravis. Pyridostigmine bromide therapy via tablets has been prescribed. The nurse anticipates that the dosage will be changed frequently during the first week of therapy. While the dosage is being adjusted, what action does the nurse perform? Administer the medication after meals. Administer the medication on an empty stomach. Evaluate the client's psychological responses between medication doses. Evaluate the client's muscle strength every hour after the medication is given.

Evaluate the client's muscle strength every hour after the medication is given. The onset of action of pyridostigmine is 30 to 45 minutes after administration, and the effects last up to 6 hours; the client's response will influence dosage levels. Pyridostigmine usually is administered before meals to promote mastication. Pyridostigmine should be administered with food to prevent gastric irritation. There are no psychological side effects associated with pyridostigmine.

A client has surgery for the insertion of an implanted infusion port for chemotherapy. The client asks, "The doctor said after my chemotherapy is finished, the port will stay in, but it needs to be flushed routinely. How often does this have to be done?" What should the nurse tell the client about how often the port will most likely need to be flushed when not in use? Every day Once a week Every month Twice a year

Every month Once-a-month flushes usually are adequate to keep an implanted infusion port from clotting. Every day or once a week is unnecessary. Twice a year may jeopardize the viability of the port.

In a clinical study, subjects were given chlorhexidine and betadine as antiseptics. How will a nurse researcher categorize this research? Evaluation research Descriptive research Correlational research Experimental research

Experimental research The nurse will categorize this study as experimental research. In experimental research, the investigator gives variables randomly to the subjects. In this case, subjects are given chlorhexidine and betadine to test their efficacy in reducing infection. Evaluation research is an initial study that refines a hypothesis, such as testing a new exercise in older dementia clients. In a descriptive study, the characteristics of a person or a situation are measured. For example, a researcher may examine the nurses' bias while caring for obese clients. Correlational research is used to find out the relationship between different variables without the interference of a researcher. An example would be determining the educational status of nurses and their satisfaction with the job provided.

A monoamine oxidase inhibitor (MAOI) is prescribed, and the nurse is formulating a teaching plan. What will the nurse instruct the client to avoid while taking this drug? Fermented foods Prolonged sun exposure Strenuous physical exercise Over-the-counter antihistamine drugs

Fermented foods An MAOI can cause hypertensive crisis if food or beverages that are high in tyramine, such as fermented foods, are ingested. Prolonged exposure to the sun is hazardous for clients taking one of the phenothiazines. Strenuous physical exercise is not contraindicated. Antihistamines are not prohibited with MAOI medications.

A nurse is reviewing the admission laboratory report of an infant with severe gastroenteritis. The serum potassium is 3 mEq/L. Potassium chloride 20 mEq/L is prescribed to be added to the infant's intravenous (IV) line. What should the nurse do next? Find out when the infant last had a wet diaper. Question the prescription and withhold the medication. Ask the mother whether the infant is allergic to potassium. Administer the potassium and then monitor the infant's response.

Find out when the infant last had a wet diaper. Potassium chloride is excreted by functioning kidneys; if there is anuria, which is a sign of kidney failure, the potassium should be withheld and the practitioner notified. There is no reason to question the prescription because the laboratory value is below the expected level for an infant, which is 4.1 to 5.3 mEq/L. Potassium is a component of body fluid and will not cause an allergic response. Administering the potassium without confirming adequate kidney function is unsafe because potassium can accumulate and cause lethal cardiac dysrhythmias.

Lithium carbonate, 600 mg by mouth three times a day, is prescribed for a client. The nurse concludes that the teaching about its side effects is understood when the client says that the primary healthcare provider will be notified immediately if what condition occurs? Difficulty urinating Sensitivity to bright light or sun Fine hand tremor or slurred speech Sexual dysfunction or breast enlargement

Fine hand tremor or slurred speech Fine hand tremor or slurred speech in a person taking lithium may signal the development of toxicity; signs of toxicity include marked tremors, lack of coordination, sluggishness, and confusion. Lithium carbonate can cause polyuria and incontinence, not urine retention. Sensitivity to bright light or sun is a side effect of the phenothiazine group of medications. Neither sexual dysfunction nor breast enlargement is associated with lithium carbonate intake.

A client reports fever, cough, muscle aches, night sweats, and chest pain. The laboratory reports of the client indicate the presence of Coccidioides organisms in the respiratory tract. Which drug is beneficial for the client? Oseltamivir Fluconazole Pyrazinamide Cephalosporin

Fluconazole Coccidioides organisms cause coccidioidomycosis. The symptoms of coccidioidomycosis are fever, cough, muscle aches, night sweats, and chest pain. Fluconazole is an antifungal drug beneficial in treating coccidioidomycosis. Oseltamivir is an antiviral drug used to treat influenza. Pyrazinamide is an antitubercular drug, used to treat tuberculosis. Cephalosporin is an antibiotic, and may be used in treatment of bacterial pharyngitis.

What medication does the nurse expect to administer to actively reverse the overdose sedative effects of benzodiazepines? Lithium Flumazenil Methadone Chlorpromazine

Flumazenil Flumazenil is the drug of choice in the management of overdose when a benzodiazepine is the only agent ingested by a client not at risk for seizure activity. Flumazenil medication competitively inhibits activity at benzodiazepine recognition sites on gamma-aminobutyric acid-benzodiazepine receptor complexes. Lithium is used in the treatment of mood disorders. Methadone is used for narcotic addiction withdrawal. Chlorpromazine is contraindicated in the presence of central nervous system depressants.

A client with a diagnosis of schizophrenia is discharged from the hospital. At home the client forgets to take the medication, is unable to function, and must be rehospitalized. What medication may be prescribed that can be administered on an outpatient basis every 2 to 3 weeks? Lithium Diazepam Fluvoxamine Fluphenazine

Fluphenazine Fluphenazine can be given intramuscularly every 2 to 3 weeks to clients who are unreliable about taking oral medications; it allows them to live in the community while keeping the disorder under control. Lithium is a mood-stabilizing medication that is given to clients with bipolar disorder. This drug is not given for schizophrenia. Diazepam is an antianxiety/anticonvulsant/skeletal muscle relaxant that is not given for schizophrenia. Fluvoxamine is a selective serotonin reuptake inhibitor; it is administered for depression, not schizophrenia.

A nurse is caring for a client who uses ritualistic behavior. What common antiobsessional medication does the nurse anticipate will be prescribed? Benztropine Amantadine Fluvoxamine Diphenhydramine

Fluvoxamine Fluvoxamine blocks the uptake of serotonin, which leads to a decrease in obsessive-compulsive behaviors. Benztropine is an antiparkinsonian agent, not an antianxiety agent. Amantadine is an antiparkinsonian agent, not an antianxiety agent. Diphenhydramine is an antihistamine, not an antianxiety agent.

A client is admitted to the intensive care unit with acute pulmonary edema. Which diuretic does the nurse anticipate will be prescribed? Furosemide Chlorothiazide Spironolactone Acetazolamide

Furosemide Furosemide acts on the loop of Henle by increasing the excretion of chloride and sodium, is available for intravenous administration, and is more effective than chlorothiazide, spironolactone, and acetazolamide. Although used in the treatment of edema and hypertension, chlorothiazide is not as efficacious as furosemide. Spironolactone is a potassium-sparing diuretic; it is less efficacious than thiazide diuretics. Acetazolamide is used in the treatment of glaucoma to lower intraocular pressure.

A nurse is caring for an 8-year-old child with acute poststreptococcal glomerulonephritis (APSGN). What medications does the nurse expect the practitioner to prescribe? Penicillin Morphine Furosemide Labetalol Phenobarbital

Furosemide Labetalol The child with APSGN is oliguric; diuretics are used to increase urine output. The child with APSGN is hypertensive; antihypertensives are used to reduce the blood pressure. Penicillin is administered if there is evidence of streptococcal infection; however, the strep infection is usually not active when APSGN develops. Children with APSGN do not experience pain; therefore morphine is not needed. If the hypertension is controlled, seizures are not expected, and phenobarbital is not necessary.

After the nurse provides education about hydrochlorothiazide, the client will agree to notify the healthcare provider regarding the development of which symptom? Insomnia Nasal congestion Increased thirst Generalized weakness

Generalized weakness Generalized weakness is a symptom of significant hypokalemia, which may be a sequela of diuretic therapy. Insomnia is not known to be related to hypokalemia or hydrochlorothiazide therapy. Although a stuffy nose is unrelated to hydrochlorothiazide therapy, it can occur with other antihypertensive drugs. Increased thirst is associated with hypernatremia. Because this drug increases excretion of water and sodium in addition to potassium and chloride, hyponatremia, not hypernatremia, may occur.

A nurse is planning to screen a school-aged child for impaired hearing because the child is receiving an antibiotic that affects hearing. Which medication does the nurse suspect may have caused hearing impairment? Amoxicillin Ciprofloxacin Clindamycin Gentamicin

Gentamicin Gentamicin can be ototoxic because of its effects on the eighth cranial nerve. Reactions to amoxicillin are usually allergic in nature. Impaired hearing does not occur with ciprofloxacin or with clindamycin.

An alcoholic mother gave birth to a baby who had alcohol dependency. Which nursing intervention would be helpful in managing alcohol dependency in the baby? Avoid alcohol in the treatment Give treatment to reduce alcohol abuse Give reduced doses of alcohol to overcome drug dependency Administer alcohol to the baby to prevent withdrawal symptoms

Give reduced doses of alcohol to overcome drug dependency The baby born to an alcoholic mother should be treated with reduced doses of alcohol to wean the baby from the alcohol dependency. Avoiding alcohol may cause withdrawal syndrome, and the baby may show symptoms of shrill crying and extreme irritability. Treatment to counter the alcohol abuse in the baby may not be effective. Alcohol should not be provided in normal doses to reduce the alcohol dependency or to prevent withdrawal symptoms.

A client is started on tetracycline antibiotic therapy. What should the nurse do when administering this drug? Administer the medication with meals or a snack. Provide orange or other citrus fruit juice with the medication. Give the medication an hour before milk products are ingested. Offer antacids 30 minutes after administration if gastrointestinal side effects occur.

Give the medication an hour before milk products are ingested. Any product containing aluminum, magnesium, or calcium ions should not be taken in the hour before or after an oral dose of tetracyclines (with the exception of doxycycline) because it decreases absorption by as much as 25% to 50%. Food interferes with absorption; it should be given one hour before or two hours after meals. Citrus juice has no influence on this drug. Antacids will interfere with absorption.

The primary healthcare provider informs the registered nurse that the client must be monitored on a regular basis because he or she is prescribed haloperidol. Which client conditions would warrant these instructions? Glaucoma Comatose Adynamic ileus Parkinson disease Prostatic hypertrophy

Glaucoma Adynamic ileus Prostatic hypertrophy Haloperidol is a first-generation antipsychotic drug. Clients with glaucoma should use the drug with caution. Adynamic ileus may cause paralysis to the intestinal motility; the drug should be cautiously used in the client. Clients with prostatic hypertrophy should be given haloperidol with caution because prostatic hyperplasia is a side effect of haloperidol. Haloperidol is contraindicated in comatose clients and clients with Parkinson disease.

Which hormone levels do not need to be monitored during the administration of menotropins for infertility? Estrogen Luteinizing hormone Follicle-stimulating hormone Gonadotropin-releasing hormone

Gonadotropin-releasing hormone that are administered for infertility act at the pituitary level. Gonadotropin-releasing hormones act at the hypothalamic level; therefore, this hormone does not need to be monitored. Menotropin is a standardized mixture of follicle-stimulating hormone and luteinizing hormone; these hormones should be monitored. Also, estrogen levels need to be monitored because the luteinizing hormone indirectly stimulates the production of estrogen.

A healthcare provider prescribes supplemental oral iron therapy for a child with iron-deficiency anemia. What side effect will the nurse tell the parents to anticipate? Bloody stool Orange urine Greenish-black stool Staining of the mouth

Greenish-black stool Iron is excreted in the feces, and the change in color results from the insoluble iron compound excreted in the stool. Blood in the stool is associated with lower intestinal bleeding, not supplemental iron ingestion. Orange urine is not associated with supplemental iron ingestion; it occurs with phenazopyridine hydrochloride or rifampin administration. Staining of the mucous membranes of the mouth should not occur with oral administration of iron if a straw is used and the teeth are brushed immediately after administration. The teeth, not the mucous membranes, may become stained if these precautions are not taken.

Which is the adverse effect of haloperidol? Ataxia Asthenia Insomnia Gynecomastia

Gynecomastia Haloperidol is an antipsychotic drug used in the long-term treatment of psychosis. Gynecomastia is one of the adverse effects of this drug. Ataxia, asthenia, and insomnia are the adverse effects of clozapine.

Which drug may cause fetal facial malformations when taken by a pregnant client? Antimicrobials Antiseizure drugs HMG-CoA reductase inhibitors Nonsteroidal antiinflammatory drugs

HMG-CoA reductase inhibitors HMG-CoA reductase inhibitors may cause fetal facial malformations and central nervous system anomalies, including holoprosencephaly and neural tube defects. Antimicrobials may cause bone defects, abnormally small or absent eyes, heart defects, and cleft lips with cleft palates. Antiseizure drugs have teratogenic effects on the central nervous system. Nonsteroidal antiinflammatory drugs cause premature closure of the ductus arteriosus.

Which conditions may result in the decreased effectiveness of estrogen therapy? Habit of smoking Use of anticoagulants Use of tricyclic antidepressants Presence of endometrial cancer Presence of thromboembolic disorders

Habit of smoking Use of anticoagulants The effectiveness of estrogen therapy decreases with smoking and decreases with the use of anticoagulants. The use of tricyclic antidepressants along with estrogen may result in drug toxicity. Estrogen therapy is not recommended in clients with endometrial cancer and thromboembolic disorders because it may increase the risk of these complications.

Which drugs may lead to a prolongation of the QT interval in a client who is on drug therapy for schizophrenia? Loxapine Haloperidol Thiothixene Thioridazine Chlorpromazine

Haloperidol Thioridazine Chlorpromazine Prolongation of the QT interval indicates severe dysrhythmias. This is due to the use of haloperidol, thioridazine, and chlorpromazine, which are first generation antipsychotics. Loxapine and thiothixene do not cause prolongation of the QT interval.

The nurse is caring for a client with type 1 diabetes. For which signs or symptoms of insulin-induced hypoglycemia should the nurse particularly be observant? Excessive hunger Headache Diaphoresis Excessive thirst Deep respirations

Headache Diaphoresis Hypoglycemia affects the central nervous system, causing headache. Hypoglycemia affects the sympathetic nervous system, causing diaphoresis. Excessive hunger is associated with hyperglycemia because glucose is not being used for cellular metabolism. Excessive thirst is associated with hyperglycemia because fluid shifts, along with the excess glucose being excreted by the kidneys, result in polyuria. Deep respirations (Kussmaul respirations) are associated with hyperglycemia because the body is attempting to blow off carbon dioxide to compensate for the metabolic acidosis.

A client is prescribed epoetin injections. To ensure the client's safety, which lab value should the nurse assess before administration? Hemoglobin Platelet count Prothrombin time Partial thromboplastin time

Hemoglobin Epoetin is used to treat anemia by increasing production of red blood cells. The lab value the nurse should assess before administration is the hemoglobin because it measures the number of red blood cells. Erythropoietin is specific for increasing red blood cells and does not have an effect on other blood components such as white blood cells or thrombocytes (platelets). The partial thromboplastin time and prothrombin time are measures of the effectiveness of anticoagulant therapy.

A nurse is providing instructions for a client who is receiving phenytoin but has limited access to health care. What side effect is the basis for the nurse's emphasis on meticulous oral hygiene? Hyperplasia of the gums Alkalinity of the oral secretions Irritation of the gingiva and destruction of tooth enamel Promotion of plaque and bacterial growth at the gum line

Hyperplasia of the gums Gingival hyperplasia is an adverse effect of long-term phenytoin therapy; incidence can be decreased by maintaining therapeutic blood levels and meticulous oral hygiene. Alkalinity is not related to phenytoin or to gingival hyperplasia caused by phenytoin. Irritation of the gingiva and destruction of tooth enamel are not direct effects of phenytoin. Plaque and bacterial growth at the gum line are unrelated to phenytoin or to hyperplasia caused by it.

Neuroleptic malignant syndrome develops in a client who is taking a conventional antipsychotic medication. What signs and symptoms does the nurse expect? Hyperpyrexia Blurred vision Increased muscle tone Respiratory depression Buccolingual lip-smacking

Hyperpyrexia Increased muscle tone Respiratory depression Neuroleptic malignant syndrome is caused by dopamine blockade in the hypothalamus, precipitated by antipsychotic medications. The hypothalamus activates, controls, and integrates many of the involuntary functions necessary for living. Signs and symptoms of a problem in this area include increased body temperature (hyperpyrexia), increased muscle tone, and respiratory depression. Blurred vision is a side effect of anticholinergics. Lip-smacking is associated with tardive dyskinesia.

The nurse in a women's health clinic is counseling a 34-year-old client who has requested a prescription for oral contraceptives. What condition would warrant additional discussion? Anemia Depression Hypertension Dysmenorrhea

Hypertension One of the side effects of oral contraceptives is hypertension; therefore they are contraindicated for any woman who already has hypertension, particularly at the client's age or older. Anemia is not a contraindication for women who want to take oral contraceptives because oral contraceptives may help this condition by decreasing bleeding. Depression is not a contraindication for women who want to take oral contraceptives. Oral contraceptives may be prescribed for women with menstrual difficulties such as dysmenorrhea.

A client with human immunodeficiency virus (HIV)-associated Pneumocystis jiroveci pneumonia is to receive pentamidine isethionate intravenously (IV) once daily. The nurse should monitor the client for what adverse effect? Hypertension Hypokalemia Hypoglycemia Hypercalcemia

Hypoglycemia Pentamidine isethionate can cause either hypoglycemia or hyperglycemia even after therapy is discontinued, and therefore blood glucose levels should be monitored. Hypotension, not hypertension, occurs with pentamidine isethionate. Hyperkalemia, not hypokalemia, occurs with pentamidine isethionate. Hypocalcemia, not hypercalcemia, occurs with pentamidine isethionate.

A 6-year-old child with Reye syndrome is receiving an intravenous solution of 10% glucose and mannitol to reduce cerebral edema. For which complication of this therapy does the nurse monitor the child? Overhydration Seizure activity Acute heart failure Hypovolemic shock

Hypovolemic shock Both hypertonic glucose and mannitol cause diuresis; the child should be monitored for excessive fluid loss. Hypertonic glucose and mannitol will cause fluid loss, not gain. Seizure activity is not anticipated as a result of this infusion. An increased fluid volume can lead to heart failure; however, hypertonic glucose and mannitol cause fluid loss, not gain.

A client with type 1 diabetes tells the nurse, "I take guaifenesin cough syrup when I have a cold." What important instruction does the nurse include in client teaching about this medication? Substitute an elixir for the cough syrup. Increase fluid intake and use a humidifier to control the cough. The small amounts of sugar in medications are not a concern with diabetes. Include the glucose in the cough syrup when calculating daily carbohydrate allowance.

Include the glucose in the cough syrup when calculating daily carbohydrate allowance. Cough syrup contains a glucose base; the client should use a glucose-free product or account for the glucose. Elixirs contain natural sweeteners. Although increasing fluid intake and using a room humidifier will also loosen secretions, this does not include important information about the medication. Additional sweeteners, even the amounts in medications, may increase serum glucose levels beyond the desired range; once control is achieved, it is unwise to alter dietary intake or medications without supervision.

A client is receiving dexamethasone to treat acute exacerbation of asthma. For what side effect should the nurse monitor the client? Hyperkalemia Liver dysfunction Orthostatic hypotension Increased blood glucose

Increased blood glucose Dexamethasone increases gluconeogenesis, which may cause hyperglycemia. Hypokalemia, not hyperkalemia, is a side effect. Liver dysfunction is not a side effect. Hypertension, not hypotension, is a side effect.

When obtaining a health history, the nurse is informed that a client has been taking digoxin. What therapeutic effect of digoxin does the nurse expect? Decreased cardiac output Decreased stroke volume of the heart Increased contractile force of the myocardium Increased electrical conduction through the atrioventricular (AV) node

Increased contractile force of the myocardium Digoxin produces a positive inotropic effect that increases the strength of myocardial contractions and thus cardiac output. The positive inotropic effect of digoxin increases, not decreases, cardiac output. Digoxin increases the strength of myocardial contractions (positive inotropic effect) and slows the heart rate (negative chronotropic effect); these effects increase the stroke volume of the heart. Digoxin decreases the refractory period of the AV node and decreases conduction through the sinoatrial (SA) and AV nodes.

The nurse is caring for an older client who is scheduled for a bronchoscopy. Midazolam has been prescribed for the procedure. What administration guidelines will the nurse follow? Increments should be smaller and rate of injection slower. The medication should be given as a rapid intravenous push. It is important to monitor for spikes in blood pressure elevation during administration. During the procedure, the medication should be given as needed for pain management.

Increments should be smaller and rate of injection slower. In an older client, peak effect may be delayed; increments should be smaller and rate of injection slower. When used for sedation/anxiolysis/amnesia for a procedure, the dosage must be individualized and titrated. Midazolam should always be titrated slowly; administer over at least 2 minutes and allow an additional 2 or more minutes to fully evaluate the sedative effect. Titration to effect with multiple small doses is essential for safe administration. Central nervous system depression is the most serious side effect. A sudden rise in blood pressure shortly after administration has not been evidenced. Midazolam is given for sedation/anxiolysis/amnesia for a procedure.

Which drugs are used for the treatment of clients with rheumatoid arthritis that inhibit tumor necrosis factor-A? Anakinra Infliximab Abatacept Etanercept Golimumab

Infliximab Etanercept Golimumab Biological response modifiers (BRMs) are the substances that modify immune responses by either enhancing an immune response or suppressing it. Infliximab, etanercept, and golimumab are BRMs used in the treatment of rheumatoid arthritis that inhibit tumor necrosis factor (TNF)-A. Anakinra is aninterleukin-1 receptor antagonist used in the treatment of rheumatoid arthritis. Abatacept is a selective T-lymphocyteco-stimulator modulator (T-cell inhibitor) used in the treatment of rheumatoid arthritis.

A 4-year-old child is admitted to the pediatric unit with the diagnosis of acute lymphocytic leukemia (ALL). A blood transfusion is ordered, and an intravenous line is started. What will the nurse do in regard to administering the transfusion? Infuse the blood over no more than 4 hours. Take the vital signs 3 hours after the transfusion. Check the vital signs 15 minutes after starting the transfusion. Have the blood warm at room temperature for 1 hour before administration.

Infuse the blood over no more than 4 hours. Blood should be administered within 4 hours; the risk for bacterial proliferation increases over time and exposure to room temperature. Taking the vital signs 3 hours after the transfusion is too long to wait; the vital signs should be checked every 5 minutes during the absorption of the first 50 mL of blood and then routinely thereafter (every 15 minutes to 1 hour, depending on hospital policy). Vital signs must be checked every 5 minutes during the administration of the first 50 mL of blood to detect a transfusion reaction. Blood should be used within 30 minutes after its arrival from the blood bank; the risk for bacterial proliferation increases over time and exposure to room temperature.

A nurse assesses a primigravida who has been in labor for 5 hours. The fetal heart rate tracing is reassuring. Contractions, which are of mild intensity, are lasting 30 seconds and are 3 to 5 minutes apart. An oxytocin infusion is prescribed. What is the priority nursing intervention at this time? Checking cervical dilation every hour Keeping the labor environment dark and quiet Infusing oxytocin by piggybacking into the primary line Positioning the client on the left side throughout the infusion

Infusing oxytocin by piggybacking into the primary line Piggybacking the oxytocin infusion allows it to be discontinued, if necessary, while permitting the vein to remain open by way of the primary intravenous line. Cervical dilation is checked when there is believed to be a change, not on a regular basis. Unless specifically requested by the client, there is no reason to maintain a dark, quiet labor environment. Although positioning the client on her left side is recommended, it is not the primary concern at this time; there are no data to indicate maternal hypotension.

A client diagnosed with adrenal gland hypofunction is receiving fludrocortisone therapy. Which nursing intervention would help the client reduce the risk of complications? Instruct the client to report severe diarrhea Instruct the client to report fever and vomiting Instruct the client to take the drug with meals or snacks Instruct the client to regularly monitor blood pressure

Instruct the client to regularly monitor blood pressure Fludrocortisone, prednisone, and cortisone are the drugs prescribed for the treatment of adrenal gland hypofunction. During fludrocortisone therapy, the blood pressure of the client should be regularly monitored because fludrocortisone has a potential to cause hypertension. Reporting of severe diarrhea, fever, and vomiting is required during the administration of prednisone. During the administration of cortisone, the client should take the drug with meals to reduce the risk of gastrointestinal irritation.

A nurse plans to teach a client with type 1 diabetes about the use of an insulin pump. What information will the nurse include in client teaching? Insulin pumps mimic the way a healthy pancreas works. The insulin pump's needle should be changed every day. Pumps are implanted in a subcutaneous pocket near the abdomen. The insulin pump's advantage is that it only requires glucose monitoring once a day.

Insulin pumps mimic the way a healthy pancreas works. The basal infusion rate mimics the low rate of insulin secretion during fasting, and the bolus before meals mimics the high output after meals. The subcutaneous needle may be left in place for as long as 3 days. Most insulin pumps are external to the body and access the body via a subcutaneous needle. Blood glucose monitoring is done a minimum of 4 times a day.

Which medication class helps to prevent human immunodeficiency virus (HIV) incorporating its genetic material into the client's cell? Entry inhibitors Protease inhibitors Integrase inhibitors Reverse transcriptase inhibitors

Integrase inhibitors Integrase inhibitors such as raltegravir and dolutegravir bind with integrase enzymes and prevent HIV from incorporating its genetic material into the host (client's) cell. Entry inhibitors prevent the binding of HIV. Protease inhibitors prevent the protease enzyme from cutting HIV proteins into the proper lengths needed to allow viable virions to assemble. Reverse transcriptase inhibitors inhibit the action of reverse transcriptase.

A nurse changing the dressing on the client's perineum would fall into which zone? Public zone Intimate zone Personal zone Vulnerable zone

Intimate zone Changing a client's dressing on the perineum falls under the intimate zone. For this action, the appropriate interpersonal distance between the nurse and the client should be between 0 and 18 inches. A nurse lecturing a class of students or speaking at a community forum lies within a public zone. A personal zone refers to a nurse sitting on the client's bedside, taking a client's history, or teaching a client individually. The vulnerable zone is where special care is needed.

A client with a history of alcoholism is found to have Wernicke encephalopathy associated with Korsakoff syndrome. What does the nurse anticipate will be prescribed? Traditional phenothiazine Judicious use of antipsychotics Intramuscular injections of thiamine Oral administration of chlorpromazine

Intramuscular injections of thiamine Thiamine is a coenzyme necessary for the production of energy from glucose. If thiamine is not present in adequate amounts, nerve activity is diminished and damage or degeneration of myelin sheaths occurs. A traditional phenothiazine is a neuroleptic antipsychotic that should not be prescribed because it is hepatotoxic. Antipsychotics are avoided; the use of these has a higher risk for toxic side effects in older or debilitated persons. Chlorpromazine, a neuroleptic, will not be used because it is severely toxic to the liver.

By what route does the nurse administer the chelating agent calcium disodium edetate (EDTA) to a toddler? Transdermally Orally with milk Z-track injection Intravenous infusion

Intravenous infusion Calcium EDTA is administered parenterally intramuscularly, intravenously, and subcutaneously; however, the intravenous route permits the most efficient absorption. Chelating agents are not absorbed through the skin. The oral route is contraindicated for calcium EDTA because it results in unabsorbable iron complexes within the lumen of the gastrointestinal tract. The Z-track technique is not necessary if the intramuscular route is ordered by the provider.

Which drug is used to treat acne vulgaris in adolescents but is contraindicated in pregnancy due to its teratogenic effects? Tretinoin Adapalene Isotretinoin Benzoyl peroxide

Isotretinoin Isotretinoin passes through the placental barrier and exhibits teratogenic effects, so it is contraindicated in pregnancy. Tretinoin is not harmful when used topically. Adapalene and benzoyl peroxide are safe drugs for topical use during pregnancy.

A pregnant client with an infection tells the nurse that she has taken tetracycline for infections on other occasions and prefers to take it now. What does the nurse advise regarding the reason tetracycline is avoided during pregnancy? It affects breastfeeding adversely. It alters the development of fetal teeth buds. Tetracycline causes fetal allergies. It increases fetal tolerance to the medication.

It alters the development of fetal teeth buds. Tetracycline has an affinity for calcium; if used during tooth bud development it may cause discoloration of teeth. Tetracycline does not adversely affect breast-feeding, cause fetal allergies to the medication, or increase fetal tolerance of the medication.

A client with psychosis is receiving olanzapine. What special information about this drug does the nurse recall? It must be administered by Z-track administration. A special tyramine-free diet is required. It dissolves instantly after oral administration. An empty stomach increases its effectiveness.

It dissolves instantly after oral administration. Olanzapine is an orally disintegrating tablet, meaning that it will instantly dissolve on contact with moisture. It can be given intramuscularly, but Z-track administration is not necessary. Tyramine-free diets are necessary with monoamine oxidase inhibitors, not antipsychotics. An empty stomach is not necessary with this medication.

Which statements indicate a nurse has a correct understanding of interleukin-2? It enhances natural killer cells. It produces an antiviral effect. It is used to treat various malignancies. It is used as an anti-inflammatory agent. It stimulates differentiation of T-lymphocytes.

It enhances natural killer cells. It is used to treat various malignancies. It stimulates differentiation of T-lymphocytes. Interleukin-2 is used clinically to enhance natural killer cells, treat various malignancies, and stimulate differentiation of T-lymphocytes. Interleukin-1 is used as an anti-inflammatory agent. The antiviral effect is produced by interferons.

An adolescent who has been prescribed prednisone and vincristine for leukemia tells the nurse that he is very constipated. What should the nurse cite as the probable cause of the constipation? It is a side effect of the vincristine. The spleen is compressing the bowel. It is a toxic effect from the prednisone. The leukemic mass is obstructing the bowel.

It is a side effect of the vincristine. Constipation is a side effect of vincristine because it slows gastrointestinal motility. An enlarged spleen will put pressure on the stomach and diaphragm, not on the large bowel. Constipation is not a toxic effect of prednisone. It is unlikely that leukemia is causing an obstruction.

Which statements are true about fentanyl use in children? It is an opioid analgesic. It can be used along with midazolam. It is used for long procedures like surgeries in pediatrics. Its dose must be reduced when given along with midazolam. It must be administered rapidly when given intravenously.

It is an opioid analgesic. It can be used along with midazolam. Its dose must be reduced when given along with midazolam. Fentanyl is an opioid analgesic which can be used along with midazolam. When used in combination with midazolam, the dose must be reduced to prevent oversedation. Fentanyl is used for short procedures such as wound suturing. When it is given intravenously, it must be given slowly because rapid administration may cause chest rigidity, which will require treatment with a muscle relaxant.

A nurse is teaching the parents of an 8-year-old child who is taking a high dose of oral prednisone for asthma. What critical information about prednisone will be included? It protects against infection. It should be stopped gradually. An early growth spurt may occur. A moon-shaped face will develop.

It should be stopped gradually. Gradual weaning from prednisone is necessary to prevent adrenal insufficiency or adrenal crisis. Prednisone depresses the immune system, thereby increasing susceptibility to infection. The drug usually suppresses growth. A moon face may occur, but it is not a critical, life-threatening side effect.

The client is prescribed potassium iodide solution prior to surgery for a subtotal thyroidectomy. What explanation should the nurse give as to why this medication should be taken? The metabolic rate of the body will increase. It will reduce the risk of hemorrhage during surgery. It will maintain the functioning of the parathyroid glands. The amount of thyroid hormones being secreted will decrease.

It will reduce the risk of hemorrhage during surgery. Potassium iodide, which aids in decreasing vascularity of the thyroid gland, decreases the risk for hemorrhage. Thyroid hormone antagonists help decrease the body's metabolism. Potassium iodide does not regulate parathyroid function. Thyroid hormone antagonists help decrease the amount of thyroid hormones being secreted.

The nurse is caring for a client who is receiving intermittent intravenous piggyback doses of vancomycin every 12 hours. The primary healthcare provider prescribes trough levels of the antibiotic. The nurse schedules the blood sample to be obtained at what time? Just before the medication is administered Between 30 and 60 minutes after the infusion is completed Six hours after the dose is completely infused In the morning before the client eats breakfast

Just before the medication is administered Trough levels are measured in relation to the time a drug is administered. The trough level for a drug is drawn just before a drug is given, when the drug's level is at its lowest. Any other time would be inaccurate for a drug's trough level. The drug's peak level is drawn 30 to 60 minutes after the infusion is completed. Whether the client eats breakfast does not affect this drug's trough levels, as it is an intravenous infusion.

A client who is receiving medication for an eye disorder reports bleeding in the eye. Which drug will the nurse most likely observe written in the medication administration record? Ketorolac Trifluridine Natamycin Ciprofloxacin

Ketorolac Ketorolac is a nonsteroidal antiinflammatory (NSAID) drug that may disrupt platelet aggregation and can lead to bleeding in the eyes. Trifluridine is the topical antiviral agent that may cause sensitive reactions such as itching. Natamycin is an antifungal agent that may cause itching lids and burning eyes due to sensitivity. Ciprofloxacin is an antiinfective agent that may cause blindness if not taken in prescribed amounts.

The nurse cares for a client with schizophrenia and who is receiving ziprasidone. Which conditions in the client may indicate discontinuation of the drug? Leukopenia Tachycardia Hypokalemia Hypermagnesemia Prolonged QT interval

Leukopenia Hypokalemia Prolonged QT interval Ziprasidone is a second generation antipsychotic drug indicated for schizophrenia. The drug may cause leukopenia, hypokalemia, and hypomagnesemia. This drug may cause a prolonged QT interval, which indicates torsades de points. Bradycardia may occur in torsades de pointes, but not tachycardia.

A client who has a diagnosis of endometriosis is concerned about the side effect of hot flashes from her prescribed medications. Which medication should the nurse explain causes this side effect? Estrogen Leuprolide Diclofenac Ergonovine

Leuprolide Leuprolide decreases the levels of luteinizing hormone and follicle-stimulating hormone, as well as hormone-dependent tissue. One of its side effects is hot flashes. Estrogen affects the release of pituitary gonadotropins and inhibits ovulation; it is contraindicated because the goal of treatment is to suppress the action of estrogen on the endometrial tissue. Diclofenac is used for primary dysmenorrhea; it is a nonsteroidal antiinflammatory drug that inhibits prostaglandin synthesis. Ergonovine is used to induce contraction of the postpartum uterus.

Which gonadotropin-releasing hormone agonists are used to treat endometriosis? Trazodone Diclofenac Leuprolide Isotretinoin Nafarelin acetate

Leuprolide Nafarelin acetate Leuprolide and nafarelin acetate are gonadotropin-releasing hormone (GnRH) agonists used to treat endometriosis. Trazodone is used in cases of erectile dysfunction. Diclofenac is a nonsteroidal antiinflammatory drug used to relieve pain in endometriosis. Isotretinoin is an oral agent that is effective against severe cystic acne.

The nurse finds that a client undergoing treatment for osteomyelitis is showing signs of Achilles tendon rupture. Which medication does the nurse ask the primary healthcare provider to reconsider? Gentamycin Levofloxacin Acetaminophen Cyclobenzaprine

Levofloxacin Tendon rupture (especially the Achilles tendon) can occur with use of the fluoroquinolones (e.g., ciprofloxacin, levofloxacin). Aminoglycosides such as gentamycin do not cause rupture of the Achilles tendon; instead, gentamycin can cause ototoxicity and nephrotoxicity. Acetaminophen is a nonopioid analgesic used to manage pain; it does not cause rupture of the Achilles tendon. Cyclobenzaprine is a muscle relaxant; it does not cause tendon rupture.

A nurse is caring for a client in labor whose cervix is dilated 6 cm. The client is receiving epidural analgesia. What common response to regional anesthesia does the nurse anticipate? Urticaria Lightheadedness Elevated temperature Sensation of chilliness

Lightheadedness Lightheadedness may indicate hypotension resulting from the vasodilation commonly associated with epidural analgesia. Urticaria is associated with an allergic response; this is not a common reaction to regional anesthesia. An increase in temperature may be a response to a developing infection or dehydration; these are rare adverse occurrences with regional anesthesia. Feeling chilled is an allergic response, which is not a common reaction to regional anesthesia.

Which type of drugs readily crosses the placenta? Polar drugs Ionized drugs Lipid-soluble drugs Protein-bound drugs

Lipid-soluble drugs Drugs that are lipid soluble penetrate the placenta in higher concentrations. Polar drugs are not transferred in higher concentrations through the placenta. Nonionized drugs are more likely to be transferred through the placenta than ionized drugs. Protein-bound drugs remain in the maternal plasma because the molecules are too large to cross the placenta.

The laboratory report of a pediatric client who is taking psychotherapeutic drugs indicates decreased bone density. Which drug might be responsible for this condition? Lithium Haloperidol Amitriptyline Phenothiazine

Lithium Lithium, a mood-stabilizing drug, may decrease bone density or bone formation in children. Haloperidol is associated with extrapyramidal side effects. Amitriptyline, a tricyclic antidepressant, is associated with nervousness, sleep disorders, increased blood pressure, and gastrointestinal upset. Phenothiazine is associated with extrapyramidal side effects and anticholinergic side effects such as dry mouth, constipation, and urine retention.

The nurse cares for a client diagnosed with bipolar disorder who was prescribed drug therapy. Laboratory reports reveal that the client's thyroxine levels are low. Which drug might have led to this condition? Lithium Fluoxetine Risperidone Carbamazepine

Lithium Lithium is used to treat bipolar disorder. Decreased levels of thyroxine and triiodothyronine may indicate hypothyroidism. Lithium may cause a goiter, which is associated with hypothyroidism. Fluoxetine is a serotonin reuptake inhibitor that may lead to hyponatremia. Risperidone is a second generation antipsychotic used to treat bipolar disorder that does not cause hypothyroidism. Carbamazepine is an antiepileptic drug used to treat bipolar disorder; this drug may cause leukopenia, anemia, and thrombocytopenia.

A client reports severe right-sided headache with runny nose, droopy eyelids, and tearing of the eye on the right side. Which drugs are specific to treat this headache? Lithium Acetaminophen Naproxen Ibuprofen Oral glucosamine

Lithium Oral glucosamine cluster headache

A 1-year-old exhibits a runny nose and cough after being administered a vaccine via the intranasal route. Which vaccine may have been administered to the child? Rotavirus Inactivated influenza Live attenuated influenza Haemophilus influenzae type b

Live attenuated influenza Live attenuated influenza vaccines administered intranasally may cause such mild side effects as a runny nose and cough. Rotavirus vaccines may cause a runny nose, but these vaccines are administered orally. Inactivated influenza vaccine is administered intramuscularly. Haemophilus influenzae type b vaccine is administered intramuscularly and may cause fever and local reactions.

A client has severe diarrhea, and the health care provider prescribes intravenous fluids, sodium bicarbonate, and an antidiarrheal medication. Which drug does the nurse expect the health care provider to prescribe? Psyllium Bisacodyl Loperamide Docusate sodium

Loperamide Loperamide inhibits peristalsis and prolongs transit time by its effect on the nerves in the muscle wall of the intestines. Bisacodyl is a laxative, not an antidiarrheal; it increases gastrointestinal motility. Psyllium is not an antidiarrheal; it is a bulk laxative that promotes easier expulsion of feces. Docusate sodium corrects constipation, not diarrhea; water and fat are increased in the intestine, permitting easier expulsion of feces.

Which antihistamine is considered safe for a woman who plans to breast-feed? Sertraline Loratadine Clemastine Bromocriptine

Loratadine Loratadine does not get excreted in the breast milk. Therefore it is a safe drug for a lactating mother to take. Sertraline is an antidepressant that is safe for lactating women. Clemastine and bromocriptine are contraindicated in lactating women.

A healthcare provider prescribes phenobarbital sodium for a client who had a tonic-clonic seizure. The nurse assesses the client's knowledge after teaching about the adverse effects of this drug. What responses should the client identify as a reason for calling the healthcare provider? Loss of appetite or persistent fatigue Anal itching or dizziness when standing up Diarrhea or a rash on the upper part of the body Decreased tolerance to common foods or constipation

Loss of appetite or persistent fatigue Phenobarbital depresses the central nervous system, particularly the motor cortex, producing adverse effects such as lethargy, loss of appetite, depression, and vertigo. Anal itching, diarrhea, rashes, and decreased tolerance to common foods or constipation are not side effects of phenobarbital.

A pregnant woman was prescribed a drug for pregnancy-induced hypertension. Later, the client developed muscle weakness, edema, and nausea for which calcium gluconate was administered. What drug was administered to the client to treat pregnancy-induced hypertension? Nifedipine Terbutaline Indomethacin Magnesium sulfate

Magnesium sulfate Magnesium sulfate can be administered for pregnancy-induced hypertension; this medication may cause magnesium toxicity. Signs of this toxicity include muscle weakness, edema, and nausea. Calcium gluconate is co-administered along with magnesium sulfate to counteract this toxicity. Nifedipine, an antihypertensive agent, is listed as a category C drug, and can be safely used to treat pregnancy-induced hypertension. Terbutaline is a beta-adrenergic blocker used to manage preterm labor. Indomethacin is a nonsteroidal antiinflammatory drug used as a tocolytic for the management of preterm labor.

The nurse is caring for a client who has been admitted with partial- and full-thickness burns over 25% of the total body surface area. Lactated Ringer solution and 5% dextrose have been prescribed. What is the purpose of these fluids? Prevent fluid shifts Expand the plasma Maintain blood volume Replace electrolytes lost

Maintain blood volume Fluids during the first 48 hours are given to replace fluid lost from the intravascular compartment to interstitial spaces. Administration of fluids treats the fluid shifts but does not prevent them. Lactated Ringer solution and 5% dextrose in saline are not plasma expanders, as is albumin. Electrolytes specifically are replaced based on serial assessments of serum electrolytes and arterial blood gases.

What is the priority nursing intervention for a young infant who has an intravenous (IV) line in place after undergoing abdominal surgery? Administering oral fluids Limiting handling by parents Weighing diapers after each voiding Maintaining patency of the intravenous catheter and tubing.

Maintaining patency of the intravenous catheter and tubing. It is imperative that the nurse monitor the IV site and tubing for patency. Signs of obstruction or infiltration must be detected and, if needed, a new means of circulatory access must be obtained quickly. Oral fluids are not administered after abdominal surgery until peristalsis has returned. There is no reason to limit handling the infant as long as the IV site is not disturbed. Parent-infant contact should be encouraged. Although an accurate output record, which includes the number of voidings, is important, maintenance of the IV infusion is the priority.

Which medication will the nurse administer that prevents cellular infection with human immunodeficiency virus (HIV)? Maraviroc Nelfinavir Delavirdine Emtricitabine

Maraviroc Maraviroc is a medication used to prevent an HIV infection by blocking the CCR5 receptors on the CD4+ T cells. It also prevents cellular HIV infections. Nelfinavir is a protease inhibitor that prevents viral replication and release of viral particles. Delavirdine is a non-nucleoside reverse transcriptase inhibitor that inhibits viral gene replication. Emtricitabine is a nucleoside reverse transcriptase inhibitor that suppresses viral replication.

A client with type 1 diabetes consistently has high glucose levels on awakening in the morning. What should the nurse instruct the client to do to differentiate between the Somogyi effect and the dawn phenomenon? Eat a snack before going to bed. Measure the blood glucose level between 2 AM and 4 AM. Identify whether morning symptoms are typical for hyperglycemia. Administer the prescribed bedtime insulin immediately before going to bed.

Measure the blood glucose level between 2 AM and 4 AM. During the hours of sleep, the Somogyi effect may be caused by a decline in the blood glucose level in response to too much insulin. The resulting hypoglycemia stimulates counterregulatory hormones, which precipitate lipolysis, gluconeogenesis, and glycogenolysis, which in turn produce rebound hyperglycemia and ketosis. Treatment involves decreasing the evening insulin. The client should check blood glucose between 2 AM and 4 AM and if the blood glucose is less than 70, the client is having a Somogyi effect. The dawn phenomenon is characterized by the release of counterregulatory hormones in the predawn hours, precipitating hyperglycemia on awakening. Treatment involves an increase in insulin. Eating a snack before going to bed should be done when insulin is taken before sleep, but it will not help to differentiate between the Somogyi effect and the dawn phenomenon. Administering the prescribed bedtime insulin immediately before going to bed depends on the insulin regimen prescribed by the health care provider and will not help to differentiate between the Somogyi effect and the dawn phenomenon. The manifestation (symptoms) of hyperglycemia has no role in differentiating the conditions.

When reviewing a drug to be administered, the nurse identifies that the package insert indicates that the Z-track injection technique should be used. Under what circumstance does the nurse expect that this technique will be necessary? Volume of medication to be administered is large. Medication is irritating to subcutaneous tissue and skin. Injection site must be massaged after it is administered. Procedure requires an air bubble to be drawn into the syringe.

Medication is irritating to subcutaneous tissue and skin. The Z-track method seals the puncture at the intramuscular level, preventing seepage of injected medication up the needle track and thereby avoiding injury to subcutaneous tissue and skin. The Z-track technique is unrelated to the volume of medication to be administered. When the volume of medication is large, it should be administered into a large muscle or divided into two syringes. Massage is avoided with the Z-track technique to help prevent the injected medication from flowing back up the needle track. Administration of a small air bubble at the completion of injection of medication into a muscle (air-lock technique) is no longer recommended because it does not increase the likelihood that medication will remain in the muscle without flowing back up the needle track.

A client expresses a desire to postpone her first pregnancy for at least 5 years. She smokes 1.5 packs of cigarettes a day, has never been pregnant, and does not want to use a barrier method. Which method does the nurse anticipate the primary healthcare provider will recommend? A birth control patch A vaginal ring Medroxyprogesterone Combined oral contraceptive pills

Medroxyprogesterone Medroxyprogesterone is a long-acting progestin-only contraceptive that is less likely to cause cardiovascular problems in women who smoke than contraceptives containing estrogen might. Vaginal rings, combined oral contraceptive pills, and the birth control patch all contain estrogen and are not recommended for women who smoke.

Which medication should the nurse question when it is prescribed for a client with acute pancreatitis? Ranitidine Cimetidine Meperidine Promethazine

Meperidine Meperidine should be avoided because accumulation of its metabolites can cause central nervous system irritability and even tonic-clonic seizures (grand mal seizures). Ranitidine is useful in reducing gastric acid stimulation of pancreatic enzymes. Cimetidine is useful in reducing gastric acid stimulation of pancreatic enzymes. Promethazine is useful as an antiemetic for clients with pancreatitis.

The nurse prepares an intravenous solution of lactated Ringer solution to replace the T-tube output of a client who had a cholecystectomy and common bile duct exploration. Which condition will improve if the administration of lactated Ringer solution is effective? Urinary stasis Paralytic ileus Metabolic acidosis Increased potassium level

Metabolic acidosis Lactated Ringer solution is an alkaline solution that replaces bicarbonate ions lost from T-tube bile drainage, thus preventing or treating acidosis. Urinary stasis is unrelated to the effectiveness of the administration of intravenous lactated Ringer solution. Paralytic ileus is unrelated to the effectiveness of the administration of intravenous lactated Ringer solution. An increased potassium level is unrelated to the effectiveness of the administration of intravenous lactated Ringer solution.

A women undergoing chemotherapy for cancer gave birth to a newborn with limb malformations. Which medication may cause limb malformations in the neonates? Methotrexate Nitrofurantoin Carbamazepine Cyclophosphamide

Methotrexate When taken during pregnancy, methotrexate may cause limb malformations. Nitrofurantoin is not an immunosuppressant; it may cause abnormally small eyes or absent eyes in fetuses. Carbamazepine is an antiepileptic drug that may cause neural tube defects. Cyclophosphamide may cause central nervous system malformations and secondary cancers.

Which drug should be excluded from the prescription for a client undergoing treatment for infertility? Methyldopa Clomiphene Menotropins Choriogonadotropin alfa

Methyldopa Methyldopa is an antihypertensive drug. When it is taken along with a fertility drug, prolactin levels may increase and cause infertility. Clomiphene and menotropins are ovulation stimulants that aid fertility. Choriogonadotropin alfa is a recombinant form of human gonadotropin hormone that promotes ovulation.

Which drug is administered to women after delivery to prevent postpartum uterine atony and hemorrhage but is not given to augment labor? Dinopristone Mifepristone Indomethacine Methylergonovine

Methylergonovine Methylergonovine is given to prevent postpartum uterine atony and hemorrhage but should not be used for the augmentation of labor or during a spontaneous abortion. Dinopristone is to terminate a pregnancy. Mifepristone is also used to induce an elective termination of a pregnancy. Indomethacin is used to maintain a pregnancy during preterm labor.

A nurse is teaching the parents of a child with attention deficit hyperactivity disorder. What does the nurse include as the most frequently prescribed medication for this disorder? Lorazepam Haloperidol Methylphenidate Methocarbamol

Methylphenidate Methylphenidate appears to act by stimulating release of norepinephrine from nerve endings in the brainstem. Lorazepam is a benzodiazepine used to treat anxiety and insomnia. Haloperidol is an antipsychotic medication. Methocarbamol is a muscle relaxant.

A client with mild diarrhea is diagnosed with a Clostridium difficile infection. Which is the first-line drug that would be used to treat this condition? Rifaximin Fidaxomicin Vancomycin Metronidazole

Metronidazole Metronidazole is the first line of treatment prescribed to clients with a Clostridium difficile infection. Rifaximin is used to treat traveler's diarrhea caused by Escherichia coli. Fidaxomicin is reserved for clients who are at risk for the relapse of or have recurrent Clostridium difficile infections. Vancomycin is preferred for serious Clostridium difficile infections.

A client visits a primary healthcare provider because of painful urination. The primary healthcare provider confirms that the client has candidiasis. Which medications would be prescribed? Tinidazole Miconazole Clotrimazole Azithromycin Metronidazole

Miconazole Clotrimazole Miconazole and clotrimazole are used to treat candidiasis. Tinidazole is used to treat trichomoniasis. Azithromycin is used to treat chlamydia. Metronidazole is used to treat bacterial vaginosis.

The postoperative prescriptions for a client who had repair of an inguinal hernia include docusate sodium daily. Before discharge, the nurse instructs the client about what potential side effect? Rectal bleeding Fecal impaction Nausea and vomiting Mild abdominal cramping

Mild abdominal cramping Mild abdominal cramping is the only side effect of docusate sodium; this emollient laxative permits water and fatty substances to penetrate and mix with fecal material. Rectal bleeding is more likely to occur with a saline-osmotic laxative. Docusate sodium promotes defecation, not constipation. Nausea and vomiting are more likely to occur with a saline-osmotic laxative.

The primary healthcare provider prescribed benzoyl peroxide to a client with severe acne. During a follow-up visit, the nurse finds that the client did not respond to the therapy. Which other medications should the nurse suspect to be beneficial for the client? Trazodone Isotretinoin Minocycline Streptomycin Erythromycin

Minocycline Streptomycin Erythromycin If moderate to severe acne does not respond to topical treatments, then systemic antibiotic therapy is indicated. Systemic antibiotic therapy includes minocycline, streptomycin, and erythromycin. Trazodone may cause priapism; it may not be beneficial for the client. Isotretinoin is an effective oral medication that is reserved for the treatment of severe cystic acne in clients who do not respond to other treatments.

What is the most widely used off-label drug for cervical ripening and the enhancement of uterine muscle tone? Misoprostol Mifepristone Dinopristone Methylergonovine

Misoprostol Misoprostol is the most widely used off-label drug for cervical ripening and the enhancement of uterine muscle tone because it is relatively affordable. Mifepristone is used to induce labor. Dinoprostone is used for cervical ripening but is not an off-label drug for cervical ripening. Methylergonovine is used to enhance myometrial tone but is not used to augment labor.

A client has been taking clomiphene citrate for 3 months to treat anovulatory cycles. Which finding should be reported to the primary healthcare provider immediately? Missed period Blurred vision Weight gain Hot flashes

Missed period Clomiphene is classified in pregnancy category X, and it should be discontinued if the client is pregnant. The client should notify her primary healthcare provider of the presumptive signs of pregnancy. Weight gain, blurred vision, and hot flashes are all common side effects of this ovulation inducer.

A client with narcolepsy tells the primary healthcare provider, "I often feel drowsy and fall asleep at inappropriate times." Which medication should the nurse anticipate being prescribed by the primary healthcare provider for this client? Modafinil Ramelteon Eszopiclone Pramipexole

Modafinil Drowsiness and an inability to remain awake while performing activities are signs of narcolepsy. Modafinil is a drug used to promote wakefulness and combat narcolepsy. Pramipexole is a dopamine agonist used to treat periodic limb movement disorder. Eszopiclone is a benzodiazepine receptor-like agent, which is prescribed for insomnia. Ramelteon is a melatonin receptor agonist used to treat insomnia.

A client with type 2 diabetes develops gout, and allopurinol is prescribed. The client is also taking metformin and an over-the-counter nonsteroidal antiinflammatory drug (NSAID). When teaching about the administration of allopurinol, what should the nurse instruct the client to do? Decrease the daily dose of NSAIDs. Limit fluid intake to one quart a day. Take the medication on an empty stomach. Monitor blood glucose levels more frequently.

Monitor blood glucose levels more frequently. Allopurinol can potentiate the effect of oral hypoglycemics, causing hypoglycemia; the blood glucose level should be monitored more frequently. NSAIDs can be taken concurrently with allopurinol. A daily fluid intake of 2500 to 3000 mL will limit the risk of developing renal calculi. Allopurinol should be taken with milk or food to decrease gastrointestinal irritation.

A healthcare provider prescribes enalapril for a client. Which is the most important nursing action? Assess the client for hypokalemia. Ensure that the medication is ingested with food. Monitor the client's blood pressure during therapy. Teach that a missed dose can be doubled at the next scheduled time.

Monitor the client's blood pressure during therapy. Enalapril is an antihypertensive. A lowering of the client's blood pressure reflects a therapeutic response and needs to be monitored frequently. The client may be at risk for hyperkalemia, not hypokalemia. Enalapril may be taken without regard to meals. Doubling a dose is unsafe as it may cause an extreme lowering of blood pressure. A missed dose can be taken as long as it is not close to the next scheduled dose.

A healthcare provider prescribes losartan for a client. Which is the most important nursing action? Assess the client for hypokalemia. Ensure that the medication is ingested with food. Monitor the client's blood pressure during therapy. Teach that a missed dose can be doubled at the next scheduled time.

Monitor the client's blood pressure during therapy. Losartan is an antihypertensive. It blocks vasoconstrictor and aldosterone-producing effects of angiotensin II at receptor sites. A lowering of the client's blood pressure reflects a therapeutic response and should be monitored frequently. The client may be at risk for hyperkalemia, not hypokalemia. Losartan may be taken without regard to meals. Doubling a dose is unsafe. A missed dose can be taken as long as it is not close to the next scheduled dose.

A client is started on chlorpromazine. To prevent life-threatening complications from the administration of this medication to an anxious, restless client, it is important that the nurse take which action? Provide adequate restraint. Monitor the client's vital signs. Protect against exposure to direct sunlight. Watch the client for extrapyramidal side effects.

Monitor the client's vital signs. Tachycardia, hyperpyrexia, and tachypnea are indications of neuroleptic malignant syndrome, which is a life-threatening complication. Restraint of any type may worsen the client's anxiety and result in struggling and increased agitation. Photosensitivity occurs most commonly when clients are taking large doses and are spending time outdoors in the sun, but it is not life threatening. Tardive dyskinesia results from prolonged large doses of phenothiazines in susceptible clients, but it is not life threatening.

Which interventions are included in the nursing care for a client receiving magnesium sulfate for severe preeclampsia? Monitoring deep tendon reflexes Assessing urine output every 8 hours Maintaining a dark, quiet environment Using a pump to regulate the medication Having calcium gluconate available at the bedside Notifying the primary healthcare provider if the respiratory rate is slower than 20 breaths/min

Monitoring deep tendon reflexes Maintaining a dark, quiet environment Using a pump to regulate the medication Having calcium gluconate available at the bedside Magnesium sulfate level is monitored closely because toxicity may occur with levels over 8 mg/dL. It works by relaxing skeletal muscle; therefore deep tendon reflexes should be assessed hourly. Maintaining a dark, quiet environment decreases stimulation and reduces the risk of seizures. Magnesium sulfate must be administered with the use of an infusion pump because it can be toxic and cause respiratory distress. Calcium gluconate is the antidote to magnesium sulfate and should be immediately available for the treatment of overdose. Assessing urine output every 8 hours is not sufficient. Urine output of less than 30 mL/hr must be reported to the primary healthcare provider. A respiratory rate slower than 12 breaths/min, not 20, must be reported to the primary healthcare provider.

A school-aged child with a seizure disorder has been taking carbamazepine for three years. What nursing intervention is most important to undertake regularly? Assessing the mouth for gingivitis Checking the pupillary reaction to light Keeping an accurate intake and output record Monitoring the child's complete blood cell counts

Monitoring the child's complete blood cell counts The side effects of carbamazepine include blood dyscrasias (e.g., thrombocytopenia, aplastic anemia, leukopenia, agranulocytosis). A side effect of long-term phenytoin, not carbamazepine, therapy is hyperplasia of the gingiva. Carbamazepine does not influence pupillary response directly. Keeping an accurate intake and output is unnecessary.

An older client has been prescribed an atypical antipsychotic medication. Which nursing interventions demonstrate that the nurse has determined the client's risk for injury? Monitoring the pulse for an irregular rhythm Sitting with the client during meals to encourage eating Offering a favorite beverage between meals to maintain hydration Assessing the temperature to determine the possibility of an infection Teaching the client about the importance of taking an anticholinergic medication

Monitoring the pulse for an irregular rhythm Assessing the temperature to determine the possibility of an infection Older clients prescribed atypical antipsychotic medications are at increased risk for death as a result of cardiovascular dysfunction and infection and should be monitored closely for such situations. This client is at risk for death related to complications of atypical antipsychotic medication therapy, but the risk is not related to poor nutrition, dehydration, or any condition that could be managed with anticholinergic therapy.

Which nursing action is the priority when administering chelation therapy for a preschool-age client? Assessing vital signs Monitoring urine output Conducting a behavioral assessment Providing education to reduce lead exposure

Monitoring urine output Adequate urinary output must be ensured with administration of calcium EDTA, the medication used for chelation therapy. Clients receiving the drug intramuscularly must be able to maintain adequate oral intake of fluids. Monitoring vital signs, conducting a behavioral assessment, and providing education to reduce lead exposure are not priority nursing actions when administering chelation therapy.

A client, admitted to the cardiac care unit with a myocardial infarction, complains of chest pain. What intervention will be most effective in relieving the client's pain? Nitroglycerin sublingually Oxygen per nasal cannula Lidocaine hydrochloride 50 mg IV bolus Morphine sulfate 2 mg intravenously (IV)

Morphine sulfate 2 mg intravenously (IV) Morphine is an opioid analgesic that acts on the central nervous system by a sympathetic mechanism. Morphine decreases systemic vascular resistance, which decreases left ventricular afterload, thus decreasing myocardial oxygen consumption. Nitroglycerin sublingually relieves anginal pain, not myocardial infarction pain. Oxygen administration elevates arterial oxygen tension, potentially improving tissue oxygenation; however, oxygen administration will not relieve the pain. Lidocaine is an antidysrhythmic, not an analgesic.

A client who is postoperative hip replacement is receiving morphine by patient-controlled analgesia and has a respiratory rate of 6 breaths/min. What intervention should the nurse anticipate? Nasotracheal suction Mechanical ventilation Naloxone administration Cardiopulmonary resuscitation

Naloxone administration Naloxone is an opioid antagonist and will reverse respiratory depression caused by opioids. Nasotracheal suction, mechanical ventilation, and cardiopulmonary resuscitation are not needed; naloxone will correct the respiratory depression.

The nurse is providing postoperative care for a client who has received a prescription for nalbuphine for pain. What side effects or adverse reactions does the nurse anticipate after administering this medication? Nausea Oliguria Sedation Dry mouth Flushed skin Orthostatic hypotension

Nausea Sedation Dry mouth Orthostatic hypotension Dry mouth, sedation, nausea, and hypotension are the most common side effects of nalbuphine HCl. The client may lose bladder control, but oliguria is not seen. Cold and clammy skin, not flushed skin, may occur.

A client with heart failure is receiving digoxin and hydrochlorothiazide. The nurse will assess for which signs and symptoms that indicate digoxin toxicity? Nausea Yellow vision Irregular pulse Increased urine output Heart rate of 64 beats per minute

Nausea Yellow vision Irregular pulse Signs and symptoms of digoxin toxicity include bradycardia, headache, dizziness, confusion, nausea, and visual disturbances (blurred vision or yellow vision). In addition, ECG findings may include heart block, atrial tachycardia with block, or ventricular dysrhythmias, all causing an irregular pulse. Increased urine output is an expected effect of the diuretic furosemide; a pulse rate of 64 beats per minute is an acceptable rate when a client is receiving digoxin.

A combination of drugs, including vincristine and prednisone, is prescribed for a child with leukemia. For which adverse effect of vincristine will the nurse assess the child? Hemolytic anemia Irreversible alopecia Hyperglycemia Neurologic complications

Neurologic complications Vincristine is highly neurotoxic, causing paresthesias, muscle weakness, ptosis, diplopia, paralytic ileus, vocal cord paralysis, and loss of deep tendon reflexes. Hematologic effects are rare; mild anemia may occur, but hemolytic anemia is not anticipated. Alopecia is reversible with cessation of the drug. Hyperglycemia is not an anticipated adverse effect.

Thiamine (vitamin B 1) and niacin (vitamin B 3) are prescribed for a client with alcoholism. Which body function maintained by these vitamins should the nurse include in a teaching plan? Neuronal activity Bowel elimination Efficient circulation Prothrombin development

Neuronal activity Thiamine and niacin help convert glucose for energy, and therefore influence nerve activity. These vitamins do not affect elimination. These vitamins are not related to circulatory activity. Vitamin K, not thiamine and niacin, is essential for the manufacture of prothrombin.

A 25-weeks pregnant client who is being treated with tenormin reports labor pain. What medication would the primary healthcare provider prescribe? Sucralfate Nifedipine Indomethacin Dexamethasone

Nifedipine Nifedipine inhibits myometrial activity by blocking calcium reflux. This action helps to reduce preterm labor. Indomethacin is commonly used to treat preterm labor. However, concomitant use of tenormin and indomethacin may increase maternal and fetal risk. Dexamethasone is administered if indomethacin and nifedipine is ineffective. Sucralfate is used to protect the stomach from gastrointestinal issues associated with indomethacin.

Which drug would be least likely observed in the prescription of a client with severe hypotension who is managing preterm labor? Nifedipine Terbutaline Indomethacin Dexamethasone

Nifedipine Nifedipine is a calcium channel blocker used to maintain pregnancy in preterm labor. Because it is a calcium channel blocker, nifedipine can cause hypotension so it should not be used in a client with severe hypotension. Terbutaline and indomethacin are used to maintain a pregnancy in preterm labor. Dexamethasone is used to maintain a pregnancy in preterm labor when other tocolytics are not beneficial.

A client who has been taking oral contraceptives for 3 months arrives at the clinic and tells the nurse that she has breakthrough bleeding between menstrual cycles. For what causative factor should the nurse first assess the client? Illness Anorexia nervosa Ectopic pregnancy Nonadherence to protocol

Nonadherence to protocol Nonadherence to the instructions for taking the oral contraceptive can alter hormone levels, and breakthrough bleeding may occur as a result. Illness and anorexia nervosa are more likely to cause amenorrhea, not breakthrough bleeding. If nonadherence is determined not to be a concern, then the nurse would assess for far less likely causes such as ectopic pregnancy.

What is the mechanism of action of norepinephrine in managing anaphylaxis? Multiple choice question Norepinephrine blocks the effects of histamine Norepinephrine inhibits the degranulation of mast cells Norepinephrine increases blood pressure and cardiac output Norepinephrine rapidly stimulates alpha- and beta-adrenergic receptors

Norepinephrine increases blood pressure and cardiac output Norepinephrine is a vasopressor that elevates the blood pressure and cardiac output in clients suffering from anaphylactic reactions. Diphenhydramine HCL blocks the effects of histamine on various organs. Corticosteroids such as dexamethasone prevent the degranulation of mast cells. Epinephrine works by rapidly stimulating alpha- and beta-adrenergic receptors.

A client is exposed to a low concentration of a teratogen in her second week of pregnancy. What could be the effect of the exposure on the fetus or child? Fetal death Child with a curved ear Normal growth of the fetus Hearing problem in the child

Normal growth of the fetus The development of the fetus occurs in three stages: presomite period (conception through week 2), embryonic period (week 3 to week 8), and fetal period (week 9 to term). During the presomite period, teratogens act in an all-or-nothing fashion. When the concentration is low, the fetus may have normal growth. Exposure to high concentrations of a teratogen during this period may cause fetal death. If the fetus is exposed to the teratogen in the embryonic stage, internal organ developmental defects may occur.

A healthcare provider prescribes ampicillin for a client with an infection. What information should the nurse include in the teaching plan about this medication? Take the ampicillin with meals. Store the ampicillin in a light-resistant container. Notify the healthcare provider if diarrhea develops. Continue the drug until a negative culture is obtained.

Notify the healthcare provider if diarrhea develops. Diarrhea is a possible side effect that can be related to superinfection or to destruction of bacterial flora in the intestine; it can lead to fluid and electrolyte imbalance. Ampicillin is absorbed best when taken with water on an empty stomach. Although storage in an airtight container is necessary, protection from light is not. A culture generally is not repeated unless the client's condition indicates that the medication was ineffective.

An 8-year-old child is being given insulin glargine before breakfast. What is the most appropriate information for the nurse to give the parents concerning a bedtime snack? Offer a snack to prevent hypoglycemia during the night. Give the child a snack if signs of hyperglycemia are present. Avoid a snack because the child is being treated with long-acting insulin. Keep a snack at the bedside in case the child gets hungry during the night.

Offer a snack to prevent hypoglycemia during the night. Insulin glargine is released continuously throughout the 24-hour period; a bedtime snack will prevent hypoglycemia during the night. Providing a snack when signs of hyperglycemia are present is unsafe because it intensifies hyperglycemia; if hyperglycemia is present, the child needs insulin. Because insulin glargine is a long-acting insulin, bedtime snacks are recommended to prevent a hypoglycemic episode during the night. When hypoglycemia develops, the child will be asleep; the child should eat the snack before going to bed.

What will a nurse teach the parents of a toddler with newly diagnosed cystic fibrosis about the administration of vitamins A, D, E, and K? Offer them in a water-miscible form. Give them during meals and snack times. The dosage is based on the child's height and weight. Present them to the child with fruit juice rather than milk.

Offer them in a water-miscible form. Because children with cystic fibrosis do not absorb fat-soluble vitamins effectively, they should be given in a water-miscible form. These vitamins may be given with other vitamins once a day; pancreatic enzymes are administered with meals and snacks. The nurse does not have to base the dosage of these vitamins on the child's height and weight. There is no reason to select juice over milk when administering these vitamins.

A client is being admitted for a total hip replacement. When is it necessary for the nurse to ensure that a medication reconciliation is completed? After reporting severe pain On admission to the hospital Upon entering the operating room Before transfer to a rehabilitation facility At time of scheduling for the surgical procedure

On admission to the hospital Before transfer to a rehabilitation facility Medication reconciliation involves the creation of a list of all medications the client is taking and comparing it to the healthcare provider's prescriptions on admission or when there is a transfer to a different setting or service, or discharge. A change in status does not require medication reconciliation. A medication reconciliation should be completed long before entering the operating room. Total hip replacement is elective surgery, and scheduling takes place before admission; medication reconciliation takes place when the client is admitted.

The primary healthcare provider notices that a client exhibits a period of mania followed by hypomania and depression and prescribes lithium carbonate. What is the mode of administration of the prescribed drug? Oral route Sublingual route Intravenous route Transdermal route

Oral route Episodes of mania followed by hypomania and depression are indicative of bipolar disease. Lithium carbonate is the drug of choice to treat this condition and should be administered only by the oral route. The sublingual route, intravenous route, and transdermal routes of drug administration are inappropriate for the administration of lithium salts.

A nurse recalls that the shift of body fluids associated with the intravenous administration of albumin occurs by which process? Osmosis Diffusion Active transport Hydrostatic pressure

Osmosis Osmosis is the movement of fluid from an area of lesser solute concentration to an area of greater solute concentration. Diffusion is the movement of particles across a semipermeable membrane from an area of greater concentration of particles to an area of lesser concentration of particles. In active transport, molecules move against a concentration gradient; this differs from diffusion and osmosis because metabolic energy is expended. Hydrostatic pressure, the pressure exerted within a closed system, is known as filtration force. Filtration is the passage of fluid through a material that prevents the passage of certain constituents; hydrostatic pressure moves fluid by pressure and concentration gradients.

A nurse is caring for a 7-year-old child in the pediatric intensive care unit who has increased intracranial pressure as a result of head trauma. The healthcare provider prescribes intravenous mannitol. The nurse monitors the child's intracranial pressure and urine output because mannitol belongs to which classification of diuretics? Loop Osmotic Potassium sparing Carbonic anhydrase inhibitor

Osmotic Osmotic diuretics, such as mannitol, increase the osmotic pressure of glomerular filtrate and thus decrease absorption of sodium; they are used to treat cerebral edema and increased intraocular pressure. Loop diuretics, such as furosemide, inhibit resorption of sodium and potassium in the loop of Henle; they are used for heart failure and pulmonary edema. Potassium-sparing diuretics, such as spironolactone, interfere with sodium resorption in the distal tubules, thus decreasing potassium excretion; they are used to treat cirrhotic ascites and pulmonary edema. Carbonic anhydrase inhibitors, such as acetazolamide, increase sodium excretion by decreasing sodium-hydrogen ion exchange. They are used to treat seizure disorders and open-angle glaucoma.

A primary healthcare provider prescribes teriparatide, a parathyroid hormone agonist, for a client with osteoporosis. Which statement about this drug does the nurse recognize as accurate? It requires increased intake of vitamin A. It prevents existing bone from being destroyed. Sunscreen should be used to prevent vitamin D absorption. Osteoblastic activity is stimulated more than osteoclastic activity.

Osteoblastic activity is stimulated more than osteoclastic activity. Teriparatide is a 34-amino acid polypeptide that represents the biologically active part of human parathyroid hormone; it enhances bone microarchitecture and increases bone mass and strength by stimulating activity by osteoblasts. Supplemental intake of vitamin A should not exceed recommended daily requirements; too much vitamin A has been associated with bone loss and an increased incidence of fractures. Alendronate sodium, a regulator of bone metabolism, not teriparatide, inhibits osteoclast-mediated bone resorption, minimizing bone destruction and loss of bone density. Sunscreen should be avoided to promote exposure to the sun so that vitamin D can be converted in the skin; vitamin D helps the body absorb calcium. Sunscreen should be used after 5 to 20 minutes of exposure to prevent the negative effects of prolonged exposure to ultraviolet rays.

A pregnant client is prescribed heparin to prevent the risk of thromboembolism. Which adverse effects should the nurse anticipate with this medication? Osteoporosis Suppress contractions in labor Increased risk of serious bleeding Stimulation of uterine contraction Compression fractures of the spine

Osteoporosis Heparin is an anticoagulant. When heparin is taken concurrently during pregnancy, it may cause osteoporosis.Which in turn can cause compression fractures of the spine. The use of aspirin in the near term of pregnancy can suppress contractions in labor. The increased risk of serious bleeding also occurs with use of aspirin during pregnancy. The use of prostaglandin during pregnancy can cause stimulation of uterine contraction and can cause abortion.

A client at 38 weeks' gestation is admitted for induction of labor. Her membranes ruptured 12 hours ago. There are no other signs of labor. Which medication does the nurse anticipate will be prescribed? Oxytocin Estrogen Ergonovine Progesterone

Oxytocin Oxytocin is a small-polypeptide hormone synthesized in the hypothalamus and secreted from the neurohypophysis (posterior pituitary gland) during parturition or suckling; it promotes powerful uterine contractions and is therefore used to induce labor. Estrogen suppresses follicle-stimulating and luteinizing hormones, thereby helping maintain a pregnancy. Ergonovine can induce sustained contractions, which is contraindicated during labor; it may be prescribed in the postpartum period to promote or maintain a contracted uterus. Progesterone causes hyperplasia of the endometrium in preparation for implantation of the fertilized ovum; later it helps maintain the pregnancy.

A client is prescribed venlafaxine. Which disorders is this drug used to treat? Panic disorder Bipolar disorder Social anxiety disorder Major depression Obsessive compulsive disorder

Panic disorder Social anxiety disorder Major depression Panic disorders, social anxiety disorders, and major depression can be treated with venlafaxine. Bipolar disorders are generally treated with mood stabilizers, atypical antipsychotics, and antiepileptic drugs. Obsessive compulsive disorders are treated with selective serotonin reuptake inhibitors.

A school-aged child who has just arrived from Africa has been exposed to diphtheria, and a nurse in the pediatric clinic is to administer the antitoxin. Which type of immunity does the antitoxin confer? Active natural Passive natural Active artificial Passive artificial

Passive artificial In the creation of passive artificial immunity an antibody is produced in another organism and then injected into the infected or presumed infected person to provide immediate immunity against the invading organism. Active natural immunity takes too much time to develop; this child needs immediate protection. Passive natural immunity is acquired from the mother and is effective only during the first few months of life. Active artificial immunity takes too much time to develop; the child needs immediate protection.

A client at 38 weeks' gestation is admitted to the prenatal unit with preeclampsia. A loading dose of magnesium sulfate is administered, and the dosage is subsequently lowered to a maintenance dosage. What is the most important parameter for the nurse to assess while monitoring the client for magnesium sulfate toxicity? Pulse rate Daily weight Patellar reflex Blood pressure

Patellar reflex An absence of deep tendon reflexes is one of the first signs of magnesium sulfate toxicity. Magnesium sulfate interferes with the release of acetylcholine at the synapses, thereby decreasing neuromuscular irritability. Magnesium sulfate toxicity cannot be determined by alterations in the maternal heart rate or blood pressure. Diuresis and its related weight loss are signs of the therapeutic effect of magnesium sulfate.

Which antimicrobial drugs are safe during breast-feeding? Penicillins Macrolides Tetracycline Cephalosporins Chloramphenicol

Penicillins Macrolides Cephalosporins Penicillins, macrolides, and cephalosporins are considered safe drugs during breast-feeding as they are least likely to affect the infant. Tetracycline and chloramphenicol should be avoided during breast-feeding.

The primary healthcare provider prescribed selective serotonin reuptake inhibitor therapy for a client who survived a bomb blast. During a follow-up visit, the nurse finds that the symptoms did not subside. Which other medications would be beneficial for the client? Buspirone Phenelzine Trazodone Mirtazapine Imipramine

Phenelzine Mirtazapine Imipramine A client who survived a bomb blast would have posttraumatic stress disorder (PTSD). Selective serotonin reuptake inhibitors (SSRIs) are first-line agents used to treat PTSD. If SSRIs are not effective, then monoamine oxidase inhibitors (such as phenelzine or mirtazapine) or a tricyclic antidepressant (such as imipramine) may be effective in treating PTSD. Buspirone and trazodone are not used to treat PTSD.

Which drug may cause sudden infant death syndrome? Nalidixic acid Promethazine Phenothiazines Hexachlorophene

Phenothiazines Phenothiazines may cause sudden infant death syndrome. Nalidixic acid may cause cartilage erosion in children. Promethazine may cause respiratory depression in children under the age of 2 years. Hexachlorophene causes central nervous system toxicity, especially in infants.

A nurse is caring for a client who is receiving total parenteral nutrition (TPN).The nurse should monitor the client for which complications? Phlebitis Infection Hepatitis Anorexia Dysrhythmias

Phlebitis Infection Phlebitis may occur because the hypertonic nature of the infusion is irritating to the vein. The concentration of glucose in the solution is a culture medium that supports the growth of microorganisms. Hepatitis usually is not associated with total parenteral nutrition. Anorexia often is present before the medical decision is made to begin total parenteral nutrition. Dysrhythmias are not related to total parenteral nutrition, but may be a sign of hyperkalemia or hypokalemia.

Which drug would be used to treat a client with severe motor tics, barking cries, and outbursts of obscene language? Loxapine Pimozide Thiothixene Thioridazine

Pimozide Severe motor tics, barking cries, and outbursts of obscene language are signs of Tourette syndrome. Pimozide is a high-potency drug used to treat this syndrome. Loxapine is a medium-potency agent indicated only for schizophrenia. Thiothixene is a high-potency agent indicated for schizophrenia. Thioridazine is a low-potency first-generation antipsychotic indicated for schizophrenia.

A client is admitted to the hospital with deep partial-thickness burns to both hands and forearms after an accident. How should the nurse apply the prescribed antimicrobial medication? Place the medication directly on the dressing in a thick layer using clean gloves. Place the medication directly on the burn wound in a thin layer using sterile gloves. Put the medication in a Hubbard tank and saturate sterile dressings with it before applying the dressings to the burns. Put the medication in a Hubbard tank and allow the client to soak in the tank for several minutes every day.

Place the medication directly on the burn wound in a thin layer using sterile gloves. Sterile aseptic technique is necessary for an open wound, and a thin layer of ointment is applied directly to the affected area. Surgically aseptic, not medically aseptic, technique is used. Although some medications may be placed directly in the tank, antimicrobial medications are placed directly on the affected area using surgically aseptic technique.

A client is scheduled to have a thyroidectomy. Which medication does the nurse anticipate the health care provider will prescribe to decrease the size and vascularity of the thyroid gland before surgery? Vasopressin Levothyroxine Propylthiouracil Potassium iodide

Potassium iodide Potassium iodide adds iodine to the body fluids, exerting negative feedback on the thyroid tissue and decreasing its metabolism and vascularity. Vasopressin is a pituitary hormone. Propylthiouracil interferes with production of thyroid hormone, but causes increased vascularity and size of the thyroid. Levothyroxine is a thyroid hormone that may be administered after a thyroidectomy if the client develops hypothyroidism.

A healthcare provider prescribes 10 mL of a 10% solution of calcium gluconate for a client with a severely depressed serum calcium level. The client also is receiving digoxin 0.25 mg daily and an intravenous (IV) solution of D 5W. What feature of calcium gluconate does the nurse consider? Can be added to any IV solution Must be administered via an intravenous piggyback Is nonirritating to surrounding tissues Potentiates the action of the digoxin preparation

Potentiates the action of the digoxin preparation Toxicity can result because the action of calcium ions is similar to that of digoxin. Calcium gluconate cannot be added to a solution containing carbonate or phosphate because a dangerous precipitation will occur. Calcium gluconate can be added to the IV solution the client is receiving. If calcium infiltrates, sloughing of tissue will result.

A woman is exposed to indomethacin during the third trimester of pregnancy. Which teratogenic effect of the medication is expected in the newborn? Neural tube defects Neonatal hypoglycemia Cleft lip with cleft palate Premature closure of the ductus arteriosus

Premature closure of the ductus arteriosus Indomethacin is a nonsteroidal antiinflammatory drug that may cause premature closure of the ductus arteriosus in newborns receiving long-term maternal dosing. Neural tube defects cannot be expected since the medication is taken in the third trimester. Neonatal hypoglycemia is caused only by oral hypoglycemic drugs. Cleft lip with cleft palate is expected by the medications administered within 8 weeks of gestation.

A client with a partial occlusion of the left common carotid artery is to be discharged while still receiving warfarin. Which clinical adverse effect should the nurse identify as a reason for the client to seek medical consultation regarding an adverse effect of the drug? Presence of blood in urine Increased swelling of the ankles Diminished ability to concentrate Occurrence of transient ischemic attacks

Presence of blood in urine Warfarin causes an increase in the prothrombin time and international normalized ratio (INR) level, leading to an increased risk for bleeding. Any abnormal or prolonged bleeding must be reported, because it may indicate an excessive level of the drug. Increased swelling of the ankles, diminished ability to concentrate, and occurrence of transient ischemic attacks are not signs of bleeding, the primary concern with warfarin.

A client is receiving penicillin G and probenecid for syphilis. What rationale should the nurse give for the need to take these two drugs? Each drug attacks the organism during different stages of cell multiplication. The penicillin treats the syphilis, whereas the probenecid relieves the severe urethritis. Probenecid delays excretion of penicillin, thus maintaining blood levels for longer periods. Probenecid decreases the potential for an allergic reaction to penicillin, which treats the syphilis.

Probenecid delays excretion of penicillin, thus maintaining blood levels for longer periods. Probenecid results in better use of penicillin by delaying the excretion of penicillin through the kidneys. Penicillin destroys Treponemapallidum during all stages of its development; probenecid delays the excretion of penicillin. Probenecid does not treat urethritis. Probenecid does not prevent allergic reactions.

Which drug impairs fertility when administered along with fertility drugs? Clomiphene Menotropins Promethazine Choriogonadotropin alfa

Promethazine When taken with fertility drugs, promethazine increases prolactin concentration, which may impair fertility. Clomiphene and menotropins are ovulation stimulants given to induce ovulation in infertile women. Choriogonadotropin alfa is a recombinant form of human gonadotropin hormone; this drug is an ovulation stimulant.

A nurse is teaching a pregnant client with sickle cell anemia about the importance of taking supplemental folic acid. How does folic acid help this client? Lessens sickling of red blood cells Prevents vaso-occlusive crises Decreases cellular oxygen need Promotes production of hemoglobin

Promotes production of hemoglobin Folic acid is needed to produce heme for hemoglobin. Supplementation with folic acid does not reduce sickling, and it will not prevent vaso-occlusive crisis. Adequate oxygenation and hydration help prevent vaso-occlusive crisis. It does not decrease cellular oxygenation need, although, through production of hemoglobin, it can improve oxygen supply.

A nurse is teaching a breast-feeding client about medications that are safe and unsafe for her to take. Which medication is contraindicated? Heparin Propylthiouracil Gentamicin Diphenhydramine

Propylthiouracil The concentration of propylthiouracil excreted in breast milk is 3 to 12 times higher than its level in maternal serum; this may cause agranulocytosis or goiter in the infant. Heparin is not excreted in breast milk. The amount of breast milk excretion of gentamicin is unknown, but it can be given to infants directly without adverse effects. Diphenhydramine is excreted in breast milk, but it does not adversely affect the infant when therapeutic doses are given to the mother.

A pregnant client who has asthma is expected on the unit for induction of labor. What medication should the nurse question when evaluating newly written prescriptions from the primary healthcare provider? Albuterol inhaler Epidural anesthesia Intravenous D 5W with piggyback oxytocin Prostaglandin E 2 vaginal suppository

Prostaglandin E 2 vaginal suppository One side effect of prostaglandin E2 is bronchoconstriction, which may cause a bronchospasm in a client with asthma. An albuterol inhaler may be used as needed. Epidural anesthesia is not contraindicated for pregnant clients with asthma. Intravenous D 5W with piggyback oxytocin is not contraindicated for a pregnant client with asthma.

The nurse is educating a client about protease inhibitors. What statement about protease inhibitors is true? Protease inhibitors prevent viral replication. Protease inhibitors prevent the interaction between viral material and the CD4+ T-cell. Protease inhibitors prevent viral and host genetic material integration. Protease inhibitors prevent the clipping of the viral strands into small functional pieces.

Protease inhibitors prevent viral replication. Protease inhibitors act by preventing viral replication and release of viral particles. NRTIs inhibits the transformation of viral single-stranded ribonucleic acid into host double-stranded deoxyribonucleic acid (DNA) by the action of the enzyme reverse transcriptase. Entry inhibitor drugs prevent the binding of the virus to the CD4 receptors. Integrase inhibitor drugs prevent the integration of viral material into the host's DNA by the action of the enzyme integrase.

Before a cholecystectomy, vitamin K is prescribed. The nurse recognizes that this is ordered because vitamin K contributes to the formation of which substance? Bilirubin Prothrombin Thromboplastin Cholecystokinin

Prothrombin Vitamin K is necessary in the formation of prothrombin to prevent bleeding. It is a fat-soluble vitamin and is not absorbed from the gastrointestinal (GI) tract in the absence of bile. Bilirubin is the bile pigment formed by the breakdown of erythrocytes. Thromboplastin converts prothrombin into thrombin during the process of coagulation. Cholecystokinin is the hormone that stimulates contraction of the gallbladder.

A school-aged child is receiving 45 units of intermediate-acting insulin at 7:00 AM and 7:00 PM. What will the nurse tell the parents regarding a bedtime snack? Offer a snack at bedtime if there are signs of hyperglycemia. Provide a bedtime snack to prevent hypoglycemia during the night. Withhold the snack after dinner to prevent hyperglycemia during sleep. Leave a snack at the bedside in case the child becomes hungry during the night.

Provide a bedtime snack to prevent hypoglycemia during the night. Intermediate-acting insulin peaks in 4 to 12 hours; a bedtime snack will prevent hypoglycemia during the night. Offering a snack at bedtime if there are signs of hyperglycemia is unsafe because it will intensify the hyperglycemia; if hyperglycemia is present, the child needs insulin. Bedtime snacks are recommended for people taking intermediate-acting insulin. When hypoglycemia develops, the child will be asleep; the snack should be eaten before bed.

What nursing care should be included for a client who is receiving doxorubicin for acute myelogenous leukemia? Serving hot liquids with each meal Providing frequent oral hygiene and increasing oral fluids Emphasizing that the disease will be cured with this treatment Administering medications intramuscularly and encouraging activity

Providing frequent oral hygiene and increasing oral fluids Stomatitis and hyperuricemia are possible complications of therapy; therefore, oral care and hydration are important. Food and fluids with extremes in temperature should be avoided because of the common occurrence of stomatitis. Emphasizing that the disease will be cured with this treatment may provide false reassurance. Abnormal bleeding is a common problem, and thus injections are contraindicated; rest is important for increased fatigability.

A 25-year-old woman on estrogen therapy has a history of smoking. Which complication does the nurse anticipate in the client? Osteoporosis Hypermenorrhea Endometrial cancer Pulmonary embolism

Pulmonary embolism Estrogen therapy increases the risk of pulmonary embolisms in clients who have a history of smoking because the medication affects blood circulation and hemostasis. Osteoporosis may be caused by reduced bone density observed in postmenopausal woman. Hypermenorrhea (excessive menstrual bleeding) is treated with estrogen therapy. Endometrial cancer is a complication of estrogen therapy seen in postmenopausal woman.

What will the nurse do when determining whether a client is experiencing adverse effects of risperidone? Monitor for episodes of diarrhea. Test sensation of lower extremities. Question if dizziness is experienced. Auscultate breath sounds to detect wheezing.

Question if dizziness is experienced. Hypotension and dizziness are adverse effects of risperidone. Risperidone may cause constipation, not diarrhea. It does not affect the neuromuscular or cardiovascular function of the legs; numbness and coldness of the feet do not occur. Risperidone does not cause wheezing or shortness of breath.

The primary healthcare provider prescribes an atypical antipsychotic drug to a client and asks the nurse to set up an appointment for the client to see an ophthalmologist. Which drug was prescribed to the client? Clozapine Quetiapine Ziprasidone Chlorpromazine

Quetiapine Quetiapine is a second-generation antipsychotic drug that may pose a risk of cataracts. Therefore, the prescriber would refer the client to an ophthalmologist for an examination of the lenses. Clozapine is a second-generation antipsychotic drug that does not have any risk of causing cataracts. Ziprasidone may cause alterations in the QT intervals. Chlorpromazine is a first-generation antipsychotic drug that does not cause cataracts.

A client begins fighting and biting other clients. The primary healthcare provider prescribes a stat injection of haloperidol. How will the nurse implement this prescription? Quickly, with an attitude of concern Before the client realizes what is happening After the client agrees to receive the injection Quietly, without any explanation of the reason for it

Quickly, with an attitude of concern Quickness is used for safety; an attitude of concern may help reduce the client's anxiety. The client must be told why sedation is being used; to do so surreptitiously will reduce the client's trust. A client who is this upset will not agree to a sedative; the client may harm self or others and must be sedated.

A pregnant client with a history of rheumatic heart disease expresses concern about the impending birth. What should the nurse tell her to expect? Induced labor Cesarean birth Regional analgesia Inhalation anesthesia

Regional analgesia Regional analgesia, such as an epidural, will relieve the stress of pain, and it does not compromise cardiovascular function. Induced labor is often more stressful and painful than spontaneous labor. Major abdominal surgery is performed in clients with cardiac problems only when absolutely necessary. Inhalation anesthesia can compromise cardiovascular function.

A head nurse teaches a group of nursing students about the physiologic changes that may occur during pregnancy. What statement by a nursing student indicates effective teaching? Hepatic metabolism decreases for antiseizure drugs. Enterohepatic recirculation causes a decrease in drug reabsorption. Bowel motility decreases, causing a decrease in intestinal transit time. Renal blood flow is doubled during the third trimester, which increases the glomerular filtration rate.

Renal blood flow is doubled during the third trimester, which increases the glomerular filtration rate. Renal blood flow doubles during the last trimester and the glomerular filtration rate is increased. Therefore drugs that are eliminated through glomerular filtration need an increased dose. Hepatic metabolism is altered and increases for some antiseizure drugs. Drugs that undergo enterohepatic recirculation cause an increase in drug reabsorption. Bowel motility and tone decreases during pregnancy, which causes an increase in intestinal transit time. Therefore the drug absorption increases.

A nurse administers carbidopa-levodopa to a client with Parkinson disease. Which therapeutic effect does the nurse expect the medication to produce? Increase in acetylcholine production Regeneration of injured thalamic cells Improvement in myelination of neurons Replacement of a neurotransmitter in the brain

Replacement of a neurotransmitter in the brain Carbidopa-levodopa is used because levodopa is the precursor of dopamine. It is converted to dopamine in the brain cells, where it is stored until needed by axon terminals; it functions as a neurotransmitter. Regeneration of injured thalamic cells is not an action of this drug; neurons do not regenerate. Increase in acetylcholine production and improvement in myelination of neurons are not actions of this drug.

What should the nurse include in a teaching plan for a client taking calcium channel blockers such as nifedipine? . Reduce calcium intake. Report peripheral edema. Expect temporary hair loss. Avoid drinking grapefruit juice. Change to a standing position slowly.

Report peripheral edema. Avoid drinking grapefruit juice. Change to a standing position slowly. Changing positions slowly helps reduce orthostatic hypotension. Peripheral edema may occur as a result of heart failure and must be reported. Grapefruit juice affects the metabolism of calcium channel blockers and should be avoided. Reducing calcium intake is unnecessary because calcium levels are not affected. Hair loss does not occur.

A neuromuscular blocking agent is administered to a client before electroconvulsive therapy. At this time, the nurse will monitor the client for which adverse effect? Seizures Vomiting Loss of memory Respiratory difficulties

Respiratory difficulties A neuromuscular blocker, such as succinylcholine, produces respiratory depression because it inhibits contractions of respiratory muscles. This medication does not cause seizures. Because the client is not permitted anything by mouth for 8 to 10 hours before the treatment, vomiting is not a major problem. The loss of memory results from the electroconvulsive therapy, not from the neuromuscular blocking agent.

Alprazolam is prescribed for a client who is anxious. For what therapeutic effect will the nurse monitor the client? Reduced anger Resting quietly Sleeping soundly Reduced blood pressure

Resting quietly Alprazolam, an anxiolytic, promotes muscle relaxation, reduces anxiety, and facilitates rest. Possible adverse reactions to alprazolam are anger and hostility. Although drowsiness is a side effect of alprazolam, caused by depression of central nervous system activity, it is not a hypnotic. Transient hypotension is a side effect of alprazolam, but this is not why it is given to an anxious client.

43. A client is receiving external beam radiation to the mediastinum for treatment of bronchial cancer. Which of the following should take priority in planning care? A) Esophagitis B) Leukopenia C) Fatigue D) Skin irritation

Review Information: The correct answer is B: Leukopenia

A client with a family history of diabetes is concerned about the effects of psychiatric medication on the endocrine system. Which psychotropic medication is most likely to cause metabolic syndrome? Lithium Diazepam Alprazolam Risperidone

Risperidone Atypical antipsychotics such as risperidone can cause metabolic syndrome, in which the client experiences weight gain and increases in cholesterol and triglyceride levels. Diabetes mellitus and diabetic ketoacidosis may occur between 5 weeks and 17 months after initiation of therapy. Although lithium, diazepam, and alprazolam may cause weight gain, none causes metabolic syndrome.

A client admitted to the hospital with a diagnosis of chronic obstructive pulmonary disease has received a prescription for a medication that is delivered via a nebulizer. When teaching about use of the nebulizer, the nurse should teach the client to do what? Hold the breath while spraying the medication into the mouth. Position the lips loosely around the mouthpiece and take rapid, shallow breaths. Seal the lips around the mouthpiece and breathe in and out, taking slow, deep breaths. Inhale the medication from the nebulizer, remove the mouthpiece from the mouth, and then exhale.

Seal the lips around the mouthpiece and breathe in and out, taking slow, deep breaths. Sealing the lips around the mouthpiece ensures that medication is delivered on inspiration; slow, deep breaths promote better deposition and efficacy of medication deep into the lungs. The breath should not be held; a nebulizer treatment delivers medication by inhaling it into the mouth through a mouthpiece. Positioning the lips loosely around the mouthpiece may allow room air to be inhaled, which will dilute the aerosolized medication; rapid, shallow breaths mainly will deposit medication in the oral cavity and will not effectively deliver medication deep into the lung. Inhaling the medication from the nebulizer, removing the mouthpiece from the mouth, and then exhaling allows valuable aerosolized medication to be deposited into the air when the client removes the mouthpiece from the mouth to exhale; the client will not receive the full dose of aerosolized medication.

A toddler with cystic fibrosis has been hospitalized with bacterial pneumonia. The nurse determines that the child has no known allergies. What does the nurse conclude is the reason that the healthcare provider selected a specific antibiotic? Tolerance of the child Sensitivity of the bacteria Selectivity of the bacteria Preference of the healthcare provider

Sensitivity of the bacteria When the causative organism is isolated, it is tested for antimicrobial susceptibility (sensitivity) to various antimicrobial agents. When a microorganism is sensitive to a medication, the medication is capable of destroying the microorganism. The tolerance of the child of the particular antibiotic is unknown because up to this time the child has not exhibited any allergies. Bacteria are not selective. Although the healthcare provider may have a preference for a particular antibiotic, it first must be determined whether the bacteria are sensitive to it.

Which drugs are used as the first-line treatment for posttraumatic stress disorder (PTSD)? Sertraline Paroxetine Phenelzine Venlafaxine Amitriptyline

Sertraline Paroxetine Sertraline and paroxetine are selective serotonin reuptake inhibitors that are approved by the Food and Drug Administration as a first-line treatment for posttraumatic stress disorder ( PTSD). If these drugs are ineffective, the use of phenelzine, venlafaxine, and amitriptyline is indicated.

A nurse administers the prescribed regular insulin to a client in diabetic ketoacidosis. In addition, an intravenous (IV) solution with potassium is prescribed even though the serum potassium level is within normal limits. What does the nurse recognize as the reason for potassium administration? Potassium loss occurs rapidly from diaphoresis present during coma. Potassium is carried with glucose to the kidneys to be excreted in the urine in increased amounts. Potassium is quickly used up during the rapid series of catabolic reactions stimulated by insulin and glucose. Serum potassium levels will decrease as potassium ions shift from the extracellular fluid to the intracellular fluid compartment.

Serum potassium levels will decrease as potassium ions shift from the extracellular fluid to the intracellular fluid compartment. Insulin stimulates cellular uptake of glucose and stimulates the sodium/potassium pump, leading to the influx of potassium into cells. The resulting hypokalemia is offset by parenteral administration of potassium. Potassium is not lost from the body by profuse diaphoresis. Potassium moves from the extracellular to the intracellular compartment rather than being excreted in the urine. Anabolic reactions are stimulated by insulin and glucose administration; potassium is drawn into the intracellular compartment, necessitating a replenishment of extracellular potassium.

Which teratogenic effects can be suspected in the newborn if a pregnant client was prescribed cyclophosphamide? Shortened limbs Secondary cancer Internal organ defects Limb malformations Central nervous system malformation

Shortened limbs Internal organ defects Limb malformations Central nervous system malformation Cyclophosphamide is a drug used in the treatment of cancer. It is shown to have teratogenic effect if taken during pregnancy. The teratogenic effect includes shortened limbs, internal organ defects, limb malformations, and central nervous system malformation.

Lithium is prescribed for a client with bipolar disorder experiencing a manic episode. When teaching the client about this medication, the nurse will emphasize which fact about lithium? Cannot be taken safely with any other antipsychotic medications Can be taken safely with diuretics if the potassium level is maintained Should be discontinued and the primary healthcare provider notified if depression occurs Should temporarily be stopped and the primary healthcare provider notified if diarrhea results

Should temporarily be stopped and the primary healthcare provider notified if diarrhea results A decrease in serum sodium because of diarrhea decreases the excretion of lithium, which can result in a toxic level in the blood. In addition, diarrhea is a sign of lithium toxicity. It is safe to take lithium with other antipsychotics, because the client's behavior may require combination pharmacotherapy. Lithium should not be taken with diuretics, regardless of the potassium level, because diuretics decrease the sodium level, and lithium is not excreted when the sodium level is decreased. Lithium is useful in both the manic and depressive phases of a bipolar disorder.

Which statement by the nurse is true for collecting a urine sample in toddlers? A hat is placed under the toilet seat. Urine can be squeezed from the diaper. Force the child to void in the unfamiliar receptacle. Single-use bags can be placed over the child's urethral meatus.

Single-use bags can be placed over the child's urethral meatus. Single-use bags are placed over the child's urethral meatus for collecting urine in toddlers. A potty chair or specimen hat placed under the toilet seat is usually effective in cases of young children. Specimens obtained by squeezing urine from the diaper are not used because the results will be inaccurate. A young child is often reluctant to void in unfamiliar receptacles. They should not be forced to void.

A client with ascites is scheduled to receive albumin. To have the greatest therapeutic effect, the nurse expects what infusion rate and what oral fluid intake? Slow intravenous (IV) rate and liberal fluid intake Slow IV rate and restricted fluid intake Rapid IV rate and withheld fluid intake Rapid IV rate and moderate fluid intake

Slow IV rate and restricted fluid intake When albumin is administered slowly and oral fluid intake is restricted, fluid moves from the interstitial spaces into the circulatory system so it can be eliminated by the kidneys. Administration should not exceed 5 to 10 mL/min. Oral fluids are restricted to facilitate the optimal effects of the albumin, which shifts fluids from the interstitial spaces to the intravascular compartment. Rapid administration may cause circulatory overload; fluid is restricted, not withheld. Unrestricted fluid intake will limit the shift of fluid from the interstitial to the intravascular compartment, interfering with the optimal effects of the albumin.

A client with heart failure is to receive digoxin and asks the nurse why the medication is necessary. What physiologic response will the nurse include when answering the client's question? Reduces edema Increases cardiac conduction Increases rate of ventricular contractions Slows and strengthens cardiac contractions

Slows and strengthens cardiac contractions Digoxin increases the strength of myocardial contractions (positive inotropic effect) and, by altering the electrophysiological properties of the heart, slows the heart rate (negative chronotropic effect). Digoxin increases the strength of the contractions but decreases the heart rate. Although a reduction in edema may result from the increased blood supply to the kidneys, it is not the reason for administering digoxin. Digoxin decreases, not increases, cardiac impulses through the conduction system of the heart.

A client's cells are deprived of oxygen during a cardiac arrest. What medication should the nurse be prepared to administer? Regular insulin Calcium gluconate Potassium chloride Sodium bicarbonate

Sodium bicarbonate In the absence of oxygen, the body derives its energy anaerobically; this results in buildup of lactic acid. Sodium bicarbonate, an alkaline drug, will help neutralize the acid, raising the pH. Insulin is used to treat diabetes; it lowers blood sugar by facilitating transport of glucose across cell membranes. Calcium gluconate is used to treat hypocalcemia. Although potassium is essential for cardiac function, it will not correct acidosis. With acidosis, serum hydrogen ions will exchange with intracellular potassium, leading to a temporary hyperkalemic state; therefore, potassium chloride is contraindicated until acidosis is corrected.

A nurse on the Code Blue/Arrest team responds to a code that is called for a client with hyperkalemia who is experiencing cardiac standstill. What would an appropriate immediate treatment plan include? Defibrillation Furosemide Sodium bicarbonate Anticoagulation therapy

Sodium bicarbonate Sodium bicarbonate decreases the potassium level. It works by increasing the movement of potassium from the blood into the cells. The body stores potassium inside the cells, with only a small amount in the bloodstream. Electrical defibrillation should not be applied indiscriminately to the patient in asystole. This is not only fruitless but also detrimental, eliminating any possibility of recovering a rhythm. Furosemide is a diuretic commonly used for heart failure. There is no indication for anticoagulation therapy.

After an infant undergoes surgery for the repair of a myelomeningocele, diarrhea and metabolic acidosis develop, accompanied by decreased urine output. In light of the infant's status, what prescription does the nurse anticipate? Isotonic saline Sodium lactate Serum albumin Potassium chloride

Sodium lactate Sodium lactate is converted to sodium bicarbonate; it helps correct the sodium deficiency and the metabolic acidosis. Normal saline solution results in the combination of the chloride with the hydrogen ion, intensifying the acidosis. Albumin is a colloid found in blood plasma; it is not used in the treatment of metabolic acidosis. Potassium is not administered until urine function has been restored.

A client with a history of a pulmonary embolus is to receive 3 mg of warfarin daily. The client has blood drawn twice weekly to ascertain that the international normalized ratio (INR) stays within a therapeutic range. The nurse provides dietary teaching. Which food selected by the client indicates that further teaching is necessary? Poached eggs Spinach salad Sweet potatoes Cheese sandwich

Spinach salad Dark green, leafy vegetables are high in vitamin K. Influencing the level of vitamin K alters the activity of warfarin because vitamin K acts as a catalyst in the liver for the production of blood-clotting factors and prothrombin. The intake of foods containing vitamin K must be consistent to regulate the warfarin dose so that the INR remains within the therapeutic range. Eggs contain protein and are permitted on the diet. Yellow vegetables contain vitamin A and are permitted on the diet. Dairy products containing protein and bread supplying carbohydrates are permitted on the diet.

Which medication requires the nurse to monitor the client for signs of hyperkalemia? Furosemide Metolazone Spironolactone Hydrochlorothiazide

Spironolactone Spironolactone is a potassium-sparing diuretic; hyperkalemia is an adverse effect. Furosemide, metolazone, and hydrochlorothiazide generally cause hypokalemia.

After surgery a client develops a deep vein thrombosis and a pulmonary embolus. Heparin via a continuous drip is prescribed. Several hours later, vancomycin intravenously every 12 hours is prescribed. The client has one intravenous (IV) site: a peripheral line in the left forearm. What action should the nurse take? Stop the heparin, flush the line, and administer the vancomycin. Use a piggyback setup to administer the vancomycin into the heparin. Start another IV line for the vancomycin and continue the heparin as prescribed. Hold the vancomycin and tell the healthcare provider that the drug is incompatible with heparin.

Start another IV line for the vancomycin and continue the heparin as prescribed. The vancomycin and heparin are incompatible in the same IV and therefore must be administered separately. By instituting a second line for the antibiotic, heparin can continue to infuse. Twice a day both drugs must run concurrently. Also, flushing the line may not eliminate remnants of the heparin, which is incompatible with vancomycin. Using a piggyback setup to administer the vancomycin into the heparin is unsafe because heparin and vancomycin are incompatible and should not be administered via the same intravenous line. The client has two medications prescribed, and it is a nurse's responsibility, not the healthcare provider's, to administer them safely.

A client is scheduled for a craniotomy to remove a brain tumor. To prevent the development of cerebral edema after surgery, the nurse anticipates the use of drugs from which class? Steroids Diuretics Anticonvulsants Antihypertensives

Steroids Glucocorticoids are used for their antiinflammatory action, which decreases the development of cerebral edema. Diuretics may be used in conjunction with steroids; they reduce edema after it is present. Anticonvulsants prevent seizure activity, not cerebral edema. Antihypertensives control hypertension, not cerebral edema.

A nurse administers two serial intramuscular injections of betamethasone to a woman at 32 weeks' gestation who has been admitted in preterm labor. The nurse knows that this medication is given to accomplish what purpose? Stop the process of labor. Increase placental perfusion. Stimulate surfactant production. Reduce intensity of contractions.

Stimulate surfactant production. Corticosteroids stimulate surfactant production; they also have been shown to reduce the incidence of intraventricular hemorrhage. Betamethasone does not affect the labor process, increase placental perfusion, or affect the intensity of contractions.

A healthcare provider prescribes steroid therapy for a 4-year-old child who has nephrotic syndrome. What goal of this treatment does the nurse explain to the child's parents? Prevents infection Stimulates diuresis Provides hemopoiesis Reduces blood pressure

Stimulates diuresis Although the exact mechanism is unknown, steroids produce diuresis in most children with nephrotic syndrome. Steroids will not prevent infection and will mask the signs and symptoms of infection. Steroids have no effect on the production of red blood cells. Steroids do not reduce hypertension, and hypertension is not a common finding in children with nephrotic syndrome.

A client with hypertension is to take an angiotensin II receptor blocker (ARB). What should the nurse teach about this medication? Monitor the blood pressure daily. Stop treatment if a cough develops. Stop the medication if swelling of the mouth, lips, or face develops. Have blood drawn for potassium levels 2 weeks after starting the medication. Do not take nonsteroidal antiinflammatory drugs (NSAIDs) concurrently with this medication.

Stop the medication if swelling of the mouth, lips, or face develops. Have blood drawn for potassium levels 2 weeks after starting the medication. The medication should be stopped if angioedema occurs, and the healthcare provider should be notified. Electrolyte levels of potassium, sodium, and chloride should be obtained 2 weeks after the start of therapy and then periodically thereafter. Daily monitoring is not indicated. The blood pressure should be monitored at routine office visits. There is no need to avoid the use of NSAIDs while taking an ARB. A dry cough may occur during treatment with ARBs; however, it is not necessary to discontinue the medication because the cough usually resolves.

A nurse is monitoring a client with severe preeclampsia who is receiving an infusion of magnesium sulfate. Assessment reveals a pulse rate of 55 beats/min, a respiratory rate of 10 breaths/min, and a flushed face. What are the next nursing actions? Continuing the infusion and notifying the primary healthcare provider Stopping the infusion and starting an infusion of dextrose and water Continuing the infusion and documenting the findings in the clinical record Decreasing the rate of the infusion and obtaining blood for a magnesium level

Stopping the infusion and starting an infusion of dextrose and water The client's slow pulse, respirations, and flushed face are signs of magnesium sulfate toxicity. The infusion should be stopped and the intravenous site maintained with an infusion of dextrose 5% in water because an antagonist (calcium gluconate) may be prescribed. Continuing the infusion and notifying the primary healthcare provider is unsafe because continuing the infusion will make the central nervous system (CNS) depression more severe. The primary healthcare provider should be notified after the infusion has been stopped. Continuing the infusion and documenting the findings in the clinical record are unsafe; the client's clinical manifestations indicate a life-threatening condition. It is unsafe to decrease the rate of the infusion because the CNS depression will worsen. Blood for determination of the magnesium level should be obtained, but not before the infusion is stopped.

A nurse is caring for a client with pulmonary tuberculosis who is to receive several antitubercular medications. Which of the first-line antitubercular medications is associated with damage to the eighth cranial nerve? Isoniazid Rifampin Streptomycin Ethambutol

Streptomycin Streptomycin is ototoxic and can cause damage to the eighth cranial nerve, resulting in deafness. Assessment for ringing or roaring in the ears, vertigo, and hearing acuity should be made before, during, and after treatment. Isoniazid does not affect the ear; however, blurred vision and optic neuritis, as well as peripheral neuropathy, may occur. Rifampin does not affect hearing; however, visual disturbances may occur. Ethambutol does not affect hearing; however, visual disturbances may occur.

A newborn who is receiving drug therapy for an infection has developed jaundice. The newborn has poor sucking and extreme sleepiness. Which drug may be responsible for the newborn's condition? Tetracycline Sulfonamide Phenothiazine Chloramphenicol

Sulfonamide Jaundice, poor sucking, and extreme sleepiness are the clinical manifestations of kernicterus. Kernicterus is a condition seen in newborns due to deposition of bilirubin in the brain and is caused by sulfonamides. Tetracycline can cause staining of developing teeth in pediatric clients. Phenothiazine may cause sudden infant death syndrome. Chloramphenicol may cause Gray syndrome in pediatric clients.

Which drug class may cause kernicterus in neonates? Salicylates Tetracyclines Sulfonamides Glucocorticoids

Sulfonamides Sulfonamides may cause kernicterus in neonates. Salicylates may cause Reye syndrome. Tetracyclines may cause the discoloration of developing teeth. Glucocorticoids may cause growth suppression.

What are the side effects of oral psoralen in phototherapy? Atrophy Sunburn Mucositis Occular damage Persistent pruritus

Sunburn Persistent pruritus Oral psoralen is one form of phototherapy used in the treatment of many dermatologic conditions. Sunburn and persistent pruritus are side effects of oral psoralen. Atrophy, mucositis, and ocular damage are the adverse reactions of radiation therapy.

A nurse is caring for a first-grader receiving prednisone. What outcome does the nurse expect with adrenocorticosteroid therapy? Accelerated wound healing Development of hyperkalemia Increased antibody production Suppressed inflammatory process

Suppressed inflammatory process Because of the suppression of the inflammatory process, the nurse must be alert to the subtle symptoms of infection, such as changes in appetite, sleep patterns, and behavior. Adrenocorticosteroid therapy delays, not accelerates, wound healing. Adrenocorticosteroid therapy may cause hypokalemia, not hyperkalemia, because of the accompanying retention of sodium and fluid. Adrenocorticosteroid therapy decreases, not increases, the production of antibodies.

A client with hyperthyroidism is treated initially with propylthiouracil (PTU). What should the nurse include when teaching the client about this medication? This medication will have to be taken for the remainder of the client's life. Milk should be taken with the medication so that gastric irritation does not occur. The medication should be taken between meals so that it is more readily absorbed. Symptoms may not subside until the client has taken the medication for several weeks.

Symptoms may not subside until the client has taken the medication for several weeks. This drug does not interfere with thyroxine already stored in the gland; symptoms remain until the hormone is depleted. Duration of therapy varies depending on the severity of the disease and the client's response to therapy. Milk does not need to be taken because this drug is not irritating to mucosal tissue, and no special precautions are necessary. Absorption is not affected by the presence of food in the stomach.

On reviewing the medical history, the nurse finds that the client is prescribed mycophenolate mofetil for late graft loss. Which other medications could also be co-administered? Tacrolimus Cyclosporine Acetaminophen Diphenhydramine Methylprednisolone

Tacrolimus Cyclosporine Mycophenolate is a lymphocyte-specific inhibitor of purine synthesis with suppressive effects on both T and B lymphocytes. Tacrolimus and cyclosporine are co-administered with mycophenolate as it shows additive effect because it acts later in the lymphocyte activation pathway by a different mechanism, reducing late graft loss. Acetaminophen, diphenhydramine, and methylprednisolone are administered to reduce the expected adverse effects such as headache, myalgia, and various gastrointestinal disturbances that occur with monoclonal antibodies such as muromonab-CD3.

The primary healthcare provider prescribes medication to prevent transplant rejection from increased concentration of interleukin-2. Which drugs could the nurse most likely administer? . Sirolimus Tacrolimus Cyclosporine Azathioprine Mycophenolate

Tacrolimus Cyclosporine Tacrolimus and cyclosporine are calcineurin inhibitors that stop the production and secretion of interleukin-2 and prevent the activation of lymphocytes involved in transplant rejection. Therefore tacrolimus and cyclosporine are most suitable for the client. Sirolimus, azathioprine, and mycophenolate are antiproliferative medications that inhibit something essential to DNA synthesis and prevent cell division in activated lymphocytes; thus they are effective for lessening transplant rejection involving DNA synthesis, not interleukin-2.

Potassium chloride effervescent tablets are prescribed for a client who is to be discharged from the emergency department. What information should the nurse include when teaching the client about this medication? Chew the tablet completely. Take the medication with food. Take the medication at bedtime. Use warm water to dissolve the tablet.

Take the medication with food. Eating food when taking the medication will decrease gastrointestinal irritation. Side effects of this medication include abdominal cramps, diarrhea, and ulceration of the small intestine. Chewing the tablet completely will cause oral mucosal irritation and is not the way the drug should be administered if it is to be most effective. Taking the medication at bedtime increases the possibility of mucosal irritation because the gastrointestinal tract is empty during the night. The tablet should be dissolved in cold water or juice to make it more palatable.

A healthcare provider prescribes lithium carbonate for a client with bipolar disorder, depressive episode. What instructions will the nurse include when teaching the client about lithium? Take the medication with food. Adjust the dosage if your mood improves. Have a snack with milk before going to bed. It may take several weeks for beneficial results to occur. You do not have to restrict your intake of dietary sodium.

Take the medication with food. It may take several weeks for beneficial results to occur. You do not have to restrict your intake of dietary sodium. Lithium should be taken with food to prevent gastric irritation. It will take 1 to 3 weeks before beneficial results occur. Lithium decreases sodium resorption by the renal tubules. If sodium intake is decreased, sodium depletion may occur. In addition, lithium retention is increased when sodium intake is decreased; restricted sodium intake can lead to lithium toxicity. The dosage should not be adjusted without healthcare provider supervision. It is not necessary to have a snack with milk when the client goes to bed.

A healthcare provider prescribes aspirin to be continued at home for a client with severe arthritis. What should the nurse teach the client about taking aspirin? Take the medicine with meals. See a dentist if bleeding gums develop. Switch to acetaminophen if tinnitus occurs. Avoid spicy foods while taking the medication.

Take the medicine with meals. Acetylsalicylic acid is irritating to the stomach lining and can cause ulceration; the presence of food, fluid, or antacids decreases this response. Bleeding gums should be reported to the healthcare provider, not the dentist. Acetaminophen does not contain the antiinflammatory properties present in aspirin; tinnitus should be reported to the healthcare provider. Avoiding spicy foods is unnecessary as long as aspirin is taken with food.

A client with the diagnosis of schizophrenia is given one of the antipsychotic drugs. The nurse understands that antipsychotic drugs can cause extrapyramidal side effects. Which effect may be irreversible? Akathisia Tardive dyskinesia Parkinsonian syndrome Acute dystonic reaction

Tardive dyskinesia Tardive dyskinesia, an extrapyramidal response characterized by vermicular movements and protrusion of the tongue, chewing and puckering movements of the mouth, and puffing of the cheeks, is often irreversible, even when the antipsychotic medication is withdrawn. Akathisia (motor restlessness), parkinsonian syndrome (a disorder featuring signs and symptoms of Parkinson disease such as resting tremors, muscle weakness, reduced movement, and festinating gait), and dystonia (impairment of muscle tonus) can usually be treated with antiparkinsonian or anticholinergic drugs while the antipsychotic medication is continued.

A 5-year-old child is given fluoroquinolones. Which potential adverse effect unique to pediatric clients should the nurse anticipate? Tendon rupture Cartilage erosion Staining of developing teeth Central nervous system toxicity

Tendon rupture Fluoroquinolones may cause tendon rupture in children. Nalidixic acid can cause cartilage erosion, and tetracycline can cause staining of developing teeth. Hexachlorophene may cause central nervous system toxicity in infants.

Which points should be included when counseling a woman who is on hormonal therapy? The client should use appropriate sun protection. The client should monitor any deviations in body weight. The client should take the medication on an empty stomach. The client should discontinue the medication if adverse effects occur. The client should take oral medication at the same time every day.

The client should use appropriate sun protection. The client should monitor any deviations in body weight. The client should take oral medication at the same time every day. Appropriate sun protection should be used because hormones make people sensitive to sunlight. The client's body weight should be monitored during hormonal therapy because abnormal bleeding can lead to weight loss and serious complications. Oral drugs should be administered at the same time every day to maintain the appropriate concentration of serum drug levels. Hormonal drugs should be taken with food to reduce gastrointestinal upset. The client should report any side effects to the primary healthcare provider and seek his advice. Hormonal therapy should never be discontinued without the knowledge of the primary healthcare provider.

74. A nurse is assessing several clients in a long term health care facility. Which client is at highest risk for development of decubitus ulcers? A) A 79 year-old malnourished client on bed rest B) An obese client who uses a wheelchair C) A client who had 3 incontinent diarrhea stools D) An 80 year-old ambulatory diabetic client

The correct answer is A: A 79 year-old malnourished client on bed rest

94. A nurse who is reassigned to the emergency department needs to understand that gastric lavage is a priority in which situation? A) An infant who has been identified to have botulism B) A toddler who ate a number of ibuprofen tablets C) A preschooler who swallowed powdered plant food D) A school aged child who took a handful of vitamins

The correct answer is A: An infant who has been identified to have botulism

90. An 85 year-old client complains of generalized muscle aches and pains. The first action by the nurse should be A) Assess the severity and location of the pain B) Obtain an order for an analgesic C) Reassure him that this is not unusual for his age D) Encourage him to increase his activity

The correct answer is A: Assess the severity and location of the pain

61. The nurse is caring for a client with clinical depression who is receiving a MAO inhibitor. When providing instructions about precautions with this medication, which action should the nurse stress to the client as important? A) Avoid chocolate and cheese B) Take frequent naps C) Take the medication with milk D) Avoid walking without assistance

The correct answer is A: Avoid chocolate and cheese

14. A client with heart failure has a prescription for digoxin. The nurse is aware that sufficient potassium should be included in the diet because hypokalemia in combination with this medication A) Can predispose to dysrhythmias B) May lead to oliguria C) May cause irritability and anxiety D) Sometimes alters consciousness

The correct answer is A: Can predispose to dysrhythmias

78. The nurse is instructing a 65 year-old female client diagnosed with osteoporosis. The most important instruction regarding exercise would be to A) Exercise doing weight bearing activities B) Exercise to reduce weight C) Avoid exercise activities that increase the risk of fracture D) Exercise to strengthen muscles and thereby protect bones

The correct answer is A: Exercise doing weight bearing activities

27. A nurse is providing care to a primigravida whose membranes spontaneously ruptured (ROM) 4 hours ago. Labor is to be induced. At the time of the ROM the vital signs were T-99.8 degrees F, P-84, R-20, BP-130/78, and fetal heart tones (FHT) 148 beats/min. Which assessment findings taken now may be an early indication that the client is developing a complication of labor? A) FHT 168 beats/min B) Temperature 100 degrees Fahrenheit. C) Cervical dilation of 4 D) BP 138/88

The correct answer is A: FHT 168 beats/min

49. The health care provider order reads "aspirate nasogastric feeding (NG) tuber every 4 hours and check pH of aspirate." The pH of the aspirate is 10. Which action should the nurse take? A) Hold the tube feeding and notify the provider B) Administer the tube feeding as scheduled C) Irrigate the tube with diet cola soda D) Apply intermittent suction to the feeding tube

The correct answer is A: Hold the tube feeding and notify the provider

63. The nurse is teaching a client about precautions with Coumadin therapy. The client should be instructed to avoid which over-the-counter medication? A) Non-steroidal anti-inflammatory drugs B) Cough medicines with guaifenesin C) Histamine blockers D) Laxatives containing magnesium salts

The correct answer is A: Non-steroidal anti inflammatory drugs

41. A 4 year-old has been hospitalized for 24 hours with skeletal traction for treatment of a fracture of the right femur. The nurse finds that the child is now crying and the right foot is pale with the absence of a pulse. What should the nurse do first? A) Notify the health care provider B) Read just the traction C) Administer the ordered prn medication D) Reassess the foot in fifteen minutes

The correct answer is A: Notify the health care provider

83. A client with diarrhea should avoid which of the following? A) Orange juice B) Tuna C) Eggs D) Macaroni

The correct answer is A: Orange juice

70. Why is it important for the nurse to monitor blood pressure in clients receiving antipsychotic drugs? A) Orthostatic hypotension is a common side effect B) Most antipsychotic drugs cause elevated blood pressure C) This provides information on the amount of sodium allowed in the diet D) It will indicate the need to institute anti parkinsonian drugs

The correct answer is A: Orthostatic hypotension is a common side effect

68. A client is receiving intravenous heparin therapy. What medication should the nurse have available in the event of an overdose of heparin? A) Protamine B) Amicar C) Imferon D) Diltiazem

The correct answer is A: Protamine . Protamine binds heparin making it ineffective.

29. The nurse is performing an assessment on a client in congestive heart failure. Auscultation of the heart is most likely to reveal A) S3 ventricular gallop B) Apical click C) Systolic murmur D) Split S2

The correct answer is A: S3 ventricular gallop

10. A client has viral pneumonia affecting 2/3 of the right lung. What would be the best position to teach the client to lie in every other hour during first 12 hours after admission? A) Side-lying on the left with the head elevated 10 degrees B) Side-lying on the left with the head elevated 35 degrees C) Side-lying on the right with the head elevated 10 degrees D) Side-lying on the right with the head elevated 35 degrees

The correct answer is A: Side-lying on the left with the head elevated 10 degrees

26. An elderly client admitted after a fall begins to seize and loses consciousness. What action by the nurse is appropriate to do next? A) Stay with client and observe for airway obstruction B) Collect pillows and pad the side rails of the bed C) Place an oral airway in the mouth and suction D) Announce a cardiac arrest, and assist with intubation

The correct answer is A: Stay with client and observe for airway obstruction

65. The nurse is caring for a client receiving a blood transfusion who develops urticaria one-half hour after the transfusion has begun. What is the first action the nurse should take? A) Stop the infusion B) Slow the rate of infusion C) Take vital signs and observe for further deterioration D) Administer Benadryl and continue the infusion

The correct answer is A: Stop the infusion

7. A client who had a vasectomy is in the post recovery unit at an outpatient clinic. Which of these points is most important to be reinforced by the nurse? A) Until the health care provider has determined that your ejaculate doesn't contain sperm, continue to use another form of contraception. B)This procedure doesn't impede the production of male hormones or the production of sperm in the testicles. The sperm can no longer enter your semen and no sperm are in your ejaculate. C) After your vasectomy, strenuous activity needs to be avoided for at least 48 hours. If your work doesn't involve hard physical labor, you can return to your job as soon as you feel up to it. The stitches generally dissolve in seven to ten days. D)The health care provider at this clinic recommends rest, ice, an athletic supporter or over-the-counter pain medication to relieve any discomfort.

The correct answer is A: Until the health care provider has determined that your ejaculate doesn't contain sperm, continue to use another form of contraception.

99. During the care of a client with a salmonella infection, the primary nursing intervention to limit transmission is which of these approaches? A) Wash hands thoroughly before and after client contact B) Wear gloves when in contact with body secretions C) Double glove when in contact with feces or vomitus D) Wear gloves when disposing of contaminated linens

The correct answer is A: Wash hands thoroughly before and after client contact

51. An antibiotic IM injection for a 2 year-old child is ordered. The total volume of the injection equals 2.0 ml The correct action is to A) administer the medication in 2 separate injections B) give the medication in the dorsal gluteal site C) call to get a smaller volume ordered D) check with pharmacy for a liquid form of the medication skip

The correct answer is A: administer the medication in 2 separate injections

48. The nurse is preparing a client who will undergo a myelogram. Which of the following statements by the client indicates a contraindication for this test? A) "I can't lie in 1 position for more than thirty minutes." B) "I am allergic to shrimp." C) "I suffer from claustrophobia." D) "I developed a severe headache after a spinal tap."

The correct answer is B: "I am allergic to shrimp."

59. The nurse has given discharge instructions to parents of a child on phenytoin (Dilantin). Which of the following statements suggests that the teaching was effective? A) "We will call the health care provider if the child develops acne." B) "Our child should brush and floss carefully after every meal." C) "We will skip the next dose if vomiting or fever occur." D) "When our child is seizure-free for 6 months, we can stop the medication."

The correct answer is B: "Our child should brush and floss carefully after every meal."

31. A client is diagnosed with a spontaneous pneumothorax necessitating the insertion of a chest tube. What is the best explanation for the nurse to provide this client? A) "The tube will drain fluid from your chest." B) "The tube will remove excess air from your chest." C) "The tube controls the amount of air that enters your chest." D) "The tube will seal the hole in your lung."

The correct answer is B: "The tube will remove excess air from your chest."

22. While caring for a client who was admitted with myocardial infarction (MI) 2 days ago, the nurse notes today's temperature is 101.1 degrees Fahrenheit (38.5 degrees Celsius). The appropriate nursing intervention is to A) Call the health care provider immediately B) Administer acetaminophen as ordered as this is normal at this time C) Send blood, urine and sputum for culture D) Increase the client's fluid intake

The correct answer is B: Administer acetaminophen as ordered as this is normal at this time

96. Which of these clients with associated lab reports is a priority for the nurse to report to the public health department within the next 24 hours? A) An infant with a positive culture of stool for Shigella B) An elderly factory worker with a lab report that is positive for acid-fast bacillus smear C) A young adult commercial pilot with a positive histopathological examination from an induced sputum for Pneumocystis carinii D) A middle-aged nurse with a history of varicella-zoster virus and with crops of vesicles on an erythematous base that appear on the skin

The correct answer is B: An elderly factory worker with a lab report that is positive for acid-fast bacillus smear

23. A client is admitted for first and second degree burns on the face, neck, anterior chest and hands. The nurse's priority should be A) Cover the areas with dry sterile dressings B) Assess for dyspnea or stridor C) Initiate intravenous therapy D) Administer pain medication

The correct answer is B: Assess for dyspnea or stridor

39. The nurse is caring for a child immediately after surgical correction of a ventricular septal defect. Which of the following nursing assessments should be a priority? A) Blanch nail beds for color and refill B) Assess for post operative arrhythmias C) Auscultate for pulmonary congestion D) Monitor equality of peripheral pulses

The correct answer is B: Assess for post operative arrhythmias

36. The most effective nursing intervention to prevent atelectasis from developing in a post operative client is to A) Maintain adequate hydration B) Assist client to turn, deep breathe, and cough C) Ambulate client within 12 hours D) Splint incision

The correct answer is B: Assist client to turn, deep breathe, and cough

72. An 86 year-old nursing home resident who has decreased mental status is hospitalized with pneumonic infiltrates in the right lower lobe. When the nurse assists the client with a clear liquid diet, the client begins to cough. What should the nurse do next? A) Add a thickening agent to the fluids B) Check the client's gag reflex C) Feed the client only solid foods D) Increase the rate of intravenous fluids

The correct answer is B: Check the client's gag reflex

87. The nurse is caring for a 7 year-old with acute glomerulonephritis (AGN). Findings include moderate edema and oliguria. Serum blood urea nitrogen and creatinine are elevated. What dietary modifications are most appropriate? A) Decreased carbohydrates and fat B) Decreased sodium and potassium C) Increased potassium and protein D) Increased sodium and fluids

The correct answer is B: Decreased sodium and potassium

37. When caring for a client with a post right thoracotomy who has undergone an upper lobectomy, the nurse focuses on pain management to promote A) Relaxation and sleep B) Deep breathing and coughing C) Incisional healing D) Range of motion exercises

The correct answer is B: Deep breathing and coughing

4. A primigravida in the third trimester is hospitalized for preeclampsia. The nurse determines that the client's blood pressure is increasing. Which action should the nurse take first? A) Check the protein level in urine B) Have the client turn to the left side C) Take the temperature D) Monitor the urine output

The correct answer is B: Have the client turn to the left side

67. A client has received 2 units of whole blood today following an episode of GI bleeding. Which of the following laboratory reports would the nurse monitor most closely? A) Bleeding time B) Hemoglobin and hematocrit C) White blood cells D) Platelets

The correct answer is B: Hemoglobin and hematocrit

84. Which statement best describes the effects of immobility in children? A) Immobility prevents the progression of language and fine motor development B) Immobility in children has similar physical effects to those found in adults C) Children are more susceptible to the effects of immobility than are adults D) Children are likely to have prolonged immobility with subsequent complications

The correct answer is B: Immobility in children has similar physical effects to those found in adults

3. A client has been diagnosed with Zollinger-Ellison syndrome.Which information is most important for the nurse to reinforce with the client? A)It is a condition in which one or more tumors called gastrinomas form in the pancreas or in the upper part of the small intestine (duodenum) B)It is critical to report promptly to your health care provider any findings of peptic ulcers c)Treatment consists of medications to reduce acid and heal any peptic ulcers and, if possible, surgery to remove any tumors D)With the average age at diagnosis at 50 years the peptic ulcers may occur at unusual areas of the stomach or intestine

The correct answer is B: It is critical to report promptly to your health care provider any findings of peptic ulcers.

13. The nurse assesses a 72 year-old client who was admitted for right sided congestive heart failure. Which of the following would the nurse anticipate finding? A) Decreased urinary output B) Jugular vein distention C) Pleural effusion D) Bibasilar crackles

The correct answer is B: Jugular vein distention

75. Constipation is one of the most frequent complaints of elders. When assessing this problem, which action should be the nurse's priority? A) Obtain a complete blood count B) Obtain a health and dietary history C) Refer to a provider for a physical examination D) Measure height and weight

The correct answer is B: Obtain a health and dietary history

88. What nursing assessment of a paralyzed client would indicate the probable presence of a fecal impaction? A) Presence of blood in stools B) Oozing liquid stool C) Continuous rumbling flatulence D) Absence of bowel movements

The correct answer is B: Oozing liquid stool

17. Which these findings would the nurse more closely associate with anemia in a 10 month-old infant? A) Hemoglobin level of 12 g/dI B) Pale mucosa of the eyelids and lips C) Hypoactivity D) A heart rate between 140 to 160

The correct answer is B: Pale mucosa of the eyelids and lips

47. The nurse is caring for a client who requires a mechanical ventilator for breathing. The high pressure alarm goes off on the ventilator. What is the first action the nurse should perform? A) Disconnect the client from the ventilator and use a manual resuscitation bag B) Perform a quick assessment of the client's condition C) Call the respiratory therapist for help D) Press the alarm re-set button on the ventilator

The correct answer is B: Perform a quick assessment of the client''s condition

64. A client diagnosed with cirrhosis of the liver and ascites is receiving Spironolactone (Aldactone). The nurse understands that this medication spares elimination of which element? A) Sodium B) Potassium C) Phosphate D) Albumin

The correct answer is B: Potassium

15. A nurse assesses a young adult in the emergency room following a motor vehicle accident. Which of the following neurological signs is of most concern? A) Flaccid paralysis B) Pupils fixed and dilated C) Diminished spinal reflexes D) Reduced sensory responses

The correct answer is B: Pupils fixed and dilated

79. The nurse has been teaching a client with congestive heart failure about proper nutrition. The selection of which lunch indicates the client has learned about sodium restriction? A)Cheese sandwich with a glass of 2% milk B) Sliced turkey sandwich and canned pineapple C) Cheeseburger and baked potato D) Mushroom pizza and ice cream

The correct answer is B: Sliced turkey sandwich and canned pineapple

55. A client is being discharged with a prescription for chlorpromazine (Thorazine). Before leaving for home, which of these findings should the nurse teach the client to report? A) Change in libido, breast enlargement B) Sore throat, fever C) Abdominal pain, nausea, diarrhea D) Dsypnea, nasal congestion

The correct answer is B: Sore throat, fever

66. Discharge instructions for a client taking alprazolam (Xanax) should include which of the following? A) Sedative hypnotics are effective analgesics B) Sudden cessation of alprazolam (Xanax) can cause rebound insomnia and nightmares C) Caffeine beverages can increase the effect of sedative hypnotics D) Avoidance of excessive exercise and high temperature is recommended

The correct answer is B: Sudden cessation of alprazolam

30. Which of these observations made by the nurse during an excretory urogram indicate a complication? A) The client complains of a salty taste in the mouth when the dye is injected B) The client's entire body turns a bright red color C) The client states "I have a feeling of getting warm." D) The client gags and complains " I am getting sick."

The correct answer is B: The client's entire body turns a bright red color

100. A nurse is reinforcing teaching with a client about compromised host precautions. The client is receiving filgrastim (Neupogen) for neutropenia. The selection of which lunch suggests the client has learned about necessary dietary changes? A) grilled chicken sandwich and skim milk B) roast beef, mashed potatoes, and green beans C) peanut butter sandwich, banana, and iced tea D) barbecue beef, baked beans, and cole slaw

The correct answer is B: roast beef, mashed potatoes, and green beans

11. A client has an indwelling catheter with continuous bladder irrigation after undergoing a transurethral resection of the prostate (TURP) 12 hours ago. Which finding at this time should be reported to the health care provider? A) Light, pink urine B) occasional suprapubic cramping C) minimal drainage into the urinary collection bag D) complaints of the feeling of pulling on the urinary catheter

The correct answer is C: minimal drainage into the urinary collection bag

98. The school nurse is teaching the faculty the most effective methods to prevent the spread of lice in the school. The information that would be most important to include would be which of these statements? A) "The treatment requires reapplication in 8 to 10 days." B) "Bedding and clothing can be boiled or steamed." C) Children are not to share hats, scarves and combs. D) Nit combs are necessary to comb out nits.

The correct answer is C: "Children are not to share hats, scarves and combs."

54. While providing home care to a client with congestive heart failure, the nurse is asked how long diuretics must be taken. What is the nurse's best response? A) "As you urinate more, you will need less medication to control fluid." B) "You will have to take this medication for about a year." C) "The medication must be continued so the fluid problem is controlled." D) "Please talk to your health care provider about medications and treatments."

The correct answer is C: "The medication must be continued so the fluid problem is controlled."

5. The nurse is caring for a client in atrial fibrillation. The atrial heart rate is 250 and the ventricular rate is controlled at 75. Which of the following findings is cause for the most concern? A) Diminished bowel sounds B) Loss of appetite C) A cold, pale lower leg D) Tachypnea

The correct answer is C: A cold, pale lower leg

89. A client in a long term care facility complains of pain. The nurse collects data about the client's pain. The first step in pain assessment is for the nurse to A) have the client identify coping methods B) get the description of the location and intensity of the pain C) accept the client's report of pain D) determine the client's status of pain

The correct answer is C: Accept the client''s report of pain

57. A client is being maintained on heparin therapy for deep vein thrombosis. The nurse must closely monitor which of the following laboratory values? A) Bleeding time B) Platelet count C) Activated PTT D) Clotting time

The correct answer is C: Activated PTT

46. A 60 year-old male client had a hernia repair in an outpatient surgery clinic. He is awake and alert, but has not been able to void since he returned from surgery 6 hours ago. He received 1000 mL of IV fluid. Which action would be most likely to help him void? A) Have him drink several glasses of water B) Crede' the bladder from the bottom to the top C) Assist him to stand by the side of the bed to void D) Wait 2 hours and have him try to void again

The correct answer is C: Assist him to stand by the side of the bed to void

33. The nurse is caring for a client undergoing the placement of a central venous catheter line. Which of the following would require the nurse's immediate attention? A) Pallor B) Increased temperature C) Dyspnea D) Involuntary muscle spasms

The correct answer is C: Dyspnea

9. The nurse is discussing with a group of students the disease Kawasaki. What statement made by a student about Kawasaki disease is incorrect? A)It also called mucocutaneous lymph node syndrome because it affects the mucous membranes (inside the mouth, throat and nose), skin and lymph nodes. B)In the second phase of the disease, findings include peeling of the skin on the hands and feet with joint and abdominal pain C)Kawasaki disease occurs most often in boys, children younger than age 5 and children of Hispanic descent D)Initially findings are a sudden high fever, usually above 104 degrees Fahrenheit, which lasts 1 to 2 weeks

The correct answer is C: Kawasaki disease occurs most often in boys, children younger than age 5 and children of Hispanic descent

85. A nurse is providing care to a 63 year-old client with pneumonia. Which intervention promotes the client's comfort? A) Increase oral fluid intake B) Encourage visits from family and friends C) Keep conversations short D) Monitor vital signs frequently

The correct answer is C: Keep conversations short

82. The nurse is teaching an 87 year-old client methods for maintaining regular bowel movements. The nurse would caution the client to AVOID A) Glycerine suppositories B) Fiber supplements C) Laxatives D) Stool softeners

The correct answer is C: Laxatives

45. A client has returned from a cardiac catheterization. Which one of the following assessments would indicate the client is experiencing a complication from the procedure? A) Increased blood pressure B) Increased heart rate C) Loss of pulse in the extremity D) Decreased urine output

The correct answer is C: Loss of pulse in the extremity

40. A client has a history of chronic obstructive pulmonary disease (COPD). As the nurse enters the client's room, his oxygen is running at 6 liters per minute, his color is flushed and his respirations are 8 per minute. What should the nurse do first? A) Obtain a 12-lead EKG B) Place client in high Fowler's position C) Lower the oxygen rate D) Take baseline vital signs

The correct answer is C: Lower the oxygen rate

12. A nurse is performing CPR on an adult who went into cardiopulmonary arrest. Another nurse enters the room in response to the call. After checking the client's pulse and respirations, what should be the function of the second nurse? A) Relieve the nurse performing CPR B) Go get the code cart C) Participate with the compressions or breathing D) Validate the client's advanced directive

The correct answer is C: Participate with the compressions or breathing

77. A client was just taken off the ventilator after surgery and has a nasogastric tube draining bile colored liquids. Which nursing measure will provide the most comfort to the client? A) Allow the client to melt ice chips in the mouth B) Provide mints to freshen the breath C) Perform frequent oral care with a tooth sponge D) Swab the mouth with glycerin swabs

The correct answer is C: Perform frequent oral care with a tooth sponge

92. A child is admitted to the pediatric unit with a diagnosis of suspected meningococcal meningitis. Which admission orders should the nurse do first? A) Institute seizure precautions B) Monitor neurologic status every hour C) Place in respiratory/secretion precautions D) Cefotaxime IV 50 mg/kg/day divided q6h

The correct answer is C: Place in respiratory/secretion precautions

34. The nurse is performing a physical assessment on a client who just had an endotracheal tube inserted. Which finding would call for immediate action by the nurse? A) Breath sounds can be heard bilaterally B) Mist is visible in the T-Piece C) Pulse oximetry of 88 D) Client is unable to speak

The correct answer is C: Pulse oximetry of 88

42. The nurse is assessing a client 2 hours postoperatively after a femoral popliteal bypass. The upper leg dressing becomes saturated with blood. The nurse's first action should be to A) Wrap the leg with elastic bandages B) Apply pressure at the bleeding site C) Reinforce the dressing and elevate the leg D) Remove the dressings and re-dress the incision

The correct answer is C: Reinforce the dressing and elevate the leg

73. The nurse is planning care for a client with a CVA. Which of the following measures planned by the nurse would be most effective in preventing skin breakdown? A) Place client in the wheelchair for four hours each day B) Pad the bony prominence C) Reposition every two hours D) Massage reddened bony prominence

The correct answer is C: Reposition every two hours

8. A client who is to have antineoplastic chemotherapy tells the nurses of a fear of being sick all the time and wishes to try acupuncture. Which of these beliefs stated by the client would be incorrect about acupuncture? A)Some needles go as deep as 3 inches, depending on where they're placed in the body and what the treatment is for. The needles usually are left in for 15 to 30 minutes. B) In traditional Chinese medicine, imbalances in the basic energetic flow of life — known as qi or chi — are thought to cause illness. * C) The flow of life is believed to flow through major pathways or nerve clusters in your body. D) By inserting extremely fine needles into some of the over 400 acupuncture points in various combinations it is believed that energy flow will rebalance to allow the body's natural healing mechanisms to take over.

The correct answer is C: The flow of life is believed to flow through major pathways or nerve clusters in your body.

53. A client with heart failure has Lanoxin (digoxin) ordered. What would the nurse expect to find when evaluating for the therapeutic effectiveness of this drug? A) diaphoresis with decreased urinary output B) increased heart rate with increase respirations C) improved respiratory status and increased urinary output D) decreased chest pain and decreased blood pressure

The correct answer is C: improved respiratory status and increased urinary output

91. A 20 year-old client has an infected leg wound from a motorcycle accident, and the client has returned home from the hospital. The client is to keep the affected leg elevated and is on contact precautions. The client wants to know if visitors can come. The appropriate response from the home health nurse is that: A) Visitors must wear a mask and a gown B) There are no special requirements for visitors of clients on contact precautions C) Visitors should wash their hands before and after touching the client D) Visitors

The correct answer is C:Visitors should wash their hands before and after touching the client

69. The nurse has been teaching a client with Insulin Dependent Diabetes Mellitus. Which statement by the client indicates a need for further teaching? A) "I use a sliding scale to adjust regular insulin to my sugar level." B) "Since my eyesight is so bad, I ask the nurse to fill several syringes." C) "I keep my regular insulin bottle in the refrigerator." D) "I always make sure to shake the NPH bottle hard to mix it well."

The correct answer is D: "I always make sure to shake the NPH bottle hard to mix it well."

16. A 14 year-old with a history of sickle cell disease is admitted to the hospital with a diagnosis of vaso-occlusive crisis. Which statements by the client would be most indicative of the etiology of this crisis? A)"I knew this would happen. I've been eating too much red meat lately." B)"I really enjoyed my fishing trip yesterday. I caught 2 fish." C)"I have really been working hard practicing with the debate team at school." D)"I went to the health care provider last week for a cold and I have gotten worse."

The correct answer is D: "I went to the doctor last week for a cold and I have gotten worse."

86. After a myocardial infarction, a client is placed on a sodium restricted diet. When the nurse is teaching the client about the diet, which meal plan would be the most appropriate A) 3 oz. broiled fish, 1 baked potato, . cup canned beets, 1 orange, and milk B) 3 oz. canned salmon, fresh broccoli, 1 biscuit, tea, and 1 apple C) A bologna sandwich, fresh eggplant, 2 oz fresh fruit, tea, and apple juice D) 3 oz. turkey, 1 fresh sweet potato, 1/2 cup fresh green beans, milk, and 1 orange

The correct answer is D: 3 oz. turkey, 1 fresh sweet potato, 1/2 cup fresh green beans, milk, and 1 orange

19. Which of these clients who are all in the terminal stage of cancer is least appropriate to suggest the use of patient controlled analgesia (PCA) with a pump? A) A young adult with a history of Down's syndrome B) A teenager who reads at a 4th grade level C) An elderly client with numerous arthritic nodules on the hands D) A preschooler with intermittent episodes of alertness

The correct answer is D: A preschooler with intermittent episodes of alertness

93. Which of these nursing diagnoses of 4 elderly clients would place 1 client at the greatest risk for falls? A) Sensory perceptual alterations related to decreased vision B) Alteration in mobility related to fatigue C) Impaired gas exchange related to retained secretions D) Altered patterns of urinary elimination related to nocturia

The correct answer is D: Altered patterns of urinary elimination related to nocturia

62. A parent asks the school nurse how to eliminate lice from their child. What is the most appropriate response by the nurse? A) Cut the child's hair short to remove the nits B) Apply warm soaks to the head twice daily C) Wash the child's linen and clothing in a bleach solution D) Application of pediculicides

The correct answer is D: Application of pediculicides

38. A nurse is to collect a sputum specimen for acid-fast bacillus (AFB) from a client. Which action should the nurse take first? A) Ask client to cough sputum into container B) Have the client take several deep breaths C) Provide a appropriate specimen container D) Assist with oral hygiene

The correct answer is D: Assist with oral hygiene

71. The nurse is teaching the client to select foods rich in potassium to help prevent digitalis toxicity. Which choice indicates the client understands dietary needs? A) Three apricots B) Medium banana C) Naval orange D) Baked potato

The correct answer is D: Baked potato.

80. Which bed position is preferred for use with a client in an extended care facility on falls risk prevention protocol? A) All 4 side rails up, wheels locked, bed closest to door B) Lower side rails up, bed facing doorway C) Knees bent, head slightly elevated, bed in lowest position D) Bed in lowest position, wheels locked, place bed against wall

The correct answer is D: Bed in lowest position, wheels locked, place bed against wall

97. A client is diagnosed with methicillin resistant staphylococcus aureus pneumonia. What type of isolation is most appropriate for this client? A) Reverse B) Airborne C) Standard precautions D) Contac

The correct answer is D: Contact

44. A client has a chest tube in place following a left lower lobectomy inserted after a stab wound to the chest. When repositioning the client, the nurse notices 200 cc of dark, red fluid flows into the collection chamber of the chest drain. What is the most appropriate nursing action? A) Clamp the chest tube B) Call the surgeon immediately C) Prepare for blood transfusion D) Continue to monitor the rate of drainage

The correct answer is D: Continue to monitor the rate of drainage

25. A middle aged woman talks to the nurse in the health care provider's office about uterine fibroids also called leiomyomas or myomas. What statement by the woman indicates more education is needed? A) I am one out of every 4 women that get fibroids, and of women my age - between the 30s or 40s, fibroids occurs more frequently. B) My fibroids are noncancerous tumors that grow slowly. C) My associated problems I have had are pelvic pressure and pain, urinary incontinence, frequent urination or urine retention and constipation. D) Fibroids that cause no problems still need to be taken out.

The correct answer is D: Fibroids that cause no problems still need to be taken out.

58. A client with amyotrophic lateral sclerosis has a percutaneous endoscopic gastrostomy (PEG) tube for the administration of feedings and medications. Which nursing action is appropriate? A) Pulverize all medications to a powdery condition B) Squeeze the tube before using it to break up stagnant liquids C) Cleanse the skin around the tube daily with hydrogen peroxide D) Flush adequately with water before and after using the tube Skip

The correct answer is D: Flush adequately with water before and after using the tube

21. As the nurse is speaking with a group of teens which of these side effects of chemotherapy for cancer would the nurse expect this group to be more interested in during the discussion? A) Mouth sores B) Fatigue C) Diarrhea D) Hair loss

The correct answer is D: Hair loss

95. A newly admitted adult client has a diagnosis of hepatitis A. The charge nurse should reinforce to the staff members that the most significant routine infection control strategy, in addition to hand washing, to be implemented is which of these? A) Apply appropriate signs outside and inside the room B) Apply a mask with a shield if there is a risk of fluid splash C) Wear a gown to change soiled linens from incontinence D) Have gloves on while handling bedpans with feces

The correct answer is D: Have gloves on while handling bedpans with feces

24. Which of these clients who call the community health clinic would the nurse ask to come in that day to be seen by the health care provider? A) I started my period and now my urine has turned bright red. B) I am an diabetic and today I have been going to the bathroom every hour. C) I was started on medicine yesterday for a urine infection. Now my lower belly hurts when I go to the bathroom. D) I went to the bathroom and my urine looked very red and it didn't hurt when I went.

The correct answer is D: I went to the bathroom and my urine looked very red and it didn't hurt when I went.

1. Which information is a priority for the RN to reinforce to an older client after intravenous pylegraphy? A) Eat a light diet for the rest of the day B) Rest for the next 24 hours since the preparation and the test is tiring. C) During waking hours drink at least 1 8-ounce glass of fluid every hour for the next 2 days D) Measure the urine output for the next day and immediately notify the health care provider if it should decrease.

The correct answer is D: Measure the urine output for the next day and immediately notify the health care provider if it should decrease.

56. A client is recovering from a hip replacement and is taking Tylenol #3 every 3 hours for pain. In checking the client, which finding suggests a side effect of the analgesic? A) Bruising at the operative site B) Elevated heart rate C) Decreased platelet count D) No bowel movement for 3 days Skip

The correct answer is D: No bowel movement for 3 days

60. Although non steroidal anti-inflammatory drugs such as ibuprofen (Motrin) are beneficial in managing arthritis pain, the nurse should caution clients about which of the following common side effects? A) Urinary incontinence B) Constipation C) Nystagmus D) Occult bleeding

The correct answer is D: Occult bleeding

20. The nurse is about to assess a 6 month-old child with nonorganic failure-to thrive (NOFTT). Upon entering the room, the nurse would expect the baby to be A) Irritable and "colicky" with no attempts to pull to standing B) Alert, laughing and playing with a rattle, sitting with support C)Skin color dusky with poor skin turgor over abdomen D) Pale, thin arms and legs, uninterested in surroundings

The correct answer is D: Pale, thin arms and legs, uninterested in surroundings

52. The nurse receives an order to give a client iron by deep injection. The nurse know that the reason for this route is to A) enhance absorption of the medication B) ensure that the entire dose of medication is given C) provide more even distribution of the drug D) prevent the drug from tissue irritation

The correct answer is D: prevent the drug from tissue irritation

35. A nurse checks a client who is on a volume-cycled ventilator. Which finding indicates that the client may need suctioning? A) drowsiness B) complaint of nausea C) pulse rate of 92 D) restlessness

The correct answer is D: restlessness

2. A client has altered renal function and is being treated at home. The nurse recognizes that the most accurate indicator of fluid balance during the weekly visits is A) difference in the intake and output B) changes in the mucous membranes C) skin turgor D) weekly weight

The correct answer is D: weekly weight

A drug is administered to a client in her third trimester of pregnancy. Which statement regarding the drug administration is correct? All drugs should not be given to the pregnant client. The dose of a drug should not be altered for a pregnant client. The dose of a drug should be increased for pregnant clients. The dose of a drug should be decreased for pregnant clients.

The dose of a drug should be increased for pregnant clients. During pregnancy, a client's hepatic metabolism and glomerular filtration are increased. As a result, the excretion rate is faster. Therefore the dose of a drug should be increased for the drug action to be optimal. It is true that all drugs should not be given to the pregnant client, because of the teratogenic effects on the fetus. The client should see her healthcare provider if in doubt. The dose of a drug should be altered for a pregnant client depending on the trimester she is in. How a drug affects the fetus depends on the stage of development of the fetus and the dosage and strength of the drug administered. During the first 3-8 weeks after fertilization, the major organs are developing and the dose of a drug should be decreased or withdrawn for pregnant clients.

A nurse is caring for a 3-month-old infant with severe diarrhea following antibiotic therapy. After the effects of dehydration are stabilized, the healthcare provider prescribes Lactobacillus granules. What explanation does the nurse give to the infant's parents about the reason for giving lactobacilli? They diminish the inflammatory mucosal edema. The discomfort caused by gastric hyperacidity is lessened. They relieve the pain caused by gas in the gastrointestinal tract. The flora that inhabit a healthy gastrointestinal tract must be recolonized.

The flora that inhabit a healthy gastrointestinal tract must be recolonized. Lactobacilli are part of the flora in the healthy gastrointestinal tract. The purpose of administering lactobacilli granules is to help recolonize the normal gastrointestinal flora that were destroyed with antibiotic therapy. The other options are not the actions of lactobacilli granules.

What intrauterine medication exposure may lead to the child being born with a cleft lip? The mother was on phenytoin therapy. The mother was on multivitamin therapy. The mother was taking methotrexate. The mother was on nitrofurantoin therapy.

The mother was on nitrofurantoin therapy. Nitrofurantoin should not be administered during pregnancy because this drug may cause abnormally small or absent eyes, heart defects, or a cleft lip with a cleft palate. Phenytoin, multivitamins, and methotrexate do not cause a cleft lip.

A neonate diagnosed with congestive heart failure has been prescribed furosemide. What changes to the dosage or time intervals between doses should be made? The time between doses should be shortened. The time between doses should be lengthened. The dosage should be doubled. The dosage should be cut in half.

The time between doses should be lengthened. In neonates, the half-life of furosemide is increased. To avoid toxicity of the drug, the nurse should lengthen the time interval between the doses. If the time interval is shortened or the dosage is doubled, the level of drug circulating in the blood will be increased leading to toxic effects of the drug. Halving the dose is not an appropriate solution.

A client who has a habit of smoking is on estrogen therapy. What condition is the client at risk of developing? Thrombosis Gastrointestinal upset Risk of developing cancer Decreased effectiveness of estrogen

Thrombosis Smoking along with estrogen therapy may increase the risk of thrombosis. Estrogen taken on an empty stomach may cause gastrointestinal upset. Estrogen is not prescribed to clients with endometrial or breast cancer. The effectiveness of estrogen decreases with the use of anticoagulants, rifampicin, and St. John wort.

A client with cancer of the thyroid is scheduled for a thyroidectomy. What should the nurse teach the client? The dietary intake of carbohydrates must be restricted. Thyroxine replacement therapy will be required indefinitely. Chemotherapy may be used in conjunction with the surgery. A tracheostomy requires an alternative means of communication.

Thyroxine replacement therapy will be required indefinitely. Thyroxine is given postoperatively to suppress thyroid-stimulating hormone (TSH) and prevent hypothyroidism. Increased intake of carbohydrates and proteins is needed because of the increased metabolic activity associated with hyperthyroidism. Chemotherapy is uncommon; radiation may be used to eradicate remaining tissue. A tracheostomy is not planned; it is needed only in an emergency related to respiratory distress.

Why would a client with acquired immunodeficiency syndrome (AIDS) be prescribed diphenoxylate hydrochloride? To manage pain To manage diarrhea To manage candidal esophagitis To manage behavioral problems

To manage diarrhea Diphenoxylate hydrochloride is an antidiarrheal drug prescribed to clients with AIDS to manage frequent diarrhea experienced by a client with AIDS. Opioid analgesics such as tramadol are used to manage pain. Ketoconazole can be used to treat candidal esophagitis associated with AIDS. Behavioral problems are managed with psychotropic drugs.

The primary healthcare provider prescribed zoledronic acid to a client with osteoporosis. Before administration, the nurse instructs the client to have a dental examination. What is the rationale for this nursing intervention? To prevent kidney failure To prevent atrial fibrillation To prevent bronchoconstriction To prevent maxillary osteonecrosis

To prevent maxillary osteonecrosis Zoledronic acid is a bisphosphonate used to treat osteoporosis that can cause maxillary osteonecrosis. The client should have a dental examination before starting the drug therapy to prevent maxillary osteonecrosis. The client's serum creatinine should be checked before and after administration of the drug to prevent kidney failure. To prevent atrial fibrillation, the drug should not be infused too quickly. The drug should not be given to a client who is sensitive to aspirin as it may cause bronchoconstriction.

A client with gastric ulcer disease asks the nurse why the health care provider has prescribed metronidazole. What purpose does the nurse provide? To augment the immune response To potentiate the effect of antacids To treat Helicobacter pylori infection To reduce hydrochloric acid secretion

To treat Helicobacter pylori infection Approximately two thirds of clients with peptic ulcer disease are found to have Helicobacter pylori infecting the mucosa and interfering with its protective function. Antibiotics do not augment the immune response, potentiate the effect of antacids, or reduce hydrochloric acid secretion.

What potentially dangerous adverse effect of an intravenous titrated drip of lidocaine should the nurse immediately report to the healthcare provider? Tremors Anorexia Tachycardia Hypertension

Tremors Tremors are a precursor to the major adverse effect of seizures. Although anorexia may occur, it is not a dangerous side effect. Bradycardia, which may lead to heart block, may occur, not tachycardia. Hypotension, not hypertension, may occur.

A client is prescribed albuterol to relieve severe asthma. What adverse effect will the nurse instruct the client to anticipate? Tremors Lethargy Palpitations Visual disturbances Decreased pulse rate

Tremors Palpitations Albuterol's sympathomimetic effect causes central nervous stimulation, precipitating tremors, restlessness, and anxiety. Albuterol's sympathomimetic effect causes cardiac stimulation that may result in tachycardia and palpitations. Albuterol may cause restlessness, irritability, and tremors, not lethargy. Albuterol may cause dizziness, not visual disturbances. Albuterol will cause tachycardia, not bradycardia.

A client has been receiving fluphenazine for several months. The nurse will assess the client for which side effects? Tremors Excess salivation Rambling speech Reluctance to converse Uncoordinated movement of extremities

Tremors Uncoordinated movement of extremities Acute dystonic reactions such as tremors, dyskinesia, and akathisia are observable side effects of fluphenazine therapy. There is a decrease, not an increase, in salivation with fluphenazine therapy. Rambling speech is not a side effect of this drug; nor is reluctance to converse.

A client is receiving an oxytocin infusion for induction of labor. The uterine graph on the electronic monitor indicates no rest period between contractions, and this is confirmed on palpation. What should the nurse do first? Evaluate the fetal heart rate. Turn the oxytocin infusion off. Place the client in the left-lateral position. Prepare the client for an emergency birth.

Turn the oxytocin infusion off. Because contractions are induced by the oxytocin, stopping the infusion should stop contractions and increase uteroplacental perfusion. Although evaluating the fetus's response is important, it is not the priority. Although placing the client in the left-lateral position is important, it is not the priority. There are no data to indicate that the client is ready to give birth.

During labor a client who has been receiving epidural anesthesia has a sudden episode of severe nausea, and her skin becomes pale and clammy. What is the nurse's immediate reaction? Turning the client on her side Notifying the primary healthcare provider Checking the vaginal area for bleeding Checking the fetal heart rate every 3 minutes

Turning the client on her side Maternal hypotension is a common complication of epidural anesthesia during labor, and nausea is one of the first clues that it has occurred. Turning the client on her side will keep the uterus from putting pressure on the inferior vena cava, which causes a decrease in blood flow. If signs and symptoms do not abate after the client is turned on her side, the primary healthcare provider should be notified. Checking the vaginal area for bleeding is not an assessment specific to epidural anesthesia; it is part of the general nursing care during labor. Fetal heart rate monitoring is a continuous process, and the rate should be recorded every 15 minutes; if this monitoring is not being performed, the rate should be checked and recorded every 15 minutes.

A nurse administers an antipsychotic medication to a client. The nurse will assess the client for which common manageable side effect? Jaundice Melanocytosis Drooping eyelids Unintentional tremor

Unintentional tremor Unintentional tremor is one of the extrapyramidal side effects of antipsychotic medications; it is considered common and manageable. Jaundice is a severe but not a common occurrence; periodic liver function tests should be performed. An excessive number of melanocytes is not a side effect of antipsychotics. Drooping of the eyelids is not a common side effect.

A 2-year-old boy with hemophilia A is to start receiving prophylactic intravenous infusions of the recombinant form of factor VIII three times a week. The nurse will instruct the parents to administer the factor at what time on the designated days? At bedtime After lunch Before dinner Upon awakening

Upon awakening Factor VIII is administered once in the morning on designated days. The half-life of factor VIII is short. If factor VIII is administered later in the day (i.e., at bedtime, after lunch, or before dinner), protection will not be adequate during the day, when the child is most active and more vulnerable to bleeding.

A nurse is planning to administer a prescribed intravenous solution that contains potassium chloride. What assessment should be brought to the healthcare provider's attention before administration of the intravenous line? Uncharacteristic irritability Poor tissue turgor with tenting Urinary output of 200 mL during the previous 8 hours Oral fluid intake of 300 mL during the previous 12 hours

Urinary output of 200 mL during the previous 8 hours Decreased urinary output will result in the retention of potassium, causing hyperkalemia. Reporting uncharacteristic irritability is unnecessary; this is a sign of dehydration, which can be corrected with appropriate hydration. Reporting poor tissue turgor with tenting is unnecessary; this may indicate dehydration, which is probably the rationale for the fluid prescribed. Reporting an oral fluid intake of 300 mL during the previous 12 hours is unnecessary; this can precipitate dehydration or can compound an existing dehydration, which can be treated with appropriate hydration.

A nurse is caring for a client who reports urinary problems, and the healthcare provider prescribes a cholinergic medication. Which urinary problem will this medication correct? Urinary frequency due to bladder spasticity Urinary retention due to bladder atony Pain due to urinary tract calculi Urinary urgency due to urinary tract infections

Urinary retention due to bladder atony Cholinergics intensify and prolong the action of acetylcholine, which increases tone in the genitourinary tract, preventing urinary retention. Anticholinergics are prescribed for frequency and urgency associated with a spastic bladder. Cholinergics will not prevent renal calculi. Urinary tract infections are a secondary gain because cholinergics help prevent urinary retention that can lead to urinary tract infection, but this is not the primary purpose for administering a cholinergic.

A preterm infant is started on digoxin and furosemide for persistent patent ductus arteriosus. Which nursing assessment provides the best indication of the effectiveness of the furosemide? Pedal edema is reduced. Digoxin toxicity is prevented. Fontanels appear depressed. Urine output exceeds fluid intake.

Urine output exceeds fluid intake. Urine output exceeding fluid intake is the expected outcome. Output exceeding intake indicates that furosemide is causing diuresis. Although it is important to determine whether pedal edema is reduced, this could be influenced by other factors. Furosemide can cause hypokalemia, which may precipitate digoxin toxicity; it is not given to prevent digoxin toxicity. Depressed fontanels are not the desired outcome; this finding indicates dehydration, which may occur with excessive diuresis.

A client is receiving intravenous mannitol after sustaining a critical head injury. What assessment will the nurse perform that is specific to the safe administration of the medication? Body weight daily Urine output hourly Vital signs every 2 hours Level of consciousness every 8 hours

Urine output hourly Mannitol, an osmotic diuretic, increases the intravascular volume that must be excreted by the kidneys. The client's urine output should be monitored hourly to determine the client's response to therapy. Although with mannitol there is an increase in urinary excretion that is reflected in a decrease in body weight (1 L of fluid is equal to 2.2 pounds [1 kg]), a daily assessment of the client's weight is too infrequent to assess the client's response to therapy. Urine output can be monitored hourly and is a more frequent, accurate, and efficient assessment than is a daily weight. Vital signs should be monitored every hour considering the severity of the client's injury and the administration of mannitol. Although the level of consciousness should be monitored with a head injury, assessments every 8 hours are too infrequent to monitor the client's response to therapy.

Doxepin is prescribed for a 74-year-old client for treatment of a depressive episode. When the nurse reviews with the client the side effects of doxepin, which effects does the nurse include? Diarrhea Loss of appetite Photosensitivity Urine retention Suicidal ideation

Urine retention Suicidal ideation Doxepin, because of its significant anticholinergic properties, can lead to urine retention, particularly in older men. Doxepin may cause an increase in psychiatric symptoms and precipitate suicidal ideation. Doxepin may cause constipation, not diarrhea. Doxepin may cause an increase, not a decrease, in appetite. Although photosensitivity is a side effect of doxepin, it can be managed through nursing interventions such as avoiding the sun, wearing clothing, and using sunblock.

A client develops severe bone marrow suppression related to cancer treatment. What is important for the nurse to include in the client's teaching? Be prepared to experience alopecia. Increase fluids to at least 3 liters/day. Use a soft toothbrush for oral hygiene. Monitor your intake and output of fluids.

Use a soft toothbrush for oral hygiene. Thrombocytopenia occurs with several cancer treatment programs; using a soft toothbrush helps prevent bleeding gums. Although alopecia does occur, it is not related to bone marrow suppression. Increasing fluids will neither reverse bone marrow suppression nor stimulate hematopoiesis. Monitoring intake and output of fluids is not related to bone marrow suppression.

A client was treated with sex hormones during pregnancy. Which teratogenic effects may be seen in the newborn if the client gave birth to a baby girl? Ebstein anomaly Vaginal carcinoma Facial malformations Masculinization of the female fetus Reproductive organ defects

Vaginal carcinoma Masculinization of the female fetus Reproductive organ defects Vaginal carcinoma, masculinization of the female fetus, and reproductive organ defects are teratogenic effects that may be seen in a female newborn if the mother has a medication history of sex hormones during pregnancy. Exposure to lithium may cause Ebstein anomaly in the newborn. HMG-CoA reductase inhibitors may cause facial malformations in the fetus.

A client with bleeding esophageal varices is to be treated via infusion of medication through an intravenous line. Which medication should the nurse anticipate will be prescribed? Vasopressin Neostigmine Lansoprazole Phytonadione

Vasopressin Vasopressin is a vasoconstrictor that can be used to control gastrointestinal bleeding. Lansoprazole is a proton pump inhibitor that is used for the treatment of gastric and duodenal ulcers. Neostigmine inhibits cholinesterase, permitting acetylcholine to function; it is used primarily for myasthenia gravis. Phytonadione is vitamin K; it promotes formation of prothrombin in the liver. Although this medication may be helpful, its effects take too long to be of value in an emergency situation.

The medication prescribed for an infant is to be given intramuscularly. Which site will the nurse select for administration of the medication? Vastus lateralis Ventrogluteal Dorsogluteal Deltoid

Vastus lateralis Intramuscular injections are given in the vastus lateralis muscle of the thigh in infants. The ventrogluteal site is not used until children have been walking. The dorsogluteal site is considered high risk for damage to the sciatic nerve or a major blood vessel. The deltoid site in the arm has a small muscle mass that limits the amount of medication that can be injected.

A nurse plans to administer vitamin K to a newborn. What site should the nurse use for the injection? Deltoid muscle Rectus femoris Vastus lateralis Gluteus maximus

Vastus lateralis The vastus lateralis is the most appropriate muscle for a newborn's intramuscular injection because it is well developed and there is little danger of nerve injury. The deltoid muscle is too small for a newborn's intramuscular injection. The rectus femoris muscle is not used; it is not as large as the vastus lateralis in a newborn. The sciatic nerve in the newborn is near the outer aspect of the gluteus maximus and might be injured if this site were used for an injection.

Which drug will most likely cause the client's eyelids to itch and eyes to burn as side effects? Ketorolac Ofloxacin Diclofenac Vidarabine

Vidarabine Vidarabine is an antiviral agent that causes sensitive reactions such as itching eyelids and burning eyes. Ketorolac, ofloxacin, and diclofenac usually do not cause sensitive reactions such as itching eyes.

A nurse in the surgical intensive care unit is caring for a client with a large surgical incision. The nurse reviews a list of vitamins and expects that which medication may be prescribed because of its major role in wound healing? Vitamin A (retinol) Vitamin K (phytonadione) Vitamin C (ascorbic acid) Vitamin B 12 (cyanocobalamin)

Vitamin C (ascorbic acid) Vitamin C (ascorbic acid) plays a major role in wound healing. It is necessary for the maintenance and formation of collagen, the major protein of most connective tissues. Vitamin A is important for the healing process; however, vitamin C is the priority because it cements the ground substance of supportive tissue. Cyanocobalamin is a vitamin B 12 preparation needed for red blood cell synthesis and a healthy nervous system. Phytonadione is vitamin K, which plays a major role in blood coagulation.

A client with chronic liver disease reports, "My gums have been bleeding spontaneously." The nurse identifies small hemorrhagic lesions on the client's face. The nurse concludes that the client needs which additional supplement? Bile salts Folic acid Vitamin A Vitamin K

Vitamin K Fat-soluble vitamin K is essential for synthesis of prothrombin by the liver; a lack results in hypoprothrombinemia, inadequate coagulation, and hemorrhage. Although cirrhosis may interfere with production of bile, which contains the bilirubin needed for optimum absorption of vitamin K, the best and quickest manner to counteract the bleeding is to provide vitamin K intramuscularly. Folic acid is a coenzyme with vitamins B 12 and C in the formation of nucleic acids and heme; thus, a deficiency may lead to anemia, not bleeding. Vitamin A deficiency contributes to the development of polyneuritis and beriberi, not hemorrhage.

What should the nurse expect the healthcare provider to prescribe if a client exhibits clinical indicators of warfarin overdose? Heparin Vitamin K Iron dextran Protamine sulfate

Vitamin K Warfarin depresses prothrombin activity and inhibits formation of several clotting factors by the liver. Its antagonist is vitamin K, which is involved in prothrombin formation. Heparin is an anticoagulant. Iron dextran is an iron supplement, not an antidote for warfarin. Protamine sulfate is the antidote for heparin overdose.

A nurse is monitoring a 6-year-old child for toxicity precipitated by digoxin. For what sign of digoxin toxicity will the nurse assess the child? Oliguria Vomiting Tachypnea Splenomegaly

Vomiting Vomiting is a sign of digoxin toxicity in children. Oliguria is associated with renal failure, not toxicity. Tachypnea is associated with heart failure, not toxicity. Splenomegaly is associated with heart failure, specifically right ventricular failure.

Warfarin is prescribed for the client who takes phenytoin for a seizure disorder. Why must the nurse observe the client closely during the initial days of treatment with warfarin? Warfarin affects the metabolism of phenytoin. Phenytoin decreases warfarin's anticoagulant effect. Warfarin's action is greater in clients with seizure disorders. Seizures increase the metabolic degradation rate of warfarin.

Warfarin affects the metabolism of phenytoin. Warfarin has been shown to inhibit metabolism of phenytoin, which results in an accumulation of phenytoin in the body. Warfarin potentiates the anticoagulant effect of heparin. That warfarin's action is greater in clients with seizure disorders is true only if the client is receiving phenytoin to control the seizure disorder. Seizures do not have a significant effect on the metabolism of warfarin.

The nurse is caring for a 6-year-old child admitted from the emergency department after an acute asthma attack. The child has a new prescription for fluticasone. What instructions must the family be given about this drug before the child's discharge? Multiple choice question Fluticasone needs to be taken with food or milk. Fluticasone is primarily used to treat acute asthma attacks. The child should suck on hard candy to help relieve dry mouth. Watch for white patches in the mouth and report them to the primary healthcare provider.

Watch for white patches in the mouth and report them to the primary healthcare provider. Fluticasone is a steroid commonly administered by way of inhalation for long-term control of asthma symptoms. Oral thrush is a side effect that manifests as white patches. Fluticasone is administered via inhalation so food or milk is not needed prior to administration. Dry mouth is not a side effect of fluticasone.

A client has been receiving oxytocin to augment labor. For what adverse reaction caused by a prolonged oxytocin infusion should the nurse monitor the client? Change in affect Hyperventilation Water intoxication Increased temperature

Water intoxication Oxytocin, a posterior pituitary hormone, has an antidiuretic effect, acting to reabsorb water from the glomerular filtrate. Affect is not altered by oxytocin. Hyperventilation is caused by an inappropriate breathing pattern, not by prolonged use of oxytocin. Fever occurs with infection or dehydration, not with prolonged use of oxytocin.

A client who has a long leg cast for a fractured bone is to be discharged from the emergency department. When discussing pain management, when does the nurse advise the client to take the prescribed as-needed oxycodone? Just as a last resort Before going to sleep As the pain becomes intense When the discomfort begins

When the discomfort begins Pain is most effectively relieved when an analgesic is administered at the onset of pain, before it becomes intense; this prevents a pain cycle from occurring. Analgesics are less effective if administered when pain is at its peak. Before going to sleep, it may or may not be necessary; the medication should be taken when the client begins to feel uncomfortable within the parameters specified by the healthcare provider's prescription. Analgesics are less effective if administered when pain is at its peak.

A client is taking lithium sodium. The nurse should notify the healthcare provider for which laboratory value? Negative protein in the urine Prothrombin of 12.0 seconds Blood urea nitrogen (BUN) of 20 mg/dL (7.1 mmol/L) White blood cell (WBC) count of 15,000 mm 3 (15 X 10 9/L)

White blood cell (WBC) count of 15,000 mm 3 (15 X 10 9/L) White cell counts can increase with this drug. The expected range of the WBC count is 5000 to 10,000 mm 3 (5-10 X 10 9/L) for a healthy adult. Urinalysis, BUN, and prothrombin are not necessary, and these are normal values.

The nurse provides teaching to a client who has received a prescription for oral pancreatic enzymes, pancrelipase. The nurse evaluates that teaching is understood when the client identifies which time for medication scheduling? At bedtime With meals One hour before meals On arising each morning

With meals The pancreatic enzymes (amylase, trypsin, and lipase) must be present when food is ingested for digestion to take place. At bedtime the food eaten for dinner has passed beyond the duodenum; at bedtime the enzyme is given too late to aid digestion. Taking pancrelipase one hour before meals or on arising each morning will have no chyme in the duodenum on which the enzyme can act.

After 3 months of supplemental oral iron therapy, there is no significant increase in an adolescent's hemoglobin level. Iron dextran is prescribed. What is the best way for the nurse to administer this medication? With a transdermal needle By massaging the injection site With the use of the Z-track method By administering a local anesthetic first

With the use of the Z-track method The Z-track injection method prevents seepage of iron dextran through the needle track, thereby limiting irritation of subcutaneous tissue and staining of the skin. The length of a transdermal needle is too short to reach a muscle; a 1.5-inch (3.8 centimeters) needle is required. Massage will force iron dextran into the subcutaneous tissue, causing irritation and staining. Although an injection may be uncomfortable, a local anesthetic is unnecessary.

A client with depression was prescribed fluoxetine. After two days, the client arrives at the hospital and reports restlessness, confusion, and poor concentration. Upon assessment, the nurse finds an elevated body temperature. Which intervention by the healthcare provider would be beneficial to the client? Withdrawing the drug Administering isocarboxazid Reducing the dose of the drug Informing the client that these are expected side effects

Withdrawing the drug Restlessness, confusion, poor concentration, and fever are symptoms of serotonin syndrome. The only treatment for serotonin syndrome is discontinuation of the drug. Isocarboxazid is a monoamine oxidase inhibitor that should not be used in a client with serotonin syndrome because it may lead to life-threatening conditions. Reducing the drug dosage may not reverse the symptoms completely. Informing the client that these are expected adverse effects is important, but the drug should be discontinued immediately.

While receiving an adrenergic beta 2 agonist drug for asthma, the client complains of palpitations, chest pain, and a throbbing headache. What is the most appropriate nursing action? Withhold the drug and notify the healthcare provider. Tell the client not to worry; these are expected side effects from the medicine. Give instructions to breathe slowly and deeply for several minutes. Explain that the effects are temporary and will subside as the body becomes accustomed to the drug.

Withhold the drug and notify the healthcare provider. These drugs cause increased heart contraction (positive inotropic effect) and increased heart rate (positive chronotropic effect). If toxic levels are reached, side effects occur, and the drug should be withheld until the healthcare provider is notified. Telling the client not to worry and that these are expected side effects from the medicine is false reassurance and a false statement. Controlled breathing may be helpful in allaying a client's anxiety; however, the drug may be producing adverse effects and should be withheld.

During a prenatal visit a nurse explains to a client who is Rh negative when Rho(D) immune globulin will be administered to her. When is the best time to administer Rho(D) immune globulin? Within 72 hours of birth if the infant is Rh positive Weekly during the ninth month if the mother is a multipara Immediately after birth if the infant's Coombs test result is positive Always during the second trimester

Within 72 hours of birth if the infant is Rh positive Rho(D) immune globulin is given to an Rh-negative mother within 72 hours after birth if the infant is Rh positive and the mother was not previously sensitized. Rho(D) immune globulin is administered once after birth if the mother was not previously sensitized. The infant's Coombs test result does not influence the timing of administration. A small dose may be given prophylactically in the twenty-eighth week of gestation if there is a minimal increase in the antibody titer. If there is a significant increase in the antibody titer, amniocentesis is performed. Treatment of the fetus depends on the results of the amniocentesis.

A client who has schizophrenia is receiving a phenothiazine antipsychotic medication. Which serious client responses to the medication will the nurse immediately report to the primary healthcare provider? Akathisia Shuffling gait Yellow sclerae Photosensitivity Involuntary tongue movements

Yellow sclerae Involuntary tongue movements Yellow sclerae are a sign of toxicity that has damaged the liver and necessitates withholding the drug. Abnormal movements of involuntary muscle groups, particularly of the face, mouth, tongue, fingers, and toes, can occur after a prolonged period of dopamine blockade. Conversion to an atypical antipsychotic is warranted. Akathisia and a shuffling gait are common side effects that usually are alleviated by antiparkinsonian agents. Photosensitivity is an expected side effect of the drug; the medication does not have to be withheld.

Which is a leukotriene modifier used to manage and prevent allergic rhinitis? Zileuton Ephedrine Scopolamine Cromolyn sodium

Zileuton Zileuton is a leukotriene modifier used to manage and prevent allergic rhinitis. Ephedrine is an ingredient in decongestants used to treat allergic rhinitis. Scopolamine is an anticholinergic used to reduce secretions. Cromolyn sodium is a mast cell stabilizing drug used to prevent mast cell membranes from opening when an allergen binds to IgE.

A nurse cares for a client suspected of having atrial fibrillation. After reviewing the client's medical history, the nurse suspects that the medication may have been administered at a fast rate. Which medication might have caused this condition? Citracal Raloxifene Alendronate Zoledronic acid

Zoledronic acid Zoledronic acid may cause atrial fibrillation when it is infused too quickly into the body. Citracal may cause hypercalcemia, cardiac dysrhythmias, and urinary stones. Raloxifene may increase the risk of liver disease and liver thromboembolism. Alendronate may cause esophagitis, esophageal ulcers, and gastric ulcers.

Which cytokine is used to treat multiple sclerosis? β-Interferon Interleukin-2 Erythropoietin Colony-stimulating factor

β-Interferon β-Interferon is a cytokine used to treat multiple sclerosis. Interleukin-2 is used to treat metastatic melanoma. Erythropoietin is a cytokine used to treat anemia related to chemotherapy. Colony-stimulating factor is a cytokine used to treat chemotherapy-induced neutropenia.


Conjuntos de estudio relacionados

CompTIA Security+ Final Assessment (40)

View Set

Ch. 5: Foundations of Employee Motivation (power points)

View Set

HIST-2306 Texas Government Chapter 7 Questions

View Set

Management Information Systems Chapter 4 The Fulfillment Process

View Set